Science Questions

12th Std Science Lesson Wise Questions in English – Part 3

12th Science Lesson 14 Questions in English

14] Human Health and Diseases

1. Which of the following statement is correct?

1)The World Health Organization [WHO] defines health as ‘a state of complete physical, mental and social wellbeing and not merely absence of disease’.

2) Personal hygiene, regular exercise and balanced diet are very important to maintain good health

a)1 alone

b)2 alone

c)1, 2

d)None

Explanation

The World Health Organization [WHO] defines health as ‘a state of complete physical, mental and social wellbeing and not merely absence of disease’. We can also say “HEALTH IS WEALTH”, when people are healthy they are more efficient at work. Health increases longevity of people and reduces infant and adult mortality. Personal hygiene, regular exercise and balanced diet are very important to maintain good health.

2. Which of the following statement is correct?

1.Disease can be defined as a disorder or malfunction of the mind or body

2.It does not involve morphological, physiological and psychological disturbances

3.Diseases can be broadly grouped into infectious and non infectious types

a)1, 2

b)1, 3

c)2, 3

d)All the above

Explanation

Disease can be defined as a disorder or malfunction of the mind or body. It involves morphological, physiological and psychological disturbances which may be due to environmental factors or pathogens or genetic anomalies or life style changes. Diseases can be broadly grouped into infectious and non-infectious types.

3. Diseases which are transmitted from one person to another are called____

a)Infectious diseases

b)Non-infectious diseases

c)Communicable diseases

d)Either a or c

Explanation

Diseases which are transmitted from one person to another are called infectious diseases or communicable diseases. Such disease causing organisms are called pathogens and are transmitted through air, water, food, physical contact and vectors.

5. Which of the following are examples of peripheral lymphoid organs?

1.Spleen

2.Tonsils

3.Adenoids

a)1, 2

b)1, 3

c)2, 3

d)All the above

Explanation

In secondary or peripheral lymphoid organs, antigen is localized so that it can be effectively exposed to mature lymphocytes. The best examples are lymph nodes, appendix, Peyer’s patches of gastrointestinal tract, tonsils, adenoids, spleen, MALT (Mucosal-Associated Lymphoid Tissue), GALT (Gut-Associated Lymphoid Tissue), BALT (Bronchial/Tracheal-Associated Lymphoid Tissue).

6. ____ is a secondary lymphoid organ located in upper part of abdominal cavity close to diaphragm

a)Tonsils

b)Spleen

c)Adenoids

d)Stomach

Explanation

Spleen is a secondary lymphoid organ located in the upper part of the abdominal cavity close to the diaphragm. Spleen contains B and T cells. It brings humoral and cell mediated immunity

6. Which of the following statement is correct?

1.The disease-causing pathogen may be virus, bacteria, fungi, protozoan parasites, helminthic parasites, etc

2.Most of the bacterial diseases are curable but all viral diseases are not

3.Some infectious disease like AIDS may be fatal.

a)1, 2

b)1, 3

c)2, 3

d)All the above

Explanation

The disease causing pathogen may be virus, bacteria, fungi, protozoan parasites, helminthic parasites, etc., Infectious diseases are common and everyone suffers from such diseases at some time or the other. Most of the bacterial diseases are curable but all viral diseases are not. Some infectious disease like AIDS may be fatal.

7. Which of the following is not a non-infectious disease?

a)Heart attack

b)Cystic fibrosis

c)Covid-19

d)Stroke

Explanation

Non-infectious diseases are not transmitted from an infected person to a healthy person. In origin they may be genetic (cystic fibrosis), nutritional (vitamin deficiency diseases) and degenerative (arthritis, heart attack, stroke).

8. Match the following

A.Bacterial diseases 1. Ascariasis

B.Viral diseases 2. Candidiasis

C.Fungal diseases 3. Mumps

Helminthic diseases 4. Cholera

  1. 3, 1, 2, 4
  2. 4, 3, 2, 1
  3. 4, 2, 3, 1
  4. 1, 3, 4, 2

Explanation

9. Bacteria are associated with human diseases are called_____

  1. Pathogenic bacteria
  2. Urogenic bacteria
  3. Useful bacteria
  4. Beneficial bacteria

Explanation

Though the number of bacterial species is very high, only a few bacteria are associated with human diseases and are called pathogenic bacteria. Such pathogens may emit toxins and affects the body.

10. Which of the following statement is incorrect?

1.If an antibiotic is used too often to fight a specific bacterial infection, the bacteria may become resistant to the specific antibiotic

2.That specific antibiotic can no longer be used to treat the bacterial infection

  1. 1 alone
  2. 2 alone
  3. 1, 2
  4. None

Explanation

If an antibiotic is used too often to fight a specific bacterial infection, the bacteria may become resistant to the specific antibiotic. Hence the specific antibiotic can no longer be used to treat the bacterial infection. Some bacteria have developed resistance to many antibiotics. Therefore, infections caused by these bacteria are difficult to be cured.

11. Which of the following are the ways for Risk of bacterial resistance?

1.Avoid using antibiotics to treat minor infections that can be taken care by our immune system.

2.Do not use an antibiotic to treat viral infections such as common cold or flu.

3.Always follow the prescription. Skipping doses or failing to complete the prescription may allow antibiotic resistance to develop

  1. 1, 2
  2. 1, 3
  3. 2, 3
  4. All the above

Explanation

Risk of bacterial resistance can be reduced by observing the following steps:

Avoid using antibiotics to treat minor infections that can be taken care by our immune system. Do not use an antibiotic to treat viral infections such as common cold or flu.

Always follow the prescription. Skipping doses or failing to complete the prescription may allow antibiotic resistance to develop

12. Match the following

A.Shigellosis 1. Larynx

B.Bubonic plague 2. Intestine

C.Diphtheria 3. Lymph node

D.Tetanus 4. Spasm of muscles

  1. 3, 1, 2, 4
  2. 2, 3, 1, 4
  3. 3, 2, 4, 1
  4. 4, 2, 1, 3

Explanation

13. Match the following

A.Typhoid 1. Mycobacterium tuberculosis

B.Pneumonia 2. Salmonella typhi

C.Tuberculosis 3. Streptococcus pneumoniae

  1. 1, 3, 2
  2. 2, 3, 1
  3. 2, 1, 3
  4. 1, 2, 3

Explanation

14. Common cold is caused by more than____ different strains of Rhino viruses

  1. 300
  2. 150
  3. 100
  4. 200

Explanation

Common cold is caused by more than 150 different strains of Rhino viruses. More over their RNA genome keeps changing due to mutation. Hence it is very difficult to prepare a common vaccine for the disease.

15. Typhoid fever can be confirmed by_____

  1. ELISA
  2. Western blot
  3. Widal test
  4. Dimer test

Explanation

Bacteria spread through air, water or by inhaling the droplets/aerosols or even by sharing utensils, dresses with an infected person. Typhoid fever can be confirmed by Widal test.

16. Which of the following statement is correct?

1.Viruses are the smallest intracellular obligate parasites

2.They multiply within living cells

3.Outside the living cells they cannot carry out the characteristics of a living organism

  1. 1 alone
  2. 1, 2
  3. 1, 3
  4. All the above

Explanation

Viruses are the smallest intracellular obligate parasites, which multiply within living cells. Outside the living cells they cannot carry out the characteristics of a living organism.

17. Match the following

A.Common cold 1. Skin and blood

B.Viral hepatitis 2. Liver

C.Poliomyelitis 3. Intestine

D.Dengue fever 4. Respiratory tract

  1. 2, 1, 3, 4
  2. 4, 2, 3, 1
  3. 3, 2, 1, 4
  4. 4, 2, 1, 3

Explanation

`

18. Which of the following are the symptoms of Chikungunya?

1.Fever

2.Dysentery

3.Joint swelling

4.Joint pain

  1. 1, 2, 4
  2. 2, 3, 4
  3. 1, 2, 3
  4. 1, 3, 4

Explanation

19. Nipah virus transmitted from______ to_____

  1. Animal, Human
  2. Human, animal
  3. Plant, animal
  4. Animal, plant

Explanation

Nipah virus is a zoonotic virus (transmitted from animals to humans) and also transmitted through contaminated food. In infected people, it causes a range of illness from asymptomatic infection to acute respiratory illness and fatal encephalitis.

20. Which of the following statement is correct?

1.Viruses invade living cells, forcing the cells to create new viruses

2.The new viruses break out of the cell, killing it and invade other cells in the body, causing diseases in human beings

3.Nipah viruses cause one of the most infectious human ailment called the “Common cold”.

  1. 1, 2
  2. 1, 3
  3. 2, 3
  4. All the above

Explanation

Outside the living cells they cannot carry out the characteristics of a living organism. Viruses invade living cells, forcing the cells to create new viruses. The new viruses break out of the cell, killing it and invade other cells in the body, causing diseases in human beings. Rhino viruses cause one of the most infectious human-ailment called the “Common cold.

21. Match the following

A.Pneumotropic diseases 1. Yellow fever

B.Dermotropic diseases 2. Rabies

C.Viscerotropic diseases 3. Influenza

D.Neurotropic diseases 4. Chicken pox

  1. 2, 1, 3, 4
  2. 3, 4, 1, 2
  3. 3, 1, 2, 4
  4. 4, 2, 1, 3

Explanation

Viral diseases are generally grouped into four types on the basis of the symptoms produced in the body organs.

Pneumotropic diseases (respiratory tract infected by influenza)

Dermotropic diseases (skin and subcutaneous tissues affected by chicken pox and measles)

Viscerotropic diseases (blood and visceral organs affected by yellow fever and dengue fever)

Neurotropic diseases (central nervous system affected by rabies and polio).

22. About____ genera of protozoans live as parasites within the human body

  1. 20
  2. 15
  3. 35
  4. 25

Explanation

About 15 genera of protozoans live as parasites within the human body and cause diseases.

23. Swine flu was first recognised in the_____ pandemic

  1. 1919
  2. 1920
  3. 1935
  4. 2012

Explanation

Swine flu was first recognised in the 1919 pandemic and still circulates as a seasonal flu virus. Swine flu is caused by the H1N1 virus strain. Symptoms include fever, cough, sore throat, chills, weakness and body aches. Children, pregnant women and the elderly are at risk from severe infection.

24. Which of the following statement is correct?

1.Amoebiasis also called amoebic dysentery or amoebic colitis is caused by Entamoeba histolytica

2.Infective stage of this parasite is the trophozoite, which penetrates the walls of the host intestine (colon) and secretes histolytic enzymes causing ulceration

3.House flies (Musca domestica) acts as a carrier for transmitting the parasite from contaminated faeces and water

  1. 1, 2
  2. 1, 3
  3. 2, 3
  4. All the above

Explanation

Amoebiasis also called amoebic dysentery or amoebic colitis is caused by Entamoeba histolytica, which lives in the human large intestine and feeds on mucus and bacteria. Infective stage of this parasite is the trophozoite, which penetrates the walls of the host intestine (colon) and secretes histolytic enzymes causing ulceration, bleeding, abdominal pain and stools with excess mucus. Symptoms of amoebiasis can range from diarrhoea to dysentery with blood and mucus in the stool. House flies (Musca domestica) acts as a carrier for transmitting the parasite from contaminated faeces and water.

25. _____ species of Trypanosoma cause sleeping sickness in man

  1. 2
  2. 5
  3. 4
  4. 3

Explanation

African sleeping sickness is caused by Trypanosoma species. Trypanosoma is generally transmitted by the blood sucking Tsetse flies. Three species of Trypanosoma cause sleeping sickness in man.

26. Match the following

A.T. gambiense 1. Triatoma megista

B.T. rhodesiense 2. Glossina palpalis

C.T. cruzi 3. Glossina morsitans

  1. 2, 1, 3
  2. 3, 1, 2
  3. 3, 2, 1
  4. 1, 3, 2

Explanation

T. gambiense is transmitted by Glossina palpalis (Tsetse fly) and causes Gambian or Central African sleeping sickness

T. rhodesiense is transmitted by Glossina morsitans causing Rhodesian or East African sleeping sickness.

T. cruzi is transmitted by a bug called Triatoma megista and causes Chagas disease or American trypanosomiasis.

27. Which of the following statement is correct?

1.Kala – azar or visceral leishmaniasis is caused by Leishmania Donovan

2.Infection may occur in the endothelial cells, bone marrow, liver, lymph glands and blood vessels of the spleen

3.Symptoms of Kala azar are weight loss, anaemia, fever, enlargement of spleen and liver

  1. 1, 2
  2. 1, 3
  3. 2, 3
  4. All the above

Explanation

Kala – azar or visceral leishmaniasis is caused by Leishmania donovani, which is transmitted by the vector Phlebotomus (sand fly). Infection may occur in the endothelial cells, bone marrow, liver, lymph glands and blood vessels of the spleen. Symptoms of Kala azar are weight loss, anaemia, fever, enlargement of spleen and liver.

28. Which of the following statement is correct?

1.Malaria is caused by different types of Plasmodium species

2.Plasmodium lives in the RBC of human in its mature condition it is called as trophozoite

3.It is transmitted from one person to another by the bite of the infected female Anopheles mosquito

  1. 1, 2
  2. 1, 3
  3. 2, 3
  4. All the above

Explanation

Malaria is caused by different types of Plasmodium species such as P. vivax, P. ovale, P. malariae and P. falciparum. Plasmodium lives in the RBC of human in its mature condition it is called as trophozoite. It is transmitted from one person to another by the bite of the infected female Anopheles mosquito.

29. How many phases are involved in life cycle of Plasmodium?

  1. 4
  2. 5
  3. 3
  4. 2

Explanation

Plasmodium vivax is a digenic parasite, involving two hosts, man as the secondary host and female Anopheles mosquito as the primary host. The life cycle of Plasmodium involves three phases namely schizogony, gamogony and sporogony.

30. Which of the following statement is correct?

1.The parasite first enters the human blood stream through the bite of an infected female Anopheles mosquito

2.As it feeds, the mosquito injects the saliva containing the sporozoites

3.Further in the liver they undergo multiple asexual fission (schizogony) and produce merozoites

  1. 1, 2
  2. 1, 3
  3. 2, 3
  4. All the above

Explanation

The parasite first enters the human blood stream through the bite of an infected female Anopheles mosquito. As it feeds, the mosquito injects the saliva containing the sporozoites. The sporozoite within the blood stream immediately enters the hepatic cells of the liver. Further in the liver they undergo multiple asexual fission (schizogony) and produce merozoites. After being released from liver cells, the merozoites penetrate the RBC’s

31. Incubation period of malaria is about____ days

  1. 12
  2. 23
  3. 33
  4. 2

Explanation

The pathological changes caused by malaria, affects not only the erythrocytes but also the spleen and other visceral organs. Incubation period of malaria is about 12 days.

32. Which of the following are the early symptoms of malaria?

1.Headache

2.Nausea

3.Muscular pain

  1. 1, 2
  2. 1, 3
  3. 2, 3
  4. All the above

Explanation

The early symptoms of malaria are headache, nausea and muscular pain. The classic symptoms first develop with the synchronized release of merozoites, haemozoin toxin and erythrocyte debris into the blood stream resulting in malarial paroxysms – shivering chills, high fever followed by sweating. Fever and chills are caused partly by malarial toxins that induce macrophages to release tumour necrosis factor (TNF-α) and interleukin.

33. Match the malaria causative agent with the duration of erythrocytic cycle:

A.P. vivax 1. 36 – 48 hours

B.P. malariae 2. 48 hours

C.P. falciparum 3. 72 hours

  1. 3, 1, 2
  2. 2, 1, 3
  3. 2, 3, 1
  4. 1, 3, 2

Explanation

34. Which of the following statement is correct?

1.It is possible to break the transmission cycle by killing the insect vector

2.Larvae hatch and develop in water but breathe air by moving to the surface

3.Oil can be sprayed over the water surface, to make it impossible for mosquito larvae and pupae to breathe

  1. 1, 2
  2. 1, 3
  3. 2, 3
  4. All the above

Explanation

It is possible to break the transmission cycle by killing the insect vector. Mosquitoes lay their eggs in water. Larvae hatch and develop in water but breathe air by moving to the surface. Oil can be sprayed over the water surface, to make it impossible for mosquito larvae and pupae to breathe.

35. Which of the following statement is correct?

1.Ponds, drainage ditches and other permanent bodies of water can be stocked with fishes such as Gambusia which feed on mosquito larvae

2.Preparations containing Bacillus thuringiensis can be sprayed to kill the mosquito larvae since it is not toxic to other forms of life

  1. 1 alone
  2. 2 alone
  3. 1, 2

d) None

Explanation

Ponds, drainage ditches and other permanent bodies of water can be stocked with fishes such as Gambusia which feed on mosquito larvae. Preparations containing Bacillus thuringiensis can be sprayed to kill the mosquito larvae since it is not toxic to other forms of life. The best protection against malaria is to avoid being bitten by mosquito. People are advised to use mosquito nets, wire gauging of windows and doors to prevent mosquito bites.

36. When did WHO introduced the Malaria eradication programme?

  1. 1995
  2. 1950
  3. 2005
  4. 1985

Explanation

In the 1950’s the World Health Organisation (WHO) introduced the Malaria eradication programme. This programme was not successful due to the resistance of Plasmodium to the drugs used to treat it and resistance of mosquitoes to DDT and other insecticides.

37. Which of the following statement is correct?

1.Malaria vaccine is used to prevent malaria.

2.It requires four injections and has relatively high efficacy

3.WHO does not recommend the use of RTS,S vaccine in babies between 6 and 12 weeks of age

  1. 1, 2
  2. 1, 3
  3. 2, 3
  4. All the above

Explanation

Malaria vaccine is used to prevent malaria. The only approved vaccine as of 2015 is RTS,S (Mosquirix). It requires four injections and has relatively low efficacy (26–50%). Due to this low efficacy, WHO does not recommend the use of RTS,S vaccine in babies between 6 and 12 weeks of age.

38. ______ was recognized as a causative agent of human diseases much earlier than bacteria

  1. Algae
  2. Fungi
  3. Virus
  4. Protozoan

Explanation

Fungi was recognized as a causative agent of human diseases much earlier than bacteria. Dermatomycosis is a cutaneous infection caused by fungi belonging to the genera Trichophyton, Microsporum and Epidermophyton.

39. Ringworm is one of the most common_____ disease in humans

  1. Bacterial
  2. Viral
  3. Fungal
  4. Protozoan

Explanation

Ringworm is one of the most common fungal disease in humans. Appearance of dry, scaly lesions on the skin, nails and scalp are the main symptoms of the disease. Heat and moisture help these fungi to grow and makes them to thrive in skin folds such as those in the groin or between the toes.

40. Ringworms of the feet is known as Athlete’s foot caused by_____

  1. Mucormycosis
  2. Tinea pedis
  3. P. vivax
  4. P. volatile

Explanation

Ringworms of the feet is known as Athlete’s foot caused by Tinea pedis. Ringworms are generally acquired from soil or by using clothes, towels and comb used by infected persons.

41. Which of the following statement is correct?

1.Helminthes are mostly endoparasitic in the gut and blood of human beings and cause diseases called helminthiasis

2.The two most prevalent helminthic diseases are Ascariasis and Filariasis.

  1. 1 alone
  2. 2 alone
  3. 1, 2
  4. None

Explanation

Helminthes are mostly endo-parasitic in the gut and blood of human beings and cause diseases called helminthiasis. The two most prevalent helminthic diseases are Ascariasis and Filariasis.

42. Which of the following statement is incorrect?

1.Ascaris is a monogenic parasite and exhibits sexual dimorphism.

2.Ascariasis is a disease caused by the intestinal endoparasite Ascaris lumbricoides commonly called the round worms

  1. 1 alone
  2. 2 alone
  3. 1, 2
  4. None

Explanation

Ascaris is a monogenic parasite and exhibits sexual dimorphism. Ascariasis is a disease caused by the intestinal endoparasite Ascaris lumbricoides commonly called the round worms. It is transmitted through ingestion of embryonated eggs through contaminated food and water

43. Helminthes are mostly endo-parasitic in_____

  1. Liver
  2. Gut
  3. Lung
  4. Intestine

Explanation

Helminthes are mostly endo-parasitic in the gut and blood of human beings and causes diseases called helminthiasis. The two most prevalent helminthic diseases are Ascariasis and Filariasis.

44. Which of the following statement is correct?

1.Ascaris is a monogenic parasite and exhibits sexual unique

2.Ascariasis is a disease caused by the intestinal endoparasite Ascaris lumbricoides commonly called the round worms

  1. 1 alone
  2. 2 alone
  3. 1, 2
  4. None

Explanation

Ascaris is a monogenic parasite and exhibits sexual dimorphism. Ascariasis is a disease caused by the intestinal endoparasite Ascaris lumbricoides commonly called the round worms. It is transmitted through ingestion of embryonated eggs through contaminated food and water. Children playing in contaminated soils are also prone to have a chance of transfer of eggs from hand to mouth.

45. Filariasis is caused by_____

  1. Plasmodium Vivax
  2. Wuchereria bancrofti
  3. Ascaris lumbricoides
  4. Mucormycosis

Explanation

Filariasis is caused by Wuchereria bancrofti, commonly called filarial worm. It is found in the lymph vessels and lymph nodes of man.

46. Which of the following statement is correct?

1.Hygiene is a set of practices performed to conserve good health

2.According to the World Health Organization (WHO), hygiene refers to “conditions and practices that help to maintain health and prevent the spread of diseases.”

  1. 1 alone
  2. 2 alone
  3. 1, 2
  4. None

Explanation

Hygiene is a set of practices performed to conserve good health. According to the World Health Organization (WHO), hygiene refers to “conditions and practices that help to maintain health and prevent the spread of diseases.” Personal hygiene refers to maintaining one’s body clean by bathing, washing hands, trimming fingernails, wearing clean clothes and also includes attention to keeping surfaces in the home and workplace, including toilets, bathroom facilities, clean and pathogen-free.

47. Which of the following diseases are transmitted through contaminated food and water?

  1. Typhoid
  2. Amoebiasis
  3. Ascariasis
  4. All the above

Explanation

Our public places teem with infection, contamination and germs. It seems that every surface we touch and the air we breathe are with pollutants and microbes. It’s not just the public places that are unclean, but we might be amazed at the number of people who do not wash their hands before taking food, after visiting the restroom, or who sneeze without covering their faces. Many infectious diseases such as typhoid, amoebiasis and ascariasis are transmitted through contaminated food and water.

48. Which of the following diseases can be controlled by vaccination?

  1. Polio
  2. Diphtheria
  3. Tetanus
  4. All the above

Explanation

Advancement in science and technology provide effective controlling measures for many infectious and non-infectious diseases. The use of vaccines and adopted immunization programmes have helped to eradicate small pox in India. Moreover a large number of infectious diseases like polio, diphtheria, pneumonia and tetanus have been controlled by the use of vaccines and by creating awareness among the people.

49. Which of the following statement is incorrect?

1.Immunology is the study of immune system.

2.It refers to all the mechanisms used by the body for protection from environmental agents that are foreign to the body

  1. 1 alone
  2. 2 alone
  3. 1, 2
  4. None

Explanation

Immunology is the study of immune system. This system protects an individual from various infective agents. It refers to all the mechanisms used by the body for protection from environmental agents that are foreign to the body.

50. Which of the following statement is correct?

1.When the immune system does not function efficiently in an individual, it leads to infection causing disease

2.The overall ability of body to fight against the disease causing pathogen is called immunity.

  1. 1 alone
  2. 2 alone
  3. 1, 2
  4. None

Explanation

When the immune system does not function efficiently in an individual, it leads to infection causing disease. The overall ability of body to fight against the disease causing pathogen is called immunity. It is also called disease resistance and the lack of immunity is known as susceptibility. Immunity is highly specific.

51. Which of the following can induce immune response?

  1. Pollen grain
  2. RBC
  3. Nucleic acid

1, 2

1, 3

2, 3

All the above

Explanation

Almost all the macromolecules e.g. proteins, polysaccharides, nucleic acids, etc., as long as they are foreign to recipient organism can induce immune response. Any substance capable of eliciting immune response is called an ANTIGEN

52. Which of the following statement is correct?

1.Innate immunity is the artificial phenomenon of resistance to infection

2.The innate defense mechanisms are non-specific in the sense that they are effective against a wide range of potentially infectious agents

  1. 1 alone
  2. 2 alone
  3. 1, 2
  4. None

Explanation

Innate immunity is the natural phenomenon of resistance to infection which an individual possesses right from the birth. The innate defense mechanisms are non-specific in the sense that they are effective against a wide range of potentially infectious agents. It is otherwise known as non-specific immunity or natural immunity.

53. Which of the following are anatomical barrier?

1.Skin

2.Bones

3.Mucus membrane

  1. 1, 2
  2. 1, 3
  3. 2, 3
  4. All the above

Explanation

54. Which acid is secreted by gastric system?

  1. H2SO4
  2. HCl
  3. HNO3
  4. Ethanoic acid

Explanation

55. Which of the following are phagocytes?

1.Monocytes

2.Neutrophils

3.Esonophils

4.Tissue macrophages

  1. 1, 2, 4
  2. 2, 3, 4
  3. 1, 3, 4
  4. All the above

Explanation

56. Which of the following are the chemotic signals of tissue damage?

  1. Serotonin
  2. Histamine
  3. Prostaglandins
  4. All the above

Explanation

57. Which of the following are the unique features of acquired immunity?

1.Antigenic specificity

2.Diversity

3.Immunological memory

  1. 1, 2
  2. 1, 3
  3. 2, 3
  4. All the above

Explanation

The immunity that an individual acquires after birth is known as acquired immunity. It is the body’s resistance to a specific pathogen. The unique features of acquired immunity are antigenic specificity, diversity, recognition of self and non-self and immunological memory.

58. Which of the following statement is incorrect?

1.When pathogens are destroyed by cells without producing antibodies, then it is known Cell mediated immunity

2.This is brought about by T cells, macrophages and natural killer cells

  1. 1 alone
  2. 2 alone
  3. 1, 2
  4. None

Explanation

Acquired immunity has two components – cell mediated immunity (CMI) and antibody mediated immunity or humoral immunity. When pathogens are destroyed by cells without producing antibodies, then it is known as cell mediated immune response or cell mediated immunity. This is brought about by T cells, macrophages and natural killer cells.

59. Antibody mediated immunity is brought about by____

  1. B-cells
  2. T-cells
  3. Antigen presenting cells
  4. All the above

Explanation

When pathogens are destroyed by the production of antibodies, then it is known as antibody mediated or humoral immunity. This is brought about by B cells with the help of antigen presenting cells and T helper cells. Antibody production is the characteristic feature of vertebrates only.

60. Which of the following statement about passive immunity is correct?

1.It retains memory

2.Passive immunity is received passively and there is no active host participation

3.It is transient and less effective

1, 2

1, 3

2, 3

All the above

Explanation

61. Which of the following statement about active immunity is correct?

1.Active immunity is produced actively by host’s immune system

2.Immunological memory is absent

3.Immunity is effective only after a short period.

  1. 1, 2
  2. 1, 3
  3. 2, 3
  4. All the above

Explanation

62. The process of production of blood cells in the bone marrow is called____

  1. Haematosis
  2. Haematopoiesis
  3. Haematoporosis
  4. None

Explanation

The immune responses may be primary or secondary. The process of production of blood cells in the bone marrow is called haematopoiesis.

63. Which of the following statement about Primary Immune Response is correct?

1.It occurs as a result of primary contact with an antigen

2.Antibody level reaches peak in 7 to 10 weeks

3.There is rapid decline in antibody level

  1. 1, 2
  2. 1, 3
  3. 2, 3
  4. All the above

Explanation

64. Which of the following statement is correct about secondary immune response?

1.It occurs as a result of second and subsequent contacts with the same antigen.

2.Antibody level reaches peak in 3 to 5 days

3.It appears mainly in the bone marrow, followed by the spleen and lymph nodes.

  1. 1, 2
  2. 1, 3
  3. 2, 3
  4. All the above

Explanation

65. Which of the following statement is correct?

1.Immune system of an organism consists of several structurally and functionally different organs and tissues that are widely dispersed in the body.

2.Based on their functions, they are classified into primary or central lymphoid organs and secondary or peripheral lymphoid organs.

3.The secondary lymphoid organs trap antigens and make it available for mature lymphocytes, which can effectively fight against these antigens

  1. 1, 2
  2. 1, 3
  3. 2, 3
  4. All the above

Explanation

Immune system of an organism consists of several structurally and functionally different organs and tissues that are widely dispersed in the body. The organs involved in the origin, maturation and proliferation of lymphocytes are called lymphoid organs. Based on their functions, they are classified into primary or central lymphoid organs and secondary or peripheral lymphoid organs. The primary lymphoid organs provide appropriate environment for lymphocytic maturation. The secondary lymphoid organs trap antigens and make it available for mature lymphocytes, which can effectively fight against these antigens.

66. Which of the following constitutes Primary lymphoid organs?

1.Bursa of Fabricius

2.Bone marrow

3.Thymus gland

  1. 1, 2
  2. 1, 3
  3. 2, 3
  4. All the above

Explanation

Bursa of Fabricius of birds, bone marrow and thymus gland of mammals constitute the primary lymphoid organs involved in the production and early selection of lymphocytes. These lymphocytes become dedicated to a particular antigenic specificity.

67. In mammals, B cell maturation occurs in____

  1. Bone marrow
  2. Thymus
  3. Bursa of fabricius
  4. All the above

Explanation

Only when the lymphocytes mature in the primary lymphoidal organs, they become immunocompetent cells. In mammals, B cell maturation occurs in the bone marrow and T cells maturation occurs in the thymus.

68. Thymus is located_____

  1. Above heart
  2. Below heart
  3. Above lungs
  4. Parallel to spleen

Explanation

The thymus is a flat and bilobed organ located behind the sternun, above the heart. Each lobe of the thymus contains numerous lobules, separated from each other by connective tissue called septa.

69. Which of the following statement about thymus is correct?

1.One of its main secretions is the hormone thymosin

2.It stimulates the B cell to become mature and immunocompetent

3.Thymus is most active during the neonatal and pre-adolescent periods

  1. 1, 2
  2. 1, 3
  3. 2, 3
  4. All the above

Explanation

One of thymus main secretions is the hormone thymosin. It stimulates the T cell to become mature and immunocompetent. By the early teens, the thymus begins to atrophy and is replaced by adipose tissue. Thus, thymus is most active during the neonatal and pre-adolescent periods.

70. Which of the following statement about bone marrow is correct?

1.Bone marrow is a lymphoid tissue found within the spongy portion of the bone

2.Bone marrow contains stem cells known as haematopoietic cells.

3.These cells have the potential to multiply through cell division

  1. 1, 2
  2. 1, 3
  3. 2, 3
  4. All the above

Explanation

Bone marrow is a lymphoid tissue found within the spongy portion of the bone. Bone marrow contains stem cells known as haematopoietic cells. These cells have the potential to multiply through cell division and either remain as stem cells or differentiate and mature into different kinds of blood cells.

71. Which of the following statement about Peyer’s patches is incorrect?

1.Peyer’s patches are oval-shaped areas of thickened tissue that are embedded in the mucus-secreting lining of the small intestine of humans and other vertebrate animals

2.Peyer’s patches contain a variety of immune cells, including macrophages, dendritic cells, T cells, and B cells

  1. 1 alone
  2. 2 alone
  3. 1, 2
  4. None

Explanation
Peyer’s patches are oval-shaped areas of thickened tissue that are embedded in the mucus-secreting lining of the small intestine of humans and other vertebrate animals. Peyer’s patches contain a variety of immune cells, including macrophages, dendritic cells, T cells, and B cells.

72. The tonsils are present at____ region

  1. Lumbar
  2. Pharynx
  3. Larynx
  4. All the above

Explanation

The tonsils (palatine tonsils) are a pair of soft tissue masses located at the back of the throat (pharynx). The tonsils are part of the lymphatic system, which help to fight infections. They stop invading germs including bacteria and viruses.

73. Which of the following statement is correct?

1.The adenoids are glands located in the roof of the mouth

2.Adenoids shrink during adulthood and may disappear by adolescence.

  1. 1 alone
  2. 2 alone
  3. 1, 2
  4. None

Explanation

The adenoids are glands located in the roof of the mouth, behind the soft palate where the nose connects to the throat. The adenoids produce antibodies that help to fight infections. Typically, the adenoids shrink during adolescence and may disappear by adulthood.

74. Which of the following is the first one to encounter the antigen that enters the tissue spaces?

  1. Spleen
  2. Lymph node
  3. Nissels granules
  4. Stomach acid

Explanation

Lymph node is a small bean-shaped structure and is part of the body’s immune system. It is the first one to encounter the antigen that enters the tissue spaces. Lymph nodes filter and trap substances that travel through the lymphatic fluid.

75. Which of the following are the characteristics of lymph fluid?

  1. Transparent
  2. Immobile
  3. Colourless
  4. Extracellular
  5. 1, 2, 4
  6. 2, 3, 4
  7. 1, 3, 4
  8. All the above

Explanation

Lymph is a clear, transparent, colourless, mobile and extracellular fluid connective tissue. As the lymph percolates through the lymph node, the particulate antigen brought in by the lymph will be trapped by the phagocytic cells, follicular and interdigitating dendritic cells

76. Which of the following statement is correct?

  1. Lymph node has four zones
  2. They are the cortex, paracortex and medulla.
  3. The outer most layer of the lymph node is called cortex, which consists of B-lymphocytes, macrophages, and follicular dendritic cells.
  4. 1, 2
  5. 1, 3
  6. 2, 3
  7. All the above

Explanation

Lymph node has three zones. They are the cortex, paracortex and medulla. The outer most layer of the lymph node is called cortex, which consists of B-lymphocytes, macrophages, and follicular dendritic cells. The paracortex zone is beneath the cortex, which is richly populated by T lymphocytes and interdigitating dendritic cells. The inner most zone is called the medulla which is sparsely populated by lymphocytes, but many of them are plasma cells, which actively secrete antibody molecules.

77. Visible swelling of lymph node occurs during____

  1. Passive immune response
  2. Active immune response
  3. Both a and b
  4. None

Explanation

The lymph leaving a node carries enriched antibodies secreted by the medullary plasma cells against the antigens that enter the lymph node. Sometimes visible swelling of lymph nodes occurs due to active immune response and increased concentration of lymphocytes.

78. Which of the following statement is correct?

  1. The mucosa-associated lymphoid tissue (MALT) is a diffuse system of small concentrations of lymphoid tissue in the alimentary, respiratory and urino-genital tracts
  2. MALT is populated by lymphocytes such as T and B cells
  3. It also possesses IgM antibodies.
  4. 1, 2
  5. 1, 3
  6. 2, 3
  7. All the above

Explanation

The mucosa-associated lymphoid tissue (MALT) is a diffuse system of small concentrations of lymphoid tissue in the alimentary, respiratory and urino-genital tracts. MALT is populated by lymphocytes such as T and B cells, as well as plasma cells and macrophages, each of which is well situated to encounter antigens passing through the mucosal epithelium. It also possesses IgA antibodies.

79. Which of the following statement is correct?

  1. Gut-associated lymphoid tissue (GALT) is a component of the mucosaassociated lymphoid tissue (MALT) which works in the immune system to protect the body from invasion in the gut.
  2. Bronchus Associated Lymphoid Tissues (BALT) also a component of MALT is made of lymphoid tissue (tonsils, lymph nodes, lymph follicles) is found in the respiratory mucosae from the nasal cavities to the lungs
  3. 1 alone
  4. 2 alone
  5. 1, 2
  6. None

Explanation

Gut-associated lymphoid tissue (GALT) is a component of the mucosa-associated lymphoid tissue (MALT) which works in the immune system to protect the body from invasion in the gut. Bronchus Associated Lymphoid Tissues (BALT) also a component of MALT is made of lymphoid tissue (tonsils, lymph nodes, lymph follicles) is found in the respiratory mucosae from the nasal cavities to the lungs.

80. Which of the following can be produced by stem cells?

  1. RBC
  2. Platelets
  3. WBC
  4. 1, 2
  5. 1, 3
  6. 2, 3
  7. All the above

Explanation

All these cells are derived from pluripotent haematopoetic stem cells. Each stem cell has the capacity to produce RBC, WBC and platelets. The only cells capable of specifically recognising and producing an immune response are the lymphocytes. The other types of white blood cells play an important role in non-specific immune response, antigen presentation and cytokine production.

81. Match the following

A.Red blood cells 1. 2000-7000

B.Lymphocytes 2. 4200,000 – 6500,000

C.Neutrophils 3. 1500 – 4000

D.Basophils 4. 50- 100

  1. 1, 2, 4, 3
  2. 2, 3, 1, 4
  3. 3, 2, 1, 4
  4. 4, 1, 2, 3

Explanation

82. Which of the following statement is correct?

  1. About 40-60% of the white blood cells are lymphocytes.
  2. The two main types of lymphocytes are B and T lymphocytes
  3. Some remain in the blood, while others accumulate in the lymph nodes and spleen.
  4. 1, 2
  5. 1, 3
  6. 2, 3
  7. All the above

Explanation

About 20-30% of the white blood cells are lymphocytes. They have a large nucleus filling most of the cell, surrounded by a little cytoplasm. The two main types of lymphocytes are B and T lymphocytes. Both these are produced in the bone marrow. B lymphocytes (B cells) stay in the bone marrow until they are mature. Then they circulate around the body. Some remain in the blood, while others accumulate in the lymph nodes and spleen. T lymphocytes leave the bone marrow and mature in the thymus gland.

83. Lymphocytes have receptor____ on their surface

  1. Vitamins
  2. Proteins
  3. Carbohydrates
  4. None

Explanation

T lymphocytes leave the bone marrow and mature in the thymus gland. Once mature, T cells also accumulate in the same areas of the body as B cells. Lymphocytes have receptor proteins on their surface. When receptors on a B cell bind with an antigen, the B cell becomes activated and divides rapidly to produce plasma cells. The plasma cells produce antibodies. Some B cells do not produce antibodies but become memory cells. These cells are responsible for secondary immune response. T lymphocytes do not produce antibodies.

84. _____types of dendritic cells are known

  1. 3
  2. 5
  3. 4
  4. 2

Explanation

Dendritic cells are called so because its covered with long, thin membrane extensions that resemble dendrites of nerve cells. These cells present the antigen to T-helper cells. Four types of dendritic cells are known. They are langerhans, interstitial cells, myeloid and lymphoid cells.

85. Which of the following statement is incorrect?

  1. The term antigen (Ag) is used in two senses, the first to describe a molecule which generates an immune response and the second, a molecule which reacts with antibodies
  2. Antigen is a substance that is specific to an antibody or a T-cell receptor and is often used as a synonym for immunogen
  3. 1 alone
  4. 2 alone
  5. 1, 2
  6. None

Explanation

The term antigen (Ag) is used in two senses, the first to describe a molecule which generates an immune response and the second, a molecule which reacts with antibodies. In general antigens are large, complex molecular substances that can induce a detectable immune response. Thus an antigen is a substance that is specific to an antibody or a T-cell receptor and is often used as a synonym for immunogen.

86.Which of the following statement is correct?

  1. An immunogen is a substance capable of initiating an immune response
  2. Substances that can enhance the immune response to an antigen are called adjuvants. Epitope is an antigenic determinant and is the active part of an antigen
  3. A paratope is the antigen – binding site and is a part of an antibody which recognizes and binds to an antigen
  4. 1, 2
  5. 1, 3
  6. 2, 3
  7. All the above

Explanation

An immunogen is a substance capable of initiating an immune response. Haptens are substance that are non-immunogenic but can react with the products of a specific immune response. Substances that can enhance the immune response to an antigen are called adjuvants. Epitope is an antigenic determinant and is the active part of an antigen. A paratope is the antigen – binding site and is a part of an antibody which recognizes and binds to an antigen.

87. How many types of antibodies are there?

  1. 6
  2. 4
  3. 3
  4. 5

Explanation

The antibodies are classified into five major categories, based on their physiological and biochemical properties. They are IgG (gamma), IgM (mu), IgA (alpha), IgD (delta) and IgE (epsilon).

88. An antibody molecule is___ shaped structure

  1. Z
  2. Y
  3. C
  4. X

Explanation

In the 1950s, experiments by Porter and Edelman revealed the basic structure of the immunoglobulin. An antibody molecule is Y shaped structure that comprises of four polypeptide chains, two identical light chains (L) of molecular weight 25,000 Da (approximately 214 amino acids) and two identical heavy chains (H) of molecular weight 50,000 Da (approximately 450 amino acids).

89. The polypeptide chains are linked together by_____

  1. C – C
  2. P – P
  3. S – S
  4. K – K

Explanation

The polypeptide chains are linked together by di-sulphide (S-S) bonds. One light chain is attached to each heavy chain and two heavy chains are attached to each other to form a Y shaped structure. Hence, an antibody is represented by H2 L2. The heavy chains have a flexible hinge region at their approximate middles.

90. Which of the following statement is correct?

  1. The reaction between an antigen and antibody is the basis for humoral immunity or antibody mediated immunity
  2. The reaction between antigen and antibody occurs in four stages.
  3. 1 alone
  4. 2 alone
  5. 1, 2
  6. None

Explanation

The reaction between an antigen and antibody is the basis for humoral immunity or antibody mediated immunity. The reaction between antigen and antibody occurs in three stages. During the first stage, the reaction involves the formation of antigen – antibody complex. The next stage leads to visible events like precipitation, agglutination, etc., The final stage includes destruction of antigen or its neutralization.

91. The binding force between antigen and antibody is due to____ factors

  1. 5
  2. 4
  3. 3
  4. 2

Explanation

The binding force between antigen and antibody is due to three factors. They are closeness between antigen and antibody, noncovalent bonds or intermolecular forces and affinity of antibody.

92. Which of the following statement is correct?

  1. When antigen and antibody are closely fitted, the strength of binding is great.
  2. The bonds that hold the antigen to the antibody combining site are all covalent in nature
  3. These include hydrogen bonds, electrostatic bonds, Van der Waals forces and hydrophobic bonds
  4. 1, 2
  5. 1, 3
  6. 2, 3
  7. All the above

Explanation

When antigen and antibody are closely fitted, the strength of binding is great. When they are apart binding strength is low. The bonds that hold the antigen to the antibody combining site are all non-covalent in nature. These include hydrogen bonds, electrostatic bonds, Van der Waals forces and hydrophobic bonds. Antibody affinity is the strength of the reaction between a single antigenic determinant and a single combining site on the antibody.

93. ______ is the process by which a pathogen is marked of ingestion and destruction by a phagocyte.

  1. Opsonisation
  2. Agglutination
  3. Phagocytosis
  4. All the above

Explanation

Opsonisation or enhanced attachment is the process by which a pathogen is marked of ingestion and destruction by a phagocyte. Opsonisation involves the binding of an opsonin i.e., antibody, to a receptor on the pathogen’s cell membrane.

94. Which of the following statement is incorrect?

  1. A vaccine is a biological preparation that provides active acquired immunity to a particular disease and resembles a disease-causing microorganism and is often made from weakened or attenuated or killed forms of the microbes, their toxins, or one of its surface proteins
  2. Vaccines “teach” our body how to defend itself when viruses or bacteria, invade it.
  3. 1 alone
  4. 2 alone
  5. 1, 2
  6. None

Explanation

A vaccine is a biological preparation that provides active acquired immunity to a particular disease and resembles a disease-causing microorganism and is often made from weakened or attenuated or killed forms of the microbes, their toxins, or one of its surface proteins. Vaccines “teach” our body how to defend itself when viruses or bacteria, invade it. Vaccines deliver only very little amounts of inactivated or weakened viruses or bacteria, or parts of them.

95. The vaccines are classified into____ types

  1. 4
  2. 3
  3. 2
  4. 5

Explanation

Vaccine initiates the immunization process. The vaccines are classified as first, second and third generation vaccine

96. Which of the following is not a 1st generation vaccine?

  1. MMR
  2. Hepatitis- B
  3. Varicella
  4. Salk’s Polio

Explanation

First generation vaccine is further subdivided into live attenuated vaccine, killed vaccine and toxoids (Fig. 8.9). Live attenuated vaccines use the weakened (attenuated), aged, less virulent form of the virus. E.g. Measles, mumps and rubella (MMR) vaccine and the Varicella (chickenpox) vaccine, Killed (inactivated) vaccines are killed or inactivated by heat and other methods. E.g. Salk’s polio vaccine. Toxoid vaccines contain a toxin or chemical secreted by the bacteria or virus. They make us immune to the harmful effects of the infection, instead of to the infection itself. E.g. DPT vaccine (Diphtheria, Pertussis and Tetanus).

97. Which of the following statement is correct?

  1. Second generation vaccine contains the pure surface antigen of the pathogen
  2. Hepatitis- B vaccine is a 3rd generation vaccine
  3. Third generation vaccine contains the purest and the highest potency vaccines which are synthetic in generation.
  4. 1, 2
  5. 1, 3
  6. 2, 3
  7. All the above

Explanation

Second generation vaccine contains the pure surface antigen of the pathogen. E.g. Hepatitis-B vaccine. Third generation vaccine contains the purest and the highest potency vaccines which are synthetic in generation. The latest revolution in vaccine is DNA vaccine or recombinant vaccine.

98. Who prepared first vaccine for small pox?

  1. Stuart Jenner
  2. Edward Jenner
  3. Malies Jenner
  4. Stephen Hawkins

Explanation

Vaccino therapy is the method of use of vaccine for treatment of disease. Dr. Edward Jenner prepared first vaccine for small pox in 1796.

99. Polio vaccine was developed by____ (vaccine consists of inactivated microorganism)

  1. Dr. Edward Jenner
  2. Dr. Jonas Salk
  3. Dr. Albert Sabin
  4. Louis Pasteur

Explanation

Polio vaccine was developed by Dr. Jonas Salk (vaccine consists of inactivated microorganism) and Dr. Albert Sabin (live attenuated oral polio vaccine). Louis Pasteur (1885) discovered vaccine against rabies, anthrax and cholera. BCG vaccine was developed by Calmette and Guerin against tuberculosis in France in the year 1908.

100. Which of the following statement is correct?

  1. Vaccination is the process of administrating a vaccine into the body or the act of introducing a vaccine into the body to produce immunity to a specific disease
  2. Immunization is the process of the body building up immunity to a particular disease.
  3. 1 alone
  4. 2 alone
  5. 1, 2
  6. None

Explanation

“Vaccination is the process of administrating a vaccine into the body or the act of introducing a vaccine into the body to produce immunity to a specific disease.” Immunization is the process of the body building up immunity to a particular disease. Immunization describes the actual changes in the body after receiving a vaccine. Vaccines work by fighting the pathogen and then recording it in their memory system.

101. Allergy is a form of over active immune response mediated by___

  1. IgG
  2. IgM
  3. IgA
  4. IgE

Explanation

Allergy is a form of over active immune response mediated by IgE and mast cells. It can also be due to the release of chemicals like histamine and serotonin from the mast cells.

102. ____ is the classical immediate hypersensitivity reaction

  1. Anaphylaxis
  2. Inaphylaxis
  3. Aniophylaxis
  4. None

Explanation

Anaphylaxis is the classical immediate hypersensitivity reaction. It is a sudden, systematic, severe and immediate hypersensitivity reaction occurring as a result of rapid generalized mast-cell degranulation.

103. Which of the following statement is correct?

  1. Immunodeficiency results from the failure of one or more components of the immune system.
  2. Primary immune deficiencies are caused by genetic developmental defects.
  3. AIDS is an acronym for Acquired Immuno- Deficiency Syndrome
  4. 1, 2
  5. 1, 3
  6. 2, 3
  7. All the above

Explanation

Immunodeficiency results from the failure of one or more components of the immune system. Primary immune deficiencies are caused by genetic developmental defects. Secondary immune deficiencies arise due to various reasons like radiation, use of cytolytic and immunosuppressive drugs and infections. AIDS is an acronym for Acquired Immuno- Deficiency Syndrome. It is the deficiency of immune system, acquired during the life time of an individual indicating that it is not a congenital disease.

104. Which of the following statement is correct?

  1. AIDS is caused by Human Immuno- Deficiency Virus
  2. It selectively infects helper B cells
  3. HIV is classified into the types 1 and 2 (HIV-1, HIV-2)
  4. 1, 2
  5. 1, 3
  6. 2, 3
  7. All the above

Explanation

AIDS is an acronym for Acquired Immuno- Deficiency Syndrome. It is the deficiency of immune system, acquired during the life time of an individual indicating that it is not a congenital disease. AIDS is caused by Human Immuno- Deficiency Virus (HIV). It selectively infects helper T cells. The infected helper T cells will not stimulate antibody production by B-cells resulting in loss of natural defence against viral infection. On the basis of genetic characteristics and differences in the viral antigens, HIV is classified into the types 1 and 2 (HIV-1, HIV-2).

105. Which of the following statement is correct?

  1. The human immunodeficiency virus belongs to the genus Varicella virus
  2. HIV is seen as a spherical virus, 100-120 nm in diameter, containing a dense core surrounded by a lipoprotein envelope
  3. The core is covered by a capsid made of proteins.
  4. 1, 2
  5. 1, 3
  6. 2, 3
  7. All the above

Explanation

The human immunodeficiency virus belongs to the genus Lentivirus. When observed under the electron microscope, HIV is seen as a spherical virus, 100-120 nm in diameter, containing a dense core surrounded by a lipoprotein envelope. The envelope has glycoprotein (gp) spikes termed gp 41 and gp 120. At the core, there are two large single stranded RNA. Attached to the RNA are molecules of reverse transcriptase. It also contains enzymes like protease and ribonuclease. The core is covered by a capsid made of proteins. This is followed by another layer of matrix proteins.

106. HIV can survive for____ days inside a cell

  1. 3
  2. 1
  3. 1.5
  4. 2

Explanation

The HIV is often located within the cells especially in macrophages. HIV can survive for 1.5 days inside a cell but only about 6 hours outside a cell. Routes of HIV transmission include unsafe sexual contact, blood-contaminated needles, organ transplants, blood transfusion and vertical transmission from HIV infected mother to child. HIV is not transmitted by insects or by casual contact.

107. Which of the following statement is correct?

  1. After getting into the body of the person, the virus enters into macrophages where RNA genome
  2. There where the virus replicates to form viral DNA with the help of the enzyme reverse transcriptase.
  3. 1 alone
  4. 2 alone
  5. 1, 2
  6. None

Explanation

After getting into the body of the person, the virus enters into macrophages where RNA genome of the virus replicates to form viral DNA with the help of the enzyme reverse transcriptase. This viral DNA gets incorporated into the DNA of host cells and directs the infected cells to produce viral particles.

108. Which of the following statement is correct?

  1. A simple blood test is available that can determine whether the person has been infected with HIV
  2. Western blot test is more reliable and a confirmatory test
  3. ELISA is preliminary test.
  4. 1, 2
  5. 1, 3
  6. 2, 3
  7. All the above

Explanation

A simple blood test is available that can determine whether the person has been infected with HIV. The ELISA test (Enzyme Linked Immuno-Sorbent Assay) detects the presence of HIV antibodies. It is a preliminary test. Western blot test is more reliable and a confirmatory test. It detects the viral core proteins. If both tests detect the presence of the antibodies, the person is considered to be HIV positive.

109. Which of the following are the features of cancer cells?

  1. Large cytoplasmic volume compared to nuclei
  2. Variation in cell shape and size
  3. Large number of dividing cells
  4. 1, 2
  5. 1, 3
  6. 2, 3
  7. All the above

Explanation

110. Which of the following statement is incorrect?

1.Immunotherapy also called biological therapy uses substances made by the body or in a laboratory (monoclonal antibodies) to improve or to resist the immune system function

2.Different approaches have been attempted in the immunotherapy of cancer

  1. 1 alone
  2. 2 alone
  3. 1, 2
  4. None

Explanation

Immunotherapy also called biological therapy uses substances made by the body or in a laboratory (monoclonal antibodies) to improve or to resist the immune system function. Different approaches have been attempted in the immunotherapy of cancer. Immunotherapy appears to be important in getting rid of the residual malignant cells after the gross tumour has been removed.

111. Match the following

A.Stimulants 1. Morphine

B.Depressants 2. Alcohol

C.Hallucinogens 3. Phencyclidine

D.Analgesics 4. Cocaine

  1. 2, 1, 3, 4
  2. 4, 2, 3, 1
  3. 4, 3, 2, 1
  4. 1, 3, 2, 4

Explanation

112. Which of the following statement is correct?

1.Cocaine is a white powder that is obtained from the leaves of the coca plant, Erythroxylum coca.

2.Cocaine causes serious physical and psychological problems including hallucinations and paranoia.

  1. 1 alone
  2. 2 alone
  3. 1, 2
  4. None

Explanation

Cocaine is a white powder that is obtained from the leaves of the coca plant, Erythroxylum coca. It is commonly called coke or crack. Cocaine causes serious physical and psychological problems including hallucinations and paranoia. The other plants with hallucinogenic properties are Atropa belladonna and Datur

113. Which of the following are the symptoms of depression?

1.Loss of self confidence

2.Anxiety

3.Not being able to enjoy things that are usually pleasurable

  1. 1, 2
  2. 1, 3
  3. 2, 3
  4. All the above

Explanation

Signs and symptoms of mental depression:

Loss of self confidence and self-esteem

Anxiety

Not being able to enjoy things that are usually pleasurable or interesting

114. Alcoholic anonymous was started in___ by a businessman and a doctor

  1. 1991
  2. 1935
  3. 1919
  4. 1945

Explanation

Alcoholic anonymous was started in 1935 by a businessman and a doctor who had been a “hopeless drunk” for many years. After the men helped each other to stop drinking and to stay sober, they then founded the alcoholic anonymous to help other alcoholics. Since that time alcoholic anonymous has spread throughout the world.

115. Which of the following statement is correct?

  1. Alcoholism is the inability to control drinking due to physical and emotional dependence on alcohol.
  2. Treatment involves counselling by a healthcare professional. Detoxification programme in a hospital or medical facility is an option for those who need additional assistance
  3. 1 alone
  4. 2 alone
  5. 1, 2
  6. None

Explanation

Alcoholism is the inability to control drinking due to physical and emotional dependence on alcohol. Treatment involves counselling by a healthcare professional. Detoxification programme in a hospital or medical facility is an option for those who need additional assistance. Medications are available to reduce the desire to drink and smoke.

116. Choose the Incorrect statements.

i) Disease is defined as a disorder or malfunction of the body and physical structure only.

ii) Morphological, Physiological and psychological disturbances are the causes of disease.

iii) Genetic anomalies, pathogens, environmental factors or life style changes are the causes for the various disturbances.

a)i only

b)ii only

c)iii only

d) None of the above

Explanation

Disease can be defined as a disorder ormalfunction of the mind or body. It involvesmorphological, physiological and psychologicaldisturbances which may be due to environmentalfactors or pathogens or genetic anomalies or life style changes.

117. Which of the following disease is not caused by bacterial infection?

a) Plague

b) Typhoid

c) Chicken pox

d) Pneumonia

Explanation

118. Which of the following disease is not transmitted through droplet infection?

a) Pneumonia

b) Cholera

c) Diphtheria

d) Tuberculosis

119. Which of the following disease results in rigidity and spasm of jaw muscles?

a) Tetanus

b) Bubonic plague

c) Shigellosis

d) Typhoid

120. Which of these symptoms does not relate with pneumonia?

a) Painful breathing

b) Brown sputum

c) Dehydration

d) Cough

Explanation

121. Why there is a difficulty in preparing a common vaccine for common cold?

a) Sources of RNA are difficult to identify.

b) RNA genomes changing continuously due to mutation.

c) Symptoms of disease differ from person to person.

d) Common dosage could not be used for various age groups.

Explanation

Common cold is caused by more than150 different strains of Rhino viruses.More over their RNA genome keepschanging due to mutation. Hence it is verydifficult to prepare a common vaccine for the disease.

122. Which of this virus cause the common cold in human?

a) Rubella virus

b) Varicella virus

c) Flavi virus

d) Rhino virus

Explanation

Rhino viruses causeone of the most infectious human ailments called the “Common cold”.

123. Which disease infects the liver of the human beings?

a) Mumps

b) Viral hepatitis

c) Measles

d) Poliomyelitis

124. Identify the Incorrect Match.

A. Measles i) Rubella virus

B. Dengue fever ii) Flavi virus

C. Poliomyelitis iii) Polio virus

D. Chicken pox iv) Rhino virus

a) i only

b) ii only

c) iii only

d) iv only

125. Which of the following disease is not caused by a RNA virus?

a) Chicken pox

b) Mumps

c) Measles

d) Poliomyelitis

Explanation

126. On what basis the viral disease are grouped in to four types?

a) Based on the type of virus.

b) Severity of the disease.

c) Symptoms produced in the body organs.

d) Mutation of virus.

Explanation

Viral diseases are generally groupedinto four types on the basis of the symptomsproduced in the body organs.

127. Choose the correct statements regarding the swine flu.

i)Swine flu is caused by the H1N1 virus strain.

ii) Fever, cough, sore throat, weakness and body aches are symptoms of the swine flu.

iii) Children do not have a serious risk from this infection.

a) i only

b) ii only

c) iii only

d) All the above

Explanation

Swine flu was first recognized in the 1919pandemic and still circulates as a seasonal flu virus. Swineflu is caused by the H1N1virus strain. Symptoms include fever, cough,sore throat, chills, weakness and body aches. Children, pregnant women and the elderlyare at risk from severe infection.

128. In which of these human organs the malaria causing plasmodium lives in its mature condition?

a) RBC

b) Liver

c) Trachea

d) Intestines

Explanation

Malaria is caused by different types ofPlasmodium species such as P. vivax, P. ovale,P. malariae and P. falciparum. Plasmodium lives in the RBC of human in itsmature condition it is called as trophozoite.

129. How many hosts are involved in the plasmodium vivax parasite?

a) 3

b) 5

c) 2

d) 1

Explanation

Plasmodium vivax is a digenic parasite, involving two hosts, man as the secondary host and female Anopheles mosquito as the primary host.

130. Identify the correct Match.

A. Quartan i)P. malariae

b. Quotidian ii) P.vivax

C. Benign iii) P.ovale

D. Mild tertian iv) P.falciparum

a) i only

b) ii only

c) iii only

d) iv only

Explanation

131. What is the approximate efficiency of the vaccine RTS,S used to prevent malaria?

a) 10-20%

b) 30%

c) 26-50%

d) Above 75%

Explanation

Malaria vaccine is used to prevent malaria. The only approved vaccine as of 2015 is RTS,S(Mosquirix). It requires four injections and has relatively low efficacy (26–50%).

132. Which of this fungus causes the dermatomycosis infection?

a) Trichophyton

b) Microsporum

c) Epidermophyton.

d) All the above

Explanation

Dermatomycosis is a cutaneous infection caused by fungi belonging to the Genera Trichophyton, Microsporum and Epidermophyton.

133. Choose the correct statements.

i) Ring worm is one of the most common bacterial disease in human.

ii) Dry, scaly lesions in the skin, nails and scalp are the symptoms of this disease.

iii) Heat and moisture reduces the growth of the fungus.

a) I only
b) ii only

c) iii only

d) All the above

Explanation

Ringworm is one of the most common fungal disease in humans. Appearance of dry, scaly lesions on the skin, nails and scalp are the main symptoms of the disease. Heat and moisture help these fungi to grow and makes them to thrive in skin folds.

134. Which of these spread ringworms to human beings?

a) Soil

b) Clothes from infected persons

c) Comb of infected person

d) All the above

Explanation

Ringworms are generally acquired from soil or by using clothes, towels and comb used by infected persons.

135. Which of the following are the symptoms and causes of round worms?

i) Skin irritation

ii) Anemia

iii) Cardio vascular disease

iv) Hepatitis

v) Stunt growth in children

a) i, iii, v only

b) ii, iv, v only

c) iii, v only

d) iv, v only

Explanation

Round worms: The symptoms of the disease are abdominal pain, vomiting, headache, anaemia, irritability and diarrhoea. A heavy infection can cause nutritional deficiency and severe abdominal pain and causes stunted growth in children. It may also cause enteritis, hepatitis and bronchitis.

136. What are the diseases caused by Wuchereria bancrofti in human?

a)Inflammation of lymph nodes

b) Elephantiasis

c) Filariasis

d) All the above

Explanation

Wuchereria bancrofti is sexually dimorphic, viviparous and digenic. The life cycle is completed in two hosts, man and the female Culex mosquito. The female filarial worm gives rise to juveniles called microfilariae larvae. In the lymph glands, the juveniles develop into adults. The accumulation of the worms blocks the lymphatic system resulting in inflammation of the lymph nodes. In some cases, the obstruction of lymph vessels causes elephantiasis or filariasis of the limbs, scrotum and mammary glands.

137. Choose the correct statements.

i) Adolescence begins with a rapid physical development at the age of 10 to 15 years of age.

ii) Adolescence is a highly dynamic period of psychological changes in individuals.

a) i only

b) ii only

c) Both i and ii

d) Neither i nor ii

Explanation

Adolescence begins with a period of rapid physical and sexual development called puberty to maturity at 12 to 19 years of age. Adolescence is also a highly dynamic period of psychological and social changes in individuals.

138. Which of the following statements are not correct regarding the usage of alcohol?

a) Alcohol is a psychoactive drug acts on the brain of a person.

b) Alcohol is depressant rapidly increase the activity of the nervous system.

c) Certain drugs taken other than clinical purpose is called as drug abuse.

d) Drugs abused leads to physical, psychological and physiological impairment.

Explanation

Alcohol is a psychoactive drug, which acts on the brain, affecting a person’s mind and behavior. It is a depressant, which slows down the activity of the nervous system. The intake of certain drugs for a purpose other than their normal clinical use in an amount and frequency that impair one’s physical, physiological and psychological functions is called drug abuse.

139. Name the drugs which are commonly abused?

a) Amphetamines

b) Cannabinoids

c) Coca-alkaloids

d) All the above

Explanation

The drugs which are commonly abused include opioids, cannabinoids, coca-alkaloids, barbiturates, amphetamines and LSD.

140. Which of the following is not a property of heroin?

a) Odorless

b) White color

c) Tasteless

d) Crystalline compound

Explanation

Heroin (smack) is chemically diacetyl morphine, which is white, odourless and bitter crystalline compound. It is obtained by acetylation of morphine, which is extracted from flowers of the poppy plant.

141. What is the main source of marijuana, hashish and charas?

a) Cannobinoids

b) Barbiturates

c) Opioids

d) Poppy plants

Explanation

Cannobinoids are a group of chemicals obtained from Cannabis sativa, the Indian hemp plant. Natural cannabinoids are the main source of marijuana, ganja, hashish and charas.

142. Which of the following statements are correct regarding cocaine?

i) Cocaine is a white powder obtained from the leaves of coca plant.

ii) Commonly called as coke or crack.

iii) Cocaine causes hallucinations and paranoia.

a) i only

b) ii only

c) iii only

d) All the above

Explanation

Cocaine is a white powder that is obtained from the leaves of the coca plant, Erythroxylum coca. It is commonly called coke or crack. Cocaine causes serious physical and psychological problems including hallucinations and paranoia. The other plants with hallucinogenic properties are Atropa belladonna and Datura.

143. Which of the following drug affects the cardio vascular system?

a) Tranquilizers

b) Bhang

c) Cocaine

d) Opium

144. Match the following.

A. Analgesics i) Tobacco

B. Stimulants ii) Morphine

C. Depressants iii) LSD

D. Hallucinogens iv) Alcohol

a)ii, i, iv, iii

b) i, iv, ii, iii

c) iv, i, iii, ii

d) iii, iv, i, ii

Explanation

145. For which of these treatments tranquilizers and LSD are used as a medicine?

a) Bone marrow issues

b) Pancreas disease

c) Depression and insomnia

d) Cardio vascular disease

Explanation

Drugs like methamphetamine, amphetamines, barbiturates, tranquilizers, Lysergic acid diethylamide (LSD) are normally used as medicine to treat patients with mental illness like depression and insomnia and are often abused.

146. Which of the following in tobacco causes oxygen deficiency in the human body?

a) Nicotine

b) Carbon monoxide

c) Lead

d) Tars

Explanation

Tobacco is smoked, chewed and used as snuff. It increases the carbon monoxide content of blood and reduces the concentration of haem bound oxygen, thus causing oxygen deficiency in the body.

147. Which of these stimulates the adrenal glands?

a) Nicotine

b) LSD

c) Opium

d) Tars

Explanation

Tobacco contains nicotine, carbon monoxide and tars, which cause problems in the heart, lung and nervous system. Adrenal glands are stimulated by nicotine to release adrenaline and nor adrenaline which increases blood pressure and heart beat.

148. What are the psychological effects of alcohol and drugs addiction?

a) Life style changes

b) Response time is increased.

c) Euphoria

d) Reduces IQ

Explanation

Addiction to drugs and alcohol can lead to a psychological attachment to certain effects such as euphoria and temporary feeling of well-being.

149. What is called as withdrawal symptoms?

a) Permanent intake of drugs.

b) Abruptly stopping drugs or alcohol.

c) Long time non usage of drugs.

d) Variation in the intake of drugs.

Explanation

Physical dependence is a state in which the user’s body requires a continuous presence of the drug. If the intake of the drug or alcohol is abruptly stopped, he or she would develop withdrawal symptoms.

150. Which of the following is not a withdrawal symptom?

a) Anxiety

b) Liver dysfunction

c) Insomnia

d) Nervousness

Explanation

The withdrawal symptoms may range from mild tremors to convulsions, severe agitation and fits, depressed mood, anxiety, nervousness, restlessness, irritability, insomnia, dryness of throat, etc, depending on the type of drug abuse.

151. Which of the following is termed as liver cirrhosis?

a) Scarring of liver

b) Protein accumulation in liver

c) Decrease growth of liver

d) Malfunction of liver

Explanation

The use of alcohol during adolescence may have long-term effects. Alcohol interferes with the ability of the liver to break down fat. Over time fat accumulation and high levels of alcohol destroy the liver cells and a scar tissue grows in the place of dead cells. This scarring of the liver is called “Liver cirrhosis”.

152. Which is the chronic memory disorder caused by alcohol misuse?

a) Korsakoff syndrome

b) ABCD syndrome

c) Baboon syndrome

d) Eiken syndrome

Explanation

Korsakoff syndrome, a chronic memory disorder is most commonly caused by alcohol misuse.

153. What are the symptoms of mental depression?

a) Loss of self-confidence

b) Anxiety

c) Not being able to enjoy things usually pleasurable

d) All the above

Explanation

Signs and symptoms of mental depression: Loss of self-confidence and self- esteem Anxiety. Not being able to enjoy things that are usually pleasurable or interesting. Lifestyle changes like exercise, meditation, yoga and healthy food habits can help to be relieved from depression. Exercise stimulates the body to produce serotonin and endorphins, which are neurotransmitters that suppress depression.

154. Which of the nfolloowing is nnot caused by vitamin D deficiency?

a) Depression

b) Cardio vascular disease

c) Muscle pain

d) Fatigue

Explanation

The sedentary life style also causes deficiency of vitamins such as vitamin D resulting in fatigue, tiredness, back pain, depression, bone loss, muscle pain, etc,

155. Choose the incorrect statements.

i) Any substance capable of eliciting immune response is called an ANTIGEN.

ii) Two broad classes of immunity responses are innate immunity and acquired immunity.

a) i only

b) ii only

c) Both i and ii

d) Neither i nor ii

Explanation

Any substance capable of eliciting immune response is called anANTIGEN (ANTIbodyGENerator). There are two broad classes of immunity responses namely, innate immunity and acquired immunity.

156. Which of the following is the passive artificial adaptive immunity?

a) Infection

b) Antibody transfer

c) Maternal immunity

d) Immunization

Explanation

157. Which of the following is not a physiological barrier of innate immunity?

a) Temperature

b) Low pH

c) Mucus membrane

d) Chemical mediators

Explanation

158. Which of the following liquids are leaked in tissue damage and infection?

a) Serotonin

b) Histamine

c) Prostaglandins

d) All the above

Explanation

Tissue damage and infection induce leakage of vascular fluid, containing chemotactic signals like serotonin, histamine and prostaglandins. They influx the phagocytic cells into the affected area. This phenomenon is called diapedesis.

159. Choose the correct statements.

i) The primary lymphoid organs provide appropriate environment for lymphocytic maturation.

ii) The secondary lymphoid organs trap antigens and make it available for mature lymphocytes.

a) i only

b) ii only

c) Both i and ii

d) Neither i nor ii

Explanation

Based on their functions, they are classified Into primary or central lymphoid organs and secondary or peripheral lymphoid organs. The primary lymphoid organs provide appropriate environment for lymphocytic maturation. The secondary lymphoid organs trap antigens and make it available for mature lymphocytes, which can effectively fight against these antigens.

160. Which of these cells are seen in the spleen?

a) B cells

b) K cells

c) T cells

d) Both a and c

Explanation

Spleen is a secondary lymphoid organ located in the upper part of the abdominal cavity close to the diaphragm. Spleen contains B and T cells. It brings humoral and cell mediated immunity.

161. Which layer is the outermost of the lymph node?

a) Cortex

b) Cell wall

c) Paracortex

d) Medulla

Explanation

Lymph node has three zones. They are the cortex, paracortex and medulla. The outer most layer of the lymph node is called cortex.

162. Where the Lymphocytes get matured?

a) Bone Marrow

b) Lymph nodes

c) Spleen

d) Thymus gland

Explanation

B lymphocytes (B cells) stay in the bone marrow until they are mature. Then they circulate around the body. Some remain in the blood, while others accumulate in the lymph nodes and spleen. T lymphocytes leave the bone marrow and mature in the thymus gland.

163. Choose the Incorrect statements.

i) Two important types of T cells are Helper T cells and Killer T cells.

ii) Helper T cells release a chemical called cytokine which activates killer cells.

iii) Killer cells destroy cells which are damaged or infected around the body.

a) i only

b) ii only

c) iii only

d) None of the above

Explanation

The two important types of T cells are Helper T cells and Killer T cells. Helper T cells release a chemical called cytokine which activates B cells. Killer cells move around the body and destroy cells which are damaged or infected.

164. Which is not a type of dendritic cell?

a) Interstitial

b) Adjuvant

c) Myeloid

d) Lymphoid

Explanation

Four types of dendritic cells are known. They are Langerhans, interstitial cells, myeloid and lymphoid cells.

165. What is the property of an adjuvant?

a) Develop new antigens.

b) Enhance the immune response to an antigen.

c) Increase the number of antibodies.

d) Decrease the level of immunity.

Explanation

Substances that can enhance the immune response to an antigen are called adjuvants.

166. On what basis the antibodies are classified into five categories?

a) Physical

b) Biochemical

c) Physiological

d) Both b and c

Explanation

The antibodies are classified into five major categories, based on their physiological and biochemical properties. They are IgG(gamma), IgM(mu), IgA (alpha), IgD(delta) and IgE(epsilon).

167. Choose the Incorrect statements.

i) Each chain has a C – terminal (Carboxyl) and amino or N-terminal.

ii) Each chain has much larger constant regions at each end’

iii) Antibodies responding to different antigens have very different (V) regions and similar (C) regions.

a) i only

b) ii only

c) iii only

d) All the above

Explanation

Each chain (L and H) has two terminals. They are C – terminal (Carboxyl) and amino or N-terminal. Each chain (L and H) has two regions. They have variable (V) region at one end and a much larger constant (C) region at the other end. Antibodies responding to different antigens have very different (V) regions but their (C) regions are the same in all antibodies.

168. Which of these is not a function of immunoglobulin?

a) Agglutination

b) Precipitation

c) Sedimentation

d) Neutralization

Explanation

The functions of immunoglobulin are agglutination, precipitation, opsonisation, Neutralization.

169. Which of these bonds hold antigen to the antibody combining site?

a) Hydrogen bonds

b) Vander Waals force

c) Electrostatic bonds

d) All the above

Explanation

The bonds that hold the antigen to the antibody combining site are all non-covalent in nature. These include hydrogen bonds, electrostatic bonds, Van der Waals forces and hydrophobic bonds.

170. Which of these is not the usage of application of antigen-antibody reactions?

a) To determine the characteristics of certain immunodeficiency diseases.

b) To detect the presence or absence of protein in serum.

c) To predict the growth and development of cells.

d) To determine blood groups for transfusion

Explanation

The chief application of antigen – antibody reactions are to determine blood groups for transfusion, to study serological ascertainment of exposure to infectious agents, to develop immunoassays for the quantification of various substances, to detect the presence or absence of protein in serum and to determine the characteristics of certain immunodeficiency diseases.

171. Which of the following has the particulate antigen?

a) Neutralization

b) Complement system

c) Precipitation

d) Agglutination

Explanation

172. Which country developed the BCG vaccine in the year 1908?

a) France

b) Britain

c) Germany

d) Soviet Union

Explanation

BCG vaccine was developed by Calmette and Guerin against tuberculosis in France in the year 1908.

173. How many types of auto immune disease is classified in human beings?

a) 5

b) 2

c) 4

d) 7

Explanation

Autoimmune diseases in human can be divided into two broad categories, namely organ specific and non-organ-specific (systemic) autoimmune diseases.

174. Which of the following gland is affected by the Addison’s disease?

a) Adrenal gland

b) Thyroid gland

c) Pituitary gland

d) Hypothalamus gland

Explanation

In organ-specific disease, the autoimmune process is directed mostly against one organ. The auto antibodies may block the functions performed by the organs. Examples include Hashimoto’s thyroiditis, Graves’ disease (thyroid gland) and Addison’s disease (adrenal glands).

175. Which of these are called cancer?

a) Unchecked growth of group of cells.

b) Tumor or neoplasm

c) Tumor invading healthy tissues

d) All the above

Explanation

A tumour or neoplasm is a group of cells whose growth has gone unchecked. When a tumour continues to grow and invades healthy tissue, it is called cancer.

176. Which of the following property of normal cell is missing in the cancer cells?

a) Uncontrolled growth

b) Nucleus

c) Cell development

d) Power house

Explanation

Normal cells show a property called contact inhibition, which inhibits uncontrolled growth. Cancer cells do not have this property. As a result, cancerous cells divide continuously giving rise to mass of tissues called tumours.

12th Science Lesson 15 Questions in English

15] Biodiversity and its conservation

1. In which year the UN earth summit defined the Biodiversity?

a) 1992

b) 1990

c) 1978

d) 1965

Explanation

The 1992 UN Earth Summit defined Biodiversity as the variability among living organisms from all sources, including terrestrial, marine and other aquatic ecosystems and the ecological complexes of which they are a part. This includes diversity within species, between species and ecosystems of a region.

2. Which of these are not related to the Biodiversity?

a) Reflects the number of different organisms and their relative frequencies.

b) Maintains the ecological processes.

c) Survival of human race depends on wellbeing of plants only.

d) Provides an index of health of an ecosystem.

Explanation

Biodiversity reflects the number of different organisms and their relative frequencies in an ecological system and constitutes the most important functional component of a natural ecosystem. It helps to maintain ecological processes, create soil, recycle nutrients, influence climate, degrade waste and control diseases. It provides an index of health of an ecosystem. The survival of human race depends on the existence and wellbeing of all life forms (plants and animals) in the biosphere.

3. Who introduced the term Biodiversity?

a) Marsden

b) Walter Rosen

c) Earnshaw

d) Richard P.Feynman

Explanation

The term biodiversity was introduced by Walter Rosen (1986).

4. In which direction the number of species per unit area declines due to climatic changes?

a) From tropics towards the poles.

b) From poles to poles

c) Along the equator

d) Around the coastal areas

Explanation

Biodiversity is the assemblage of different life forms. Each species is adapted to live in its specific environments. The changes in climatic conditions are reflected in the distribution and pattern of biodiversity on our planet. The number of species per unit area declines as we move from tropics towards the poles.

5. Choose the incorrect statements.

i) The Tundra and Taiga of northern Canada possess less than 12 species of trees.

ii) The Temperate forests of the United States have about 35 species of trees.

iii) The tropical forests of Panama have only 11 species of trees in a small area.

a) i only

b) ii only

c) iii only

d) None of the above

Explanation

The Tundra and Taiga of northern Canada, Alaska, northern Europe and Russia possess less than 12 species of trees. The temperate forests of the United States have 20-35 species of trees while the tropical forests of Panama have over 110 species of trees in a relatively small area.

6. Who described the biodiversity levels?

a) Edward Wilson

b) W. Pauli

c) Henry Becquerel

d) Fredrick Reines

Explanation

Edward Wilson popularized the term ‘Biodiversity’ to describe diversity at all levels of biological organization from populations to biomes.

7. Which of the following is not a biodiversity level?

a) Genetic diversity

b) Community diversity

c) Species diversity

d) Population diversity

Explanation

There are three levels of biodiversity – Genetic diversity, Species diversity and Community/Ecosystem diversity.

8. Which of these is referred as Genetic diversity?

a) Genetic variations between distinct populations of the same species.

b) Types of genes between distinct species.

c) Genetic variation within a single species.

d) All the above

Explanation

Genetic diversity refers to the differences in genetic make-up (number and types of genes) between distinct species and to the genetic variation within a single species; also covers genetic variation between distinct populations of the same species.

9. Which of this medicinal plant of Himalaya ranges shows differences in the potency?

a) Curcuma longa

b) Rouwolfia vomitaria

c) Phyllanthus emblica

d) Piper nigrum

Explanation

Rouwolfia vomitaria, a medicinal plant growing in different ranges of the Himalayas shows differences in the potency and concentration of the active ingredient reserpine due to genetic diversity. Genetic diversity helps in developing adaptations to changing environmental conditions.

10. Define species richness.

a) Variety in number of species.

b) Number of species per unit area at a specific time.

c) Number of diverse species

d) Ratio of total species to the fittest.

Explanation

Species diversity refers to the variety in number and richness of the species in any habitat. The number of species per unit area at a specific time is called species richness, which denotes the measure of species diversity.

11. Assertion (A): The Western Ghats of India have greater amphibian species diversity than the Eastern Ghats.

Reasoning(R): The more number of species in an area leads to more richness of the species.

a) Both A and R is True and R is the correct explanation of A.

b) Both A and R is True but R is not the correct explanation of A.

c) A is True but R is False.

d) Both A and R is False.

Explanation

The Western Ghats have greater amphibian species diversity than the Eastern Ghats. The more the number of species in an area the more is the species richness.

12. Which of the following is not a diversity index?

a) Alpha

b) Tesla

c) Beta

d) Gamma

Explanation

The three indices of diversity are – Alpha, Beta and Gamma diversity.

13. Which of these refers to the diversity of the habitats over the landscape?

a) Alpha diversity

b) Gamma diversity

c) Beta diversity

d) None of the above

Explanation

Alpha diversity: It is measured by counting the number of taxa (usually species) within a particular area, community or ecosystem.

Beta diversity: It is species diversity between two adjacent ecosystems and is obtaining by comparing the number of species unique to each of the ecosystem.

Gamma diversity refers to the diversity of the habitats over the total landscape or geographical area.

14. Which of this diversity exists at the ecosystem level?

a) Community diversity

b) Alpha diversity

c) Beta diversity

d) Gamma diversity

Explanation

Community/Ecosystem diversity is the variety of habitats, biotic communities, and ecological processes in the biosphere. It is the diversity at ecosystem level due to diversity of riches, trophic levels and ecological processes like nutrient cycles, food webs, energy flow and several biotic interactions. India with its alpine meadows, rain forests, mangroves, coral reefs, grass lands and deserts has one of the greatest ecosystem diversity on earth.

15. Which of these is known as Taxonomic impediment?

a) Quantified number of species in a region at a given time.

b) Number of extinct species.

c) The exact magnitude of natural wealth.

d) All the above

Explanation

Biodiversity is often quantified as the number of species in a region at a given time. The current estimate of different species on earth is around 8-9 million. However, we really don’t know the exact magnitude of our natural wealth. This is called the ‘The Taxonomic impediment’.

16. Which of these represents the biomes of India?

a) Tropical humid forests

b) Tropical dry forests

c) Warm deserts

d) All the above

Explanation

India is very rich in terms of biological diversity due to its unique bio-geographical location, diversified climatic conditions and enormous eco-diversity and geo-diversity. According to world biogeographic classification, India represents two of the major realms (The Palearctic and Indo-Malayan) and three biomes (Tropical humid forests, Tropical Dry/ Deciduous forests and Warm Deserts/Semi deserts). With only about 2.4% of the world’s total land surface, India is known to have over 8 % of the species of animals that the world holds and this percentage accounts for about 92,000 known species.

17. What is the global position of India in terms of area?

a) 7th

b) 2nd

c) 10th

d) 20th

Explanation

India is the seventh largest country in the world in terms of area. India has a variety of ecosystems, biomes with its varied habitats like, hills, valleys, plateaus, sea shores, mangroves, estuaries, glaciers, grasslands and river basins. It also reflects different kinds of climates, precipitation, temperature distribution, river flow and soil. India is one of the 17 mega biodiversity countries of the world and has ten biogeographic zones with characteristic habitat and biota.

18. Assertion (A): The Habitat conditions are determined by the latitude and altitudes.

Reasoning(R): Organisms require different sets of conditions for their growth and metabolism.

a) Both A and R is True and R is the correct explanation of A.

b) Both A and R is True but R is not the correct explanation of A.

c) A is True but R is False.

d) Both A and R is False.

Explanation

The distribution of plants and animals is not uniform around the world. Organisms require different sets of conditions for their optimum metabolism and growth. Within this optimal range (habitat) a large number and type of organisms are likely to occur, grow and multiply. The habitat conditions are determined by their latitudes and altitudes.

19. Which of these factors determine the biodiversity distribution patterns?

a) Temperature

b) Distance of the equator

c) Altitude from sea level

d) All the above

Explanation

Latitudinal and altitudinal gradients: Temperature, precipitation, distance from the equator (latitudinal gradient), altitude from sea level (altitudinal gradient) is some of the factors that determine biodiversity distribution patterns. The most important pattern of biodiversity is latitudinal gradient in diversity. This means that there is an increasing diversity from the poles to equator.

20. In which point of the Earth zones diversity reaches maximum?

a) Tropics

b) Poles

c) Equator

d) Sea level

Explanation

Diversity increases as one moves towards the temperate zones and reaches the maximum at the tropics. Thus, tropics harbour more biodiversity than temperate or polar regions, especially between the latitudes of 23.5⁰N and 23.5⁰S (Tropic of Cancer to the Tropic of Capricorn). Harsh conditions exist in temperate areas during the cold seasons while very harsh conditions prevail for most of the year in Polar Regions.

21. Which of the factors increases the species diversity?

a) Temperature

b) Population

c) Latitude

d) Fertility

Explanation

Columbia located near the equator (0⁰) has nearly 1400 species of birds while New York at 41⁰N has 105 species and Greenland at 71⁰N has 56 species. India, with much of its land area in the tropical latitudes, is home for more than 1200 species of birds. Thus it is evident that the latitude increases the species diversity.

22. What is the effect of drop in temperature in species diversity?

a) Increases

b) No change

c) Decreases

d) Proportional change

Explanation

Decrease in species diversity occurs as one ascends a high mountain due to drop in temperature (temperature decreases @ 6.5⁰C per Km above mean sea level)

23. Which of these reasons increases the richness of biodiversity in the tropics?

a) Environmental conditions support variety of organisms.

b) Average rainfall is more than 200 mm per year.

c) Rich resource and nutrient availability.

d) All the above

Explanation

The reasons for the richness of biodiversity in the Tropics are: Warm tropical regions between the tropic of Cancer and Capricorn on either side of equator possess congenial habitats for living organisms. Environmental conditions of the tropics are favorable not only for speciation but also for supporting both variety and number of organisms. The temperatures vary between 25⁰C to 35⁰C, a range in which most metabolic activities of living organisms occur with ease and efficiency. The average rainfall is often more than 200 mm per year. Climate, seasons, temperature, humidity, photoperiods are more or less stable and encourage both variety and numbers. rich resource and nutrient availability.

24. State the relationship between the species richness and its area.

a) log S = log C + Z log A

b) log S = log C + A

c) log S = log A + Z log C

d) log S = log A + log Z

Explanation

German Naturalist and Geographer Alexander von Humboldt explored the wilderness of South American jungles and found that within a region the species richness increased with increasing area but up to a certain limit. The relationship between species richness and area for a wide variety of taxa (angiosperm plants, birds, bats, freshwater fishes) turned out to be the rectangular hyperbola. On a logarithmic scale, the relationship is a straight line described by the equation.

log S = log C + Z log A where,

S = Species richness

A = Area

Z = Slope of the line (regression coefficient)

C = Y-intercept

25. Assertion (A): Biodiversity is denoted as the number of different species of flora and fauna including microorganisms.

Reasoning(R): Organisms can inhabit different ecosystems with varying conditions and it is essential for the wellbeing of our planet.

a) Both A and R is True and R is the correct explanation of A.

b) Both A and R is True but R is not the correct explanation of A.

c) A is True but R is False.

d) Both A and R is False.

Explanation

Biodiversity is the variety of life on earth. That is, it is the number of different species of flora and fauna including microorganisms. These organisms can inhabit different ecosystems with varying conditions like the Rainforests, Coral reefs, Grasslands, Deserts, Tundra and the Polar ice caps. This variety (Biodiversity) is essential for the wellbeing of our planet and sustenance of life as a whole.

26. Who proposed the Rivet Popper Hypothesis concept related to the ecosystem?

a) Paul Ehrlich

b) Neil’s Bohr

c) James Chadwick

d) Rouwolfia vomitaria

Explanation

Ecologist Paul Ehrlich proposed the ‘Rivet Popper Hypothesis’ for better understanding the loss of each species in the ecosystem. He compared each species of an ecosystem with rivets in the body of an aero plane. Thousands of rivets (species) join all the parts of an aero plane (ecosystem). If every passenger travelling in the aero plane starts taking rivets home (loss of species), initially it may not affect flight safety (proper functioning of the ecosystem). However, the plane becomes dangerously weak over a period of time, when more and more rivets are removed. Moreover, which rivet is removed is also important. When the key rivets (removal of key stone species) on the wings of the aero plane are removed, undoubtedly it poses serious threat to the flight safety. Thus we understand the role of every species for the hormonal function of an ecosystem.

27. Which of these is not an important factor of biodiversity?

a) Ecosystem Services

b) Tradeoff between species and its richness

c) Biological resources

d) Social Benefits

Explanation

The importance of biodiversity can be viewed and measured as Ecosystem services, Biological resources, and social benefits of biodiversity.

28. Which of these are the functional attributes of the Biodiversity?

a) Soil formation and soil health

b) Water flow regulators

c) Climate stability

d) All the above

Explanation

The organization and functioning of ecosystems world over is effected and dependent on biodiversity and its richness. The major functional attributes are: continuity of nutrient cycles or biogeochemical cycles (N2, C, H2O, P, S cycles) Soil formation, conditioning or maintenance of soil health (fertility) by soil microbial diversity along with the different trophic members, increases ecosystem productivity and provide food resources, act as water traps, filters, water flow regulators and water purifiers (forest cover and vegetation) climate stability (forests are essential for rainfall, temperature regulation, CO2 absorption, which in turn regulate the density and type of vegetation), forest resource management and sustainable development, maintaining balance between biotic components cleaning up of pollutants – microbes are the biggest degraders of molecules including many anthropogenic ones which are present in effluents, sewage, garbage and agro-chemicals, ecological stability – the varieties and richness of species contribute to ecological stability and survival of species.

29. Which of these is a crucial indicator of the species richness?

a) Endemism

b) Environment

c) Extinction

d) Survival

Explanation

Bio diverse regions are reservoirs of biological resources like food resources, gene pool, genetic resource, and medicinal resources, bio-prospecting to provide unique aesthetic value and hot spots for Ecotourism. Along with forest resources and wildlife it has commercial significance an indicator of the health of the ecosystem. Endemism is a crucial indicator of richness.

30. Which of the following zone is not included in the Trans Himalaya?

a) Ladakh

b) North Sikkim

c) Aravalli hill

d) Lahaul

31. Which of the zones have thorn and deciduous forests?

a) Thar

b) Ladakh

c) Eastern Ghats

d) Sundarbans

Explanation

Bio geographical Zones of India

32. Which of these zones comprises the larger area in the Indian subcontinent?

a) Deccan Peninsula

b) Western Ghats

c) Semi-arid zones

d) Gangetic plains

Explanation

33. Which of these forests is associated with the Andaman and Nicobar Islands?

a) Thorn forests

b) Deciduous Forests

c) Evergreen Forests

d) Mangroves

Explanation

34. Which of these is the major cause for decline in the biodiversity?

a) Habitat loss

b) Climate change

c) Natural disasters

d) All the above

Explanation

The major causes for biodiversity decline are: Habitat loss, fragmentation and destruction (affects about 73% of all species) Pollution and pollutants (smog, pesticides, herbicides, oil slicks, GHGs), Climate change, Introduction of alien/exotic species Over exploitation of resources (poaching, indiscriminate cutting of trees, over fishing, hunting, mining), Intensive agriculture and aquaculture practices, Hybridization between native and nonnative species and loss of native species, Natural disasters (Tsunami, forest fire, earth quake, volcanoes) Industrialization, Urbanization, infrastructure development, Transport –Road and Shipping activity, communication towers, dam construction, unregulated tourism and monoculture are common area of specific threats, Co-extinction

35. Assertion (A): Habitat loss is mainly for the benefit of human society development.

Reasoning(R): Species become victim to predation, starvation or eventually die in human animal conflicts.

a) Both A and R is True and R is the correct explanation of A.

b) Both A and R is True but R is not the correct explanation of A.

c) A is True but R is False.

d) Both A and R is False.

Explanation

Habitat Loss: Development of human society is inevitable. Natural habitats are destroyed for the purpose of settlement, agriculture, mining, industries and construction of highways. As a result species are forced to adapt to the changes in the environment or move to other places. If not, they become victim to predation, starvation, disease and eventually die or results in human animal conflict.

36. By which of these ways the fragmentation and destruction of natural habitats happen?

a) Constructing transport ways

b) Extracting ores

c) Changing the course of rivers

d) All the above

Explanation

Over population, urbanization, industrialization and agricultural advancements require additional land, water and raw materials every year. This is made possible only through fragmentation or destruction of natural habitats by filling wetlands, ploughing grasslands, cutting down trees, forest, desilting rivers, constructing transport ways, caving mountains, extracting, and ores, changing the course of rivers and filling of seashore.

37. Which of these are the effects of Habitat fragmentation?

a) Large continuous area of habitat is reduced in area.

b) Destruction of complex interactions among the species.

c) Decreased bio diversity in the habitat fragments.

d) All the above

Explanation

Habitat fragmentation is the process where a large, continuous area of habitat is both, reduced in area and divided into two or more fragments. Fragmentation of habitats like forest land into crop lands, orchard lands, plantations, urban areas, industrial estates, transport and transit systems has resulted in the destruction of complex interactions amongst species, (food chain and webs) destruction of species in the cleared regions, annihilation of species restricted to these habitats (endemic) and decreased biodiversity in the habitat fragments. Animals requiring large territories such as mammals and birds are severely affected. The elephant corridors and migratory routes are highly vulnerable. The dwindling of many well-known birds (sparrows) and animals can be attributed to this.

38. State some of the species became extinct due to over exploitation?

a) Dodo

b) Steller’s sea cow

c) Passenger pigeon

d) All the above

Explanation

Over exploitation: We depend on nature for our basic needs such as food and shelter. However, when the need becomes greed, it leads to over exploitation of natural resources. Excessive exploitation of a species reduces the size of its population to such a level that it becomes vulnerable to extinction. Dodo, passenger pigeon and Steller’s sea cow have become extinct in the last 200-300 years due to over exploitation by humans. Overfishing due to population pressure leads to many marine fish (populations) declining around the world.

39. Which of these statements are not true regarding the exotic species?

i) Exotic species are non-native or alien introduced intentionally for commercial purposes.

ii) Exotic species become invasive and drive away the local species.

iii) Exotic species are not harmful to both aquatic and terrestrial ecosystems.

a) i only

b) ii only

c) iii only

d) None of the above

Explanation

Exotic species (non-native; alien) are organisms often introduced unintentionally or deliberately for commercial purpose, as biological control agents and other uses. They often become invasive and drive away the local species and are considered as the second major cause for extinction of species. Exotic species have proved harmful to both aquatic and terrestrial ecosystems.

40. From which of these countries the Paracoccus marginatus originated?

a) Central America

b) China

c) New Zealand

d) South Africa

Explanation

The introduction of the Nile Perch, a predatory fish into Lake Victoria in East Africa led to the extinction of an ecologically unique assemblage of more than 200 nature species of cichlid fish in the lake. Papaya Mealy Bug (Paracoccus marginatus) is native of Mexico and Central America, is believed to have destroyed huge crops of papaya in Assam, West Bengal and Tamil Nadu.

41. Which of the following is not an effect of Industrialization?

a) Economy Deficit

b) Extreme weather conditions

c) Large scale of deforestation

d) Glaciers melting

Explanation

Industrialization is a major contributor to climate change and a major threat to biodiversity. Energy drives our industries are provided by burning of fossil fuels. This increases the emission of CO2, a GHG leading to climate change. Due to large scale deforestation, the emitted CO2 cannot be absorbed fully, and its concentration in the air increases..

42. What are the negative effects of the shifting cultivation?

a) Loss of forest cover

b) CO2 discharge

c) Climate change

d) All the above

Explanation

Shifting or Jhum cultivation (Slash-and-burn agriculture): In shifting cultivation, plots of natural tree vegetation are burnt away and the cleared patches are farmed for 2-3 seasons, after which their fertility reduces to a point where crop production is no longer profitable. The farmer then abandons this patch and cuts down a new patch of forest trees elsewhere for crop production. This system is practiced in north-eastern regions of India. When vast areas are cleared and burnt, it results in loss of forest cover, pollution and discharge of CO2 which in turn attributes to loss of habitat and climate change which has an impact on the faunal diversity of that region.

43. Which of the following concepts explain the relation between the Calvaria tree and Dodo bird?

a) Co extinction of species

b) Intensive agriculture

c) Shifting Cultivation

d) Parasites

Explanation

Co extinction of a species is the loss of a species as a consequence of the extinction of another. (Eg., orchid bees and forest trees by cross pollination). Extinction of one will automatically cause extinction of the other. Another example for co-extinction is the connection between Calvaria tree and the extinct bird of Mauritius Island, the Dodo. The Calvaria tree is dependent on the Dodo bird for completion of its life cycle. The mutualistic association is that the tough horny endocarp of the seeds of Calvaria tree are made permeable by the actions of the large stones in bird’s gizzard and digestive juices thereby facilitating easier germination. The extinction of the Dodo bird led to the imminent danger of the Calvaria tree co extinction.

44. What is the base for the Intensive Agriculture?

a) High yielding varieties

b) Short time yield

c) Hybrid varieties

d) All the above

Explanation

Intensive agriculture: Spread of agriculture is sometimes at the cost of wetlands, grasslands and forests. Intensive agriculture is based on a few high yielding varieties. As a result, there is reduction in the genetic diversity. It also increases vulnerability of the crop plants to sudden attack by pathogens and pests. There are only few varieties of traditional paddy strains today due to use to hybrid varieties in Tamil Nadu.

45. Assertion (A): Extinct species destabilizes the ecological stability and the distribution of biological diversity on earth.

Reasoning(R): Natural sources and organisms are indiscriminately exploited by human beings.

a) Both A and R is True and R is the correct explanation of A.

b) Both A and R is True but R is not the correct explanation of A.

c) A is True but R is False.

d) Both A and R is False.

Explanation

Loss of biodiversity: Species have been evolving and dying out (extinction) ever since the origin of life. However, species are now becoming extinct at a faster rate. This is destabilizing the ecological stability and the distribution of biological diversity on earth. Human activities greatly contribute to the loss of biodiversity. Natural resources such as land, water and organisms are indiscriminately exploited by human beings.

46. Choose the Incorrect statements.

i) Direct and Indirect human activities have a detrimental effect on biodiversity.

ii) Harvesting, Pollution and Climate changes are the direct human activities in the biodiversity loss.

iii) The Indirect human drivers include demographic, economic, technological and religious factors.

a) i only

b) ii only

c) iii only

d) None of the above

Explanation

According to the Convention of Biological Diversity, direct and indirect human activities have a detrimental effect on biodiversity. Direct human activities like change in local land use, species introduction or removal, harvesting, pollution and climate change contribute a greater pressure on loss of biodiversity. Indirect human drivers include demographic, economic, technological, cultural and religious factors.

47. Which is not an indirect effect of human activities in the biodiversity loss?

a) Monsoon failure

b) Global warming

c) Species Extinction

d) Depletion in ozone layer

Explanation

Even though new species are being discovered, there is little hope for adding new species through speciation into the biodiversity treasure. Monsoon failure, global warming, depletion in ozone layer, landslides in hilly states, pollution are a few indirect effects of human activities which results in the loss biodiversity.

48. Which of these are the negative effects of the biodiversity loss?

a) Reduced ecosystem services

b) Adverse effect of the food chain

c) Species reduction

d) All the above

Explanation

The loss of biodiversity has a immense impact on plant animal and human life. The negative effects include dramatic influence on the food web. Even reduction in one species can adversely affect the entire food chain which further leads to an overall reduction in biodiversity. Reduced biodiversity leads to immediate danger for food security by reducing ecosystem services.

49. Which of the following statements is not true regarding hotspot?

a) High concentration of endangered species.

b) Great diversity of endemic species impacted and altered by human activities.

c) Hotspot supports endemic vascular plants and gained more than 25% of its original vegetation.

d) There are about 1000 biodiversity hotspots in the world.

Explanation

Hotspots are areas characterized with high concentration of endemic species experiencing unusual rapid rate of habitat modification loss. Norman Myers defined hot spots as “regions that harbour a great diversity of endemic species and at the same time, have been significantly impacted and altered by human activities.” A hotspot is a region that supports at least 1500 endemic vascular plant species (0.5% of the global total) has lost more than 70% of its original vegetation. There are 35 biodiversity hotspots in the world.

50. Which of these is not a bio diversity hotspot of India?

a) Himalaya

b) Indian Ocean

c) Sunderland’s

d) Western Ghats

Explanation

India is home to four biodiversity hotspots (as per ENVIS). They are Himalaya (the entire Indian Himalayan region), Western Ghats Indo-Burma: includes entire North-eastern India, except Assam and Andaman group of Islands (and Myanmar, Thailand, Vietnam, Laos, Cambodia and Southern China) , Sunderland’s: includes Nicobar group of Islands (and Indonesia, Malaysia, Singapore, Brunei, Philippines).

51. Assertion (A): Endangered species categorized by the International Union for Conservation of Nature are likely to be extinct.

Reasoning(R): Red list species are the Critically Endangered conservation status for wild populations.

a) Both A and R is True and R is the correct explanation of A.

b) Both A and R is True but R is not the correct explanation of A.

c) A is True but R is False.

d) Both A and R is False.

Explanation

A species that has been categorized as very likely to become extinct is an Endangered species. Endangered (EN), as categorized by the International Union for Conservation of Nature (IUCN) Red List, is the second most severe conservation status for wild populations in the IUCN’s scheme after Critically Endangered (CR).

52. Which of this factor decides the extinction of species?

a) Drastic environmental changes

b) Atmospheric conditions

c) Population characteristics

d) Both a and c

Explanation

The extinction of species is mainly due to drastic environmental changes and population characteristics. There are three types of Extinctions

53. Assertion (A): Natural Extinction is a slow process of replacing existing species with better adapted species.

Reasoning(R): A small population can get extinct sooner than the larger population.

a) Both A and R is True and R is the correct explanation of A.

b) Both A and R is True but R is not the correct explanation of A.

c) A is True but R is False.

d) Both A and R is False.

Explanation

Natural extinction is a slow process of replacement of existing species with better adapted species due to changes in environmental conditions, evolutionary changes, predators and diseases. A small population can get extinct sooner than the large population due to inbreeding depression (less adaptivity and variation).

54. In which year the Government of India launched the Project Tiger?

a) 1973

b) 1987

c) 1977

d) 1972

Explanation

The Government of India launched the ‘Project Tiger’ in 1973 to protect our national animal. From 9 tiger reserves since its inception, the Project Tiger coverage has increased to 50 at present. Project Tiger is an ongoing Centrally Sponsored Scheme of the Ministry of Environment and Forests, providing central assistance to the states for tiger conservation in designated tiger reserves. Project Tiger was launched in the Jim Corbett National Park, Uttarakhand in 1973. The project ensures a viable population of Bengal tigers in their natural habitats, protecting them from extinction and preserving areas of biological importance as a natural heritage.

55. Which of these causes the Mass extinction in Earth?

a) UV rays

b) Pandemic

c) Environmental Catastrophes

d) Sea level rise

Explanation

Mass extinction: The earth has experienced quite a few mass extinctions due to environmental catastrophes. A mass extinction occurred about 225 million years ago during the Permian, where 90% of shallow water marine invertebrates disappeared.

56. Which of these human activities causes the anthropogenic extinctions?

a) Habitat destruction

b) Over exploitation

c) Urbanization

d) All the above

Explanation

Anthropogenic extinctions are abetted by human activities like hunting, habitat destruction, over exploitation, urbanization and industrialization. Some examples of extinctions are Dodo of Mauritius and Steller’s sea cow of Russia. Amphibians seem to be at higher risk of extinction because of habitat destruction.

57. Which of the following statements are not true regarding IUCN?

i) The IUCN was established in the year 1948 and located at Switzerland.

ii) The mission of IUCN is to influence encourage and assist societies to conserve nature.

iii) IUCN passed several laws through international conventions on nature conservation and biodiversity.

a) i only

b) ii only

c) iii only

d) None of the above

Explanation

The International Union for Conservation of Nature (IUCN) is an organization working in the field of nature conservation and sustainable use of natural resources. It was established in 1948 and located at Gland VD, Switzerland. It is involved in data gathering and analysis research, field projects and education on conservation, sustainable development and biodiversity. IUCN’s mission is to influence, encourage and assist societies throughout the world to conserve nature and to ensure that any use of natural resources is equitable and ecologically sustainable. It influences governments and industries through partnerships by providing information and advice. The organization collects, compiles and publishes the IUCN red list of threatened species and their conservation status in the world. It plays a vital role in the implementation of several international conventions on nature conservation and biodiversity.

58. In which year the concept of Red list was mooted by IUCN?

a) 1963

b) 1976

c) 1947

d) 1955

Explanation

Red Data book or Red list is a catalogue of taxa facing risk of extinction. IUCN – International Union of Conservation of Nature and Natural Resources, which is renamed as WCU – World Conservation Union (Morges Switzerland) maintains the Red Data book. The concept of Red list was mooted in 1963.

59. Which of the following is not a purpose of Red list?

a) Identifying and documenting high risk extinct species.

b) To create awareness on the threat degree of biodiversity.

c) To enact laws and agreements on biological diversity conservation.

d) Preparing conservation priorities and conservation in action.

Explanation

The purpose of preparation of Red List are: To create awareness on the degree of threat to biodiversity, Identification and documentation of species at high risk of extinction, Provide global index on declining biodiversity, Preparing conservation priorities and help in conservation of action, Information on international agreements on conservation of biological diversity.

60. State the species which is not categorized in the Red list.

a) Vulnerable

b) Not evaluated

c) High risk

d) Data deficiency

Explanation

Red list has eight categories of species: Extinct, Extinct in wild, Critically Endangered, Endangered, Vulnerable, Lower risk, Data deficiency, Not evaluated.

61. In which year the Principle of Stockholm declaration was passed?

a) 1978

b) 1972

c) 1982

d) 1963

Explanation

The natural resources of the Earth, including air, water, land, flora and fauna of natural ecosystems must be safeguarded for the benefit of the present and future generations through careful planning and management, as appropriate – Principle of the Stockholm Declaration, 1972.

62. Which of these denotes the conservation of biodiversity?

a) Protecting and scientific management of biodiversity.

b) To maintain optimum level of biodiversity.

c) Protect species from extinction and their habitats.

d) All the above

Explanation

Conservation of biodiversity is protection and scientific management of biodiversity so as to maintain it at its optimum level and derive sustainable benefits for the present as well as future generations. It aims to protect species from extinction and their habitats and ecosystems from degradation.

63. What are the general strategies in conservation?

a) Identify and protect critical habitats

b) Implementing the Wildlife Protection Act.

c) Protecting air, water and soil on priority basis.

d) All the above

Explanation

General strategies in conservation: Identify and protect all threatened species, identify and conserve in protected areas the wild relatives of all the economically important organisms Identify and protect critical habitats for feeding, breeding, nursing, resting of each species, resting, feeding and breeding places of the organisms should be identified and protected, air, water and soil should be conserved on priority basis, wildlife Protection Act should be implemented.

64. What is defined as In-situ conservation?

a) Conservation of genetic resources within a natural or man-made ecosystem.

b) Breeding new species in an artificial ecosystem.

c) Protecting wild life sanctuaries and Forests.

d) Conservation of new species and habitats in a closed environment.

Explanation

In-situ Conservation (Conservation in the natural habitat): This is the conservation of genetic resources through their protection within a natural or manmade ecosystem in which they occur. It is conservation and protection of the whole ecosystem and its biodiversity at all levels in order to protect the threatened species.

65. Assertion (A): Protected Areas are bio geographical areas managed and maintained through legal measures.

Reasoning(R): Protected Areas includes National Parks, Wild Life Sanctuaries, community reserves and Biosphere reserves.

a) Both A and R is True and R is the correct explanation of A.

b) Both A and R is True but R is not the correct explanation of A.

c) A is True but R is False.

d) Both A and R is False.

Explanation

Protected Areas: These are bio geographical areas where biological diversity along with natural and cultural resources is protected, maintained and managed through legal measures. Protected areas include national parks, wild life sanctuaries, community reserves and biosphere reserves.

66. Which of the chapters of Indian Constitution deals with the Wildlife Protection Acts?

a) CHAPTER X

b) CHAPTER II

c) CHAPTER IV

d) CHAPTER I

Explanation

National Parks (NP): It is a natural habitat that is notified by the state government to be constituted as a National Park due to its ecological, faunal, floral, geomorphological, or zoological association of importance. No human activity is permitted inside the national park except the activities permitted by the Chief Wildlife Warden of the state under the conditions given in CHAPTER IV, of the Wildlife Protection Act (WPA) 1972.

67. Match

National Parks Year of Establishment

A. Mudumalai i) 2001

B. Gulf of Mannar ii) 1976

C. Indira Gandhi iii) 1940

D. Mukkuruthi iv) 1986

a) iii, iv, ii, i

b) ii, i, iii, iv

c) iv, iii, i, ii

d) ii, iii, iv, i

Explanation

National Parks in Tamil Nadu

68. Choose the correct statements.

i) Any area including reserve forests or territorial waters can be notified as sanctuary by the State Government.

ii) As per the Chapter IV of Wildlife Act restricted human activities are allowed inside the Sanctuary areas of State Government.

iii) Eco tourism is not permitted inside the sanctuaries protected by the State Government.

a) i only

b) ii only

c) iii only

d) All the above

Explanation

Any area other than the area comprised with any reserve forest or the territorial waters can be notified by the State Government to constitute as a sanctuary if such area is of adequate ecological, faunal, floral, geomorphological, natural or zoological significance. This is for the purpose of protecting, endangered factual species. Some restricted human activities are allowed inside the Sanctuary area details of which are given in CHAPTER IV, of the Wildlife Protection Act (WPA) 1972. Eco tourism is permitted, as long as animal life is undisturbed.

69. Which of the following wild life sanctuaries are known for the Indian Tiger and Asiatic elephant?

a) Periyar Wild life sanctuary

b) Vedanthangal Wild life sanctuary

c) Guindy National Park

d) Kaziranga National Park

Explanation

Periyar wild life sanctuary in Kerala is famous for the Indian Tiger and Asiatic Elephant.

70. Identify the Incorrect Match.

A. Coimbatore i) Indira Gandhi WLS

B. Nilgiris ii) Mundanthurai WLS

C. Nagapattinam iii) Point Calimere WLS

D. Chengalpet iv) Vedanthangal WLS

a) i only

b) ii only

c) iii only

d) iv only

71. In which year the Mudumalai Wild Life Sanctuary was established?

a) 2001

b) 1987

c) 1992

d) 1942

Explanation

Prominent WLS in Tamil Nadu

72. Who vision the Madras Crocodile Bank Trust and Centre for Herpetology?

a) James Chadwick

b) George Zweig

c) F. Strassman

d) Romulus Whitaker

Explanation

The Madras Crocodile Bank Trust and Centre for Herpetology was the brain child of the legendary Romulus Whitaker and a handful of like-minded conservation visionaries, who began work on the facility in 1976. It aimed to save India’s dwindling crocodilian population. The mission is to promote the conservation of reptiles and amphibians and their habitats through education, scientific research and capture breeding. The crocodile bank remains a world leader in the field of frontline conservation and the preservation of natural landscapes.

73. Assertion (A): Biosphere reserve is an international designation by UNESCO.

Reasoning(R): Biosphere reserves are designated to deal with the conservation of biodiversity, economic and social development and maintaining cultural values.

a) Both A and R is True and R is the correct explanation of A.

b) Both A and R is True but R is not the correct explanation of A.

c) A is True but R is False.

d) Both A and R is False.

Explanation

Biosphere Reserve (BR) is an international designation by UNESCO for representative parts of natural and cultural landscapes extending over large area of terrestrial or coastal/ marine ecosystems or a combination thereof. BRs are designated to deal with the conservation of biodiversity, economic and social development and maintenance of associated cultural values.

74. Choose the correct statements regarding the Insitu Conservation.

i) On-site conservation of genetic resources in natural populations of plant or animal species.

ii) Recovering populations and preventing their extinctions.

iii) National parks and Wild Life Sanctuaries are not from the Insitu strategies.

a) i only

b) ii only

c) iii only

d) All the above

75. Which of the following statements are not related to Exsitu Conservation?

a) Zoological parks and Botanical gardens are common Exsitu conservation programs.

b) Conservation strategies placing of threatened animals and plants in special care locations.

c) Process of protecting endangered plants and animal species.

d) Helps in recovering species from extinction under simulated conditions.

Explanation

Difference between Insitu and Exsitu Conservation

76. Which of the following is preserved by the Gene banks?

a) Genetic materials and seeds

b) Extinct species

c) New species information

d) Genitival research works

Explanation

Gene banks are a type of bio repository which preserves genetic materials. Seeds of different genetic strains of commercially important plants can be stored in long periods in seed banks, gametes of threatened species can be preserved in viable and fertile condition for long periods using cryopreservation techniques.

77. In which year the World Wild Fund for Nature was founded?

a) 1992

b) 1978

c) 1961

d) 1920

Explanation

World Wild Fund for Nature (WWF) is an international non-governmental charitable trust founded in 1961, with headquarters at Gland, Vaud, Switzerland. It aims at wildness preservation and the reduction of human impact on the environment. It was formerly named the World Wildlife Fund. The living planet report is being published every two years by WWF since 1998.

78. Choose the correct statements.

i) The vision of WWF is to conserve nature and reduce the most pressing threats to the diversity of life on Earth.

ii) WWF ensures that the value of nature is reflected in decision made by individuals, communities, governments and businesses.

a) i only

b) ii only

c) Both i and ii

d) Neither i nor ii

Explanation

The vision of WWF is to conserve nature and reduce the most pressing threats to the diversity of life on Earth by conserving the world’s most ecologically important regions, protect and restore species and their habitats, strengthen local communities’ ability to conserve the natural resources they depend upon and to ensure that the value of nature is reflected in decision made by individuals, communities, governments and businesses.

79. Which of the following is not an objective of ZSI?

a) Periodic review of the status of the threatened and endemic species.

b) Establishing new Zoological centers by continuous monitoring.

c) Red Data Book preparation and Fauna of India.

d) Maintenance and Development of National Zoological Collections.

Explanation

The objectives of ZSI are: Exploration, Survey, Inventorying and Monitoring of faunal diversity in various states, ecosystems and protected areas of India. Periodic review of the status of threatened and endemic species. Preparation of Red Data Book and Fauna of India. Biological studies on selected important species. Maintenance and Development of National Zoological Collections.

80. Which of these strategies is employed by the FREEP in India?

a) Eco development

b) Eco Tourism

c) Eco Friendly

d) Ecological services

Explanation

The Forestry Research Education and Extension Project FREEP (A World Bank Initiative) in India is employing a strategy called ‘eco development’ which enlists local commodities in the preservation of biodiversity. The strategy involves developing alternate resources and sources of income for those who depend on the protected natural habitat (forest) for their livelihood.

81. In which of these places FREEP is conducting programs in Tamil Nadu?

a) Gulf of Mannar

b) Vedanthangal Birds Life Sanctuary

c) Kalakad- Mundanthurai Tiger Reserve

d) Arignar Anna Zoological Park

Explanation

FREEP is conducting pilot eco development programs in the Kalakad-Mundanthurai Tiger Reserve (KMTR) in Tamil Nadu. The reserve contains a unique and varied array of flora ranging from thorn and dry teak to tropical evergreen, and supports a rich variety of birds and mammals, including tigers, leopards and elephants. The last tiger refuge in Tamil Nadu, the KMTR is one of 50 sites covered under the Indian Government’s Project Tiger, a programed receiving international assistance to enhance tiger habitat.

82. Which of the following facts is not related to the United Nations convention on Biological Diversity?

a) Sovereign right of nation over their genetic resources.

b) High resource countries decide the benefits of genetic resources.

c) Appropriate access to genetic resources.

d) Fair and equitable sharing of benefits from the use of genetic resources.

Explanation

The United Nations convention on Biological Diversity, known as CBD in short, was signed by India and 172 other nations on December 29, 1993. The CBD was an attempt to establish an international program for conserving and using the world’s biological resources. This historical treaty recognizes the “sovereign right of nation over their genetic resources” and considers “appropriate access to genetic resources.” The treaty also takes into account the “fair and equitable sharing” of benefits arising from the use of genetic resources.

83. Where did the convention of Earth Summit held in the year 1992?

a) Geneva

b) Switzerland

c) Brazil

d) Paris

Explanation

The Convention on Biological Diversity (CBD) is a United Nations initiative to protect Biodiversity and encourage the sustainable use of natural resources. The convention was held in 1992 at the ‘Earth Summit’ in Brazil. India is a signatory of the CBD.

84. Which of these acts preserve the biological diversity in India?

a) Earth Summit, 1992

b) Wild Life Protection Act, 1942

c) The Biological Diversity Act, 2002

d) None of the above

Explanation

The Biological Diversity Act, 2002 is an Act of the Parliament of India for preservation of biological diversity in India, and provides mechanism for equitable sharing of benefits arising out of the use of traditional biological resources and knowledge. The Act was enacted to meet the obligations under Convention on Biological Diversity (CBD), to which India is a party.

85. In which year the National Biodiversity Authority was established?

a) 2003

b) 2000

c) 1998

d) 2007

Explanation

The National Biodiversity Authority (NBA) was established by the Central Government in 2003 to implement India’s Biological Diversity Act (2002). The NBA is a Statutory Body and it performs facilitative, regulatory and advisory functions for the Government of India on issues of conservation, sustainable use of biological resources and fair and equitable sharing of benefits arising out of the use of biological resources.

86. Which of these are included in the biodiversity?

a) Diversity within species

b) Ecosystem of a region

c) Diversity between species

d) All the above

Explanation

Biodiversity as the variability among living organisms from all sources, including terrestrial, marine and other aquatic ecosystems and the ecological complexes of which they are a part. This includes diversity within species, between species and ecosystems of a region.

87. Which of these provides an index of health of an ecosystem?

a) Biodiversity

b) Waste management

c) Recycling process

d) Pollution control

Explanation

Biodiversity helps to maintain ecological processes, create soil, recycle nutrients, influence climate, degrade waste and control diseases. It provides an index of health of an ecosystem.

88. Which of these changes reflects in the pattern of biodiversity of our planet?

a) Atmospheric pressure

b) Climatic conditions

c) UV radiation

d) Pollution

Explanation

Biodiversity is the assemblage of different life forms. Each species is adapted to live in its specific environments. The changes in climatic conditions are reflected in the distribution and pattern of bio diversity on our planet.

89. Which of these techniques used to measure genetic diversity?

a) Mutation techniques

b) Bio-chemical techniques

c) Molecular techniques

d) All the above

Explanation

Genetic diversity can be measured using a variety of molecular techniques. Genetic diversity helps in developing adaptations to changing environmental conditions.

90. Match

  1. Gamma diversity i) Between two adjacent ecosystems
  2. Alpha diversity ii) Total geographic area
  3. Beta diversity iii) Within a ecosystem

a) ii, iii, i

b) i, ii, iii

c) ii, i, iii

d) iii, ii, i

Explanation

Alpha diversity: It is measured by counting the number of taxa (usually species) within a particular area, community or ecosystem.

Beta diversity: It is species diversity between two adjacent ecosystems and is obtaining by comparing the number of species unique to each of the ecosystem.

Gamma diversity refers to the diversity of the habitats over the total landscape or geographical area.

91. Which of the following is not included in the ecosystem of India?

a) Rain forests

b) Temporary glaciers

c) Coral reefs

d) Deserts

Explanation

India with its alpine meadows, rain forests, mangroves, coral reefs, grass lands and deserts has one of the greatest ecosystem diversity on earth.

92. How many bio geographic zones are available in India?

a) 17

b) 7

c) 10

d) 50

Explanation

India is one of the 17 mega biodiversity countries of the world and has ten bio geographic zones with characteristic habitat and biota.

93. Which of these pattern increases diversity from poles to equator?

a) Longitudinal gradient

b) Equator variations

c) Latitudinal gradient

d) All the above

Explanation

The most important pattern of biodiversity is latitudinal gradient in diversity. This means that there is an increasing diversity from the poles to equator.

94. In which of these temperature ranges the metabolic efficiency is ease for living organisms?

a) 25⁰C to 35⁰C

b) 5⁰C to 15⁰C

c) 75⁰C to 85⁰C

d) 25⁰C to 50⁰C

Explanation

The temperatures vary between 25⁰C to 35⁰C, a range in which most metabolic activities of living organisms occur with ease and efficiency.

95. Which of these changes the variety and number of species in a ecosystem?

a) Seasons

b) Temperature

c) Photoperiods

d) All the above

Explanation

Climate, seasons, temperature, humidity, photoperiods are more or less stable and encourage both variety and numbers.

96. Who explored the wilderness of South American jungles?

a) Alexander von Humboldt

b) John Muir

c) James Audubon

d) Freeman Tilden

Explanation

German Naturalist and Geographer Alexander von Humboldt explored the wilderness of south American jungles and found that within a region the species richness increased with increasing area but up to a certain limit. The relationship between species richness and area for a wide variety of taxa (angiosperm plants, birds, bats, freshwater fishes) turned out to be the rectangular hyperbola.

97. Which of these cycles are not included in the major nutrient cycles?

a) N2

b) H2O

c) Br

d) S

Explanation

The major functional attributes are: continuity of nutrient cycles or Bio geochemical cycles (N2, C, H2O, P, S cycles)

98. Which of the following is used to degrade the molecules of pollutants?

a) Bacteria

b) Microbes

c) Protozoa

d) Algae

Explanation

Cleaning up of pollutants – microbes are the biggest degraders of molecules including many anthropogenic ones which are present in effluents, sewage, garbage and agro-chemicals.

99. In which of this direction the Himalayas mountain chains are located in India?

a) West to North eastern

b) West to East

c) North east to West

d) North to South eastern

Explanation

Himalayas : The entire mountain chain running from western to north eastern India 7.2% Alpine forest, orchids, rhododendrons, wild sheep, mountain goats, shrew, snow leopard, panda etc.,

100. Which of this region is not included in gangetic plains of India?

a) Bihar

b) Gujarat

c) West Bengal

d) Uttar Pradesh

Explanation

Gangetic plains One of most fertile region of India which extends from eastern Rajasthan through Uttar Pradesh, Bihar and West Bengal 11% Mangrove forest, dry deciduous forest, rhinoceros, elephant, buffalo, tiger, crocodile, swamp deer, hog deer etc.,

101. Which of this region is the refuge for the Nilgiri Tahr?

a) Western Ghats

b) Deccan plateau

c) Eastern Ghats

d) Servarayan Hills

Explanation

Western Ghats Mountain ranges from Sat Pena in South Gujarat to thesouthern most tip of Kerala. 4% Evergreen to dry deciduous forests, Nilgiri langur, Indian elephant. Nilgiri tahr (state animal of Tamilnadu), the grizzled squirrel and lion tailed macaque (endemic)

102. Which of these is called as the Biogeographical gateway of India?

a) North eastern India

b) Western Ghats

c) Andaman Nicobar Islands

d) Gangetic plains

Explanation

North – East India: The plains and non Himalayan hill ranges of north eastern India. This region represents the transition zone between the Indian, Indo – Malayan and Indo – Chinese bio geographical regions. It is the meeting point of the Himalayan mountains and peninsular India. 5.2% Biogeographical Gateway for much of India’s fauna and flora and also biodiversity hot spot.

103. What percentage of evergreen forest is available in Andaman and Nicobar Islands?

a) 2%

b) 15%

c) 0.3%

d) 35%

Explanation

Andaman and Nicobar Islands in the Bay of Bengal has 0.3% Evergreen forest, a wide diversity of corals, Narcondam hornbills, giant robber crab, turtle, wild boar, water monitor, south Andaman krait etc.

104. Match

A. Natural disasters i) Loss of native species

B. Resource exploitation ii) Pesticides

C. Intensive agriculture iii) Volcanoes

D. Pollutants iv) Over fishing

a) iii, iv, i, ii

b) ii, i, iv, iii

c) iv, ii, iii, i

d) i, iii, iv, ii

Explanation

Pollution and pollutants (smog, pesticides, herbicides, oil slicks, GHGs), Climate change, Introduction of alien/exotic species Over exploitation of resources (poaching, indiscriminate cutting of trees, over fishing, hunting, mining), Intensive agriculture and aquaculture practices, Hybridization between native and nonnative species and loss of native species, Natural disasters (Tsunami, forest fire, earth quake, volcanoes) Industrialization, Urbanization, infrastructure development, Transport –Road and Shipping activity, communication towers, dam construction, unregulated tourism and monoculture are common area of specific threats, Co-extinction

105. Which of the following is called as the Lungs of the planet?

a) Western Ghats of India

b) Nile settlements

c) Amazon rainforest

d) Alp’s mountains

Explanation

The most dramatic example of habitat loss comes from the tropical rainforests 14% of the earth’s land surface once covered by these tropical forests, is not more than 6% now. The Amazon rainforest, a vast area, harboring millions of species, also called “Lungs of the planet” is destroyed and being replaced for agriculture and human settlements.

106. In which of these states Tilapia fish was introduced from South Africa?

a) Kerala

b) Gujarat

c) Odisha

d) Tamil Nadu

Explanation

Tilapia fish (Jilabi kendai) (Oreochromismosambicus) introduced from east coast of South Africa in 1952 for its high productivity into Kerala’s inland waters, became invasive, due to which the native species such asPuntiusdubiusand Labeokontiusface local extinction.

107. In which of the following states the native papaya was destroyed by Papaya mealy bug from Central America?

a) Assam

b) Tamil Nadu

c) West Bengal

d) All the above

Explanation

Papaya Mealy Bug (Paracoccus marginatus) is native of Mexico and Central America, is believed to have destroyed huge crops of papaya in Assam, West Bengal and Tamil Nadu.

108. What are the adverse effects of climate change in the existing biodiversity?

a) Raised sea level

b) Migration of animals

c) Extreme weather conditions

d) All the above

Explanation

Climate change increases land and ocean temperature, changes precipitation patterns and raises the sea level. This in turn results in melting of glaciers, water inundation, less predictability of weather patterns, extreme weather conditions, outbreak of squalor diseases, and migration of animals and loss of trees in forest. Thus climate change is an imminent danger to the existing biodiversity.

109. In which of this region the shifting cultivation method is followed in India?

a) Gangetic plains

b) North- eastern

c) Deccan region

d) Sunder ban lands

Explanation

In shifting cultivation, plots of natural tree vegetation are burnt away and the cleared patches are farmed for 2-3 seasons, after which their fertility reduces to a point where crop production is no longer profitable. The farmer then abandons this patch and cuts down a new patch of forest trees elsewhere for crop production. This system is practiced in north-eastern regions of India.

110. What is the negative impact of the Intensive agriculture?

a) Increased in pathogens and pest attack

b) Sudden loss of related species

c) Reduction in the genetic diversity

d) Changes in soil structure and characteristics.

Explanation

Intensive agriculture is based on a few high yielding varieties. As a result, there is reduction in the genetic diversity. It also increases vulnerability of the crop plants to sudden attack by pathogens and pests. There are only few varieties of traditional paddy strains today due to use to hybrid varieties in Tamil Nadu.

111. What are the main criteria of a hotspot?

a) Increased number of species and variety.

b) More than 70% of original vegetation is lost.

c) Supports more number of predators.

d) Increase forest area and species.

Explanation

A hotspot is a region that supports at least 1500 endemic vascular plant species (0.5% of the global total) has lost more than 70% of its original vegetation. There are 35 biodiversity hotspots in the world.

112. How many biodiversity hotspots are identified in India?

a) 35

b) 4

c) 17

d) 50

Explanation

India is home to four biodiversity hotspots (as per ENVIS). They are Himalaya (the entire Indian Himalayan region), Western Ghats Indo-Burma: includes entire North-eastern India, except Assam and Andaman group of Islands (and Myanmar, Thailand, Vietnam, Laos, Cambodia and Southern China

113. In which of these National Park project Tiger was launched in the year 1973?

a) Jim Corbett National Park, Uttarakhand

b) Kanha national Park, Madhya Pradesh

c) Bandhavgarh National park, Madhya Pradesh

d) Kaziranga national Park, Assam

Explanation

Project Tiger was launched in the Jim Corbett National Park, Uttarakhand in 1973. The project ensures a viable population of Bengal tigers in their natural habitats, protecting them from extinction and preserving areas of biological importance as a natural heritage.

114. What is the serious effect of loss of biodiversity?

a) Extinction of species

b) Weather condition changes

c) Information in DNA and niche is lost forever

d) Both a and c

Explanation

The most serious aspect of the loss of biodiversity is the extinction of species. The unique information contained in its genetic material (DNA) and the niche it possesses are lost forever.

115. What are the missions of IUCN?

a) To influence, encourage and assist societies throughout the world to conserve nature

b) To ensure any use of natural resources is equitable and ecologically sustainable.

c) Collects, compiles and publishes the red list of threatened species and their conservation status in the world.

d) All the above

Explanation

IUCN’s mission is to influence, encourage and assist societies throughout the world to conserve nature and to ensure that any use of natural resources is equitable and ecologically sustainable. It influences governments and industries through partnerships by providing information and advice. The organization collects, compiles and publishes the IUCN red list of threatened species and their conservation status in the world.

116. How many aspects of conservation strategies are categorized?

a) 2

b) 4

c) 3

d) 5

Explanation

There are two aspects of conservation strategies, i) In-situ conservation ii) Ex-situ conservation

117. How many national Parks are situated in India?

a) 104

b) 18

c) 300

d) 54

Explanation

World Conservation monitoring centre has recognized 37,000 protected areas world-wide. India has about 771 protected areas covering 162099 km2 comprising of National Parks (104), Wild Life Sanctuaries (544), biosphere reserves (18) and several sacred groves.

118. Which of the authorized body notifies the National Parks in India?

a) State government

b) Stand alone comittee

c) The President

d) The Parliament

Explanation

National Parks (NP) : It is a natural habitat that is notified by the state government to be constituted as a National Park due to its ecological, faunal, floral, geomorphological, or zoological association of importance. No human activity is permitted inside the national park except the activities permitted by the Chief Wildlife Warden of the state.

119. Which of these National Park in India is a protected area for the one horned Rhinoceros?

a) Kaziranga National park

b) Gir National Park

c) Namdapha National Park

d) Pench National Park

Explanation

Kaziranga National park is a protected area for the one Horned Rhinoceros in Assam.

120. Which of these activities are not allowed in a wild life sanctuary?

a) Private ownership of land

b) Hunting or poaching

c) Collection of forest products

d) Harvesting of timber

Explanation

Sanctuaries are tracts of land where wild animals and fauna can take refuge without being hunted or poached. Other activities like collection of forest products, regulated harvesting of timber, private ownership of land are permitted.

121. Which of the following are the Bio reserves in Tamil Nadu?

a) Agasthyamalai

b) Gulf of Mannar

c) Nilgiri

d) All the above

Explanation

Agasthyamalai (Karnataka – Tamil Nadu – Kerala), Nilgiri (Tamil Nadu – Kerala), Gulf of

Mannar (Tamil Nadu) are the BRs notified in Tamil Nadu

122. Which is the Head Quarters of the World Wild Fund for Nature?

a) France

b) Switzerland

c) California

d) Los Angeles

Explanation

World Wild Fund for Nature (WWF) is an international non-governmental charitable trust founded in 1961, with headquarters at Gland, Vaud, Switzerland.

123. In which year the CITES resolution came into force?

a) 1975

b) 1903

c) 1949

d) 1999

Explanation

The Convention on International Trade in Endangered Species (CITES) of wild fauna and flora, also known as the Washington Convention, is a multilateral treaty to protect endangered plants and animals. It was drafted from a resolution adopted from a meeting of members of the IUCN in 1963 and opened for signature in 1973. It came into force during July 1975.

124.How many nations signed the United Nations convention on Biological Diversity?

a) 50

b) 23

c) 173

d) 101

Explanation

The United Nations convention on Biological Diversity, known as CBD in short, was signed by India and 172 other nations on December 29, 1993.

125. What is the Head Quarters on the NBA in India?

a) Chennai

b) Mysore

c) New Delhi

d) Bangalore

Explanation

The NBA is a Statutory Body and it performs facilitative, regulatory and advisory functions for the Government of India on issues of conservation, sustainable use of biological resources and fair and equitable sharing of benefits arising out of the use of biological resources. The Headquarters of the NBA is situated in Chennai

12th Science Lesson 16 Questions in English

16] Environmental issues

1. Assertion (A): Undesired human activities change the physical, chemical and biological characteristics of the environment.

Reasoning(R): Agents causing undesirable change in the environment are called pollutants.

a) Both A and R is True and R is the correct explanation of A.

b) Both A and R is True but R is not the correct explanation of A.

c) A is True but R is False.

d) Both A and R is False.

Explanation

Pollution is any undesirable change in the physical, chemical and biological characteristics of the environment due to natural causes and human activities. The agents which cause pollution are called pollutants.

2. Based on which factor pollution is classified?

a) Origin of pollution

b) Materials

c) Types of environment

d) Grade of pollution

Explanation

Pollution is classified according to the types of environment that is affected. They are mainly air, water and soil pollution.

3. Choose the correct statements.

i) Non-degradable and Degradable pollutants are classified in terms of eco-system.

ii) Degradable pollutants are classified based on the materials causing pollution.

a) i only

b) ii only

c) Both i and ii

d) Neither i nor ii

Explanation

In terms of eco-system, pollutants can be classified into two basic groups – Non-degradable and degradable. Based on the time taken to breakdown into their ingredients, degradable pollutants are classified as rapidly degradable (non-persistent) and slowly degradable (persistent).

4. Which of these pollutants are broken by a natural process?

a) Non-degradable pollutant

b) Non-persistent pollutant

c) Slowly degradable pollutant

d) Industrial pollutant

Explanation

Rapidly degradable or non-persistent pollutants: These can be broken down by natural processes. Domestic sewage and vegetable waste are examples of such pollutants.

5. Which of the following is a persistent pollutant?

a) DDT

b) Vegetable waste

c) Nuclear wastes

d) None of the above

Explanation

Slowly degradable or persistent pollutants: These are pollutants that remain in the environment for many years in an unchanged condition and take decades or longer to degrade, as in the case of DDT.

6. State some of the common toxic pollutants.

a) Lead

b) Chromium

c) Cadmium

d) All the above

Explanation

Non-degradable pollutants: These cannot be degraded by natural processes. Once they are released into the environment, they are difficult to be eliminated and continue to accumulate (bio magnification). Toxic elements like lead, mercury, cadmium, chromium and nickel are such common pollutants.

7. What are the functions of atmosphere in Earth?

a) Thermal insulator

b) Temperature regulator

c) UV rays absorption

d) All the above

Explanation

Earth is surrounded by a gaseous envelope which is called atmosphere. The gaseous blanket of the atmosphere acts as a thermal insulator and regulates the temperature of the earth by selectively absorbing the UV rays of solar radiation.

8. Which of these does not have adverse effect of pollution?

a) Domestic wastage

b) Chlorofluorocarbon

c) Industries

d) Global warming

Explanation

The adverse effects of pollution include depletion of Ozone by Chlorofluorocarbons or CFCs, used as refrigerants and global warming by elevated CO2 (industries, deforestation, and partial combustion).

9. Choose the correct statements about the Air pollution.

i) Air pollutants include only the abundant presence of solid and gaseous substances.

ii) The nature and concentration of a pollutant determines the severity of detrimental effects on organisms.

iii) Atmospheric factors prevailing at a particular place does not create a catastrophic effect.

a) i only

b) ii only

c) iii only

d) None of the above

Explanation

The alterations or changes in the composition of the earth’s atmosphere by natural or human activities (anthropogenic factors) are referred as Air Pollution. Pollutants include the abundant presence of solid, liquid or gaseous substances produced by human or natural activity. The nature and concentration of a pollutant determines the severity of detrimental effects on organisms and human health. Along with atmospheric factors (humidity, precipitation, wind, air currents, altitude) prevailing at a place and time, its effects can be far reaching and catastrophic.

10. Which of these are the air pollutants?

a) Discharge of dusts

b) Particulate matters

c) Gas discharges

d) All the above

Explanation

Air pollutants can be discharge of dusts or particulate matter (PM: 2.5-10 μm), discharge of gases (SO2, NO2, CO, CO2).

11. Which of the following does not produce the Carbon Monoxide?

a) Automobiles

b) Nuclear waste

c) Forest fire

d) Power plants emission

Explanation

Carbon monoxide (CO) is produced mainly due to incomplete combustion of fossil fuels. Automobiles are major causes of CO pollution in large cities and towns’ Automobile exhausts, fumes from factories, emission from power plants, forest fires and burning of fire-wood contribute to CO pollution.

12. Which of these leads to main pollutant for global warming?

a) CO2

b) N2O

c) H2SO4

d) N2

Explanation

With rapid urbanization, major amount of carbon dioxide and sulphur dioxide (SO2) is released in the atmosphere. From automobiles, aero planes, power plants and other human activities that involving the burning of fossil fuels (coal, oil etc.,) CO2 is the main pollutant that is leading to global warming.

13. Which of these are the major causes of acid rain?

a) Sulphur dioxide and Nitrogen oxides

b) Carbon Monoxide and Ammonia

c) Nitrogen Sulphide and Hydrogen Peroxide

d) Chlorofluoro carbon

Explanation

Nitrogen oxides are also major air pollutants. Fossil fuel combustion and automobiles exhausts are the source of nitrogen oxides. Sulphur dioxide and nitrogen oxides are the major causes of acid rain.

14. Which of these are the main sources of particulate matter pollution?

a) Cement factories

b) Forest fire

c) Thermal power plants

d) All the above

Explanation

Particulate matters are tiny particles of solid matter suspended in a gas or liquid. Combustion of fossil fuels, fly ash produced in thermal power plants, forest fires, asbestos mining units, cement factories are the main sources of particulate matter pollution.

15. Identify the incorrect match

A. Area Sources i) Agriculture

B. Stationary Sources ii) Power Plants

C. Transport Sources iii) Trucks and Trains

D. Natural Sources iv) Fireplaces

a) i only

b) ii only

c) iii only

d) iv only

Explanation

The main sources of air pollution are: Transport sources – cars, buses, airplanes, trucks, trains

Stationary sources – power plants, incinerators, oil refineries, industrial facilities, and factories

Area sources – agricultural – wood / stubble burning, fireplaces

Natural sources – wind-blown dust, wildfires, volcanoes.

16. Which of the following are the effects of Air pollution?

i) Contaminated air reduces the body’s defense mechanism.

ii) Polluted air causes breathing problems and aggravates existing health conditions.

iii) Frequent exposure to polluted air increases the risk of cardiovascular diseases.

a) i only

b) ii only

c) iii only

d) All the above

Explanation

Effects of Air Pollution: Affects all organisms as they depend on the atmosphere for respiration. Causes irritation in the throat, nose, lungs and eyes. It causes breathing problems and aggravates existing health conditions such as emphysema and asthma. Contaminated air reduces the body’s defense mechanism and decreases the body’s capacity to fight other infections in the respiratory system. Frequent exposure to polluted air increases the risk of cardiovascular diseases. Breathing air that is filled with fine particulate matter can induce hardening of the arteries, triggering cardiac arrhythmia or even a heart attack. People who exercise outdoors can sometimes be susceptible to adverse effects of air pollution because it involves deeper and faster breathing. Hence it is advisable to walk or jog in the mornings in places with ample tree cover.

17. Assertion (A): Low concentration carbon monoxide causes headache and high concentration leads to coma and death in humans.

Reasoning(R): The atmosphere CO interacts with the oxygen as hemoglobin has greater affinity for carbon monoxide.

a) Both A and R is True and R is the correct explanation of A.

b) Both A and R is True but R is not the correct explanation of A.

c) A is True but R is False.

d) Both A and R is False.

Explanation

Gas leaks can be lethal or affect the quality of air in the affected area. CO in the atmosphere interferes with O2 transport since hemoglobin has greater affinity for carbon monoxide. At low concentration it causes headache and blurred vision. In higher concentration, it can lead to coma and death.

18. Which of the following statements are not true about the smog?

a) Smog is a type of air pollution caused by tiny particles in the air.

b) Smog is derived from the words small and fog.

c) Photochemical smog is created by the reaction of sunlight with nitrogen oxides.

d) The smog reactions create ground level ozone and reduces visibility.

Explanation

Smog is a type of air pollution caused by tiny particles in the air. The word comes from a mixture of the words smoke and fog. Today, smog generally refers to photochemical smog, which is created when sunlight reacts with nitrogen oxides and volatile organic compounds found in fossil fuel emissions from automobiles, factories, and power plants. These reactions create ground-level ozone and particulate matter, reducing visibility.

19. Which of the following are the impacts of the smog?

a) Breathing difficulties

b) Damages crops

c) Corrosive damage in buildings

d) All the above

Explanation

Smog can make breathing more difficult, especially for people with asthma. Smog also affects plants and animals. It damages crops as well as causes health problems in pets, farm animals and human beings. Smog has also been known to cause corrosive damage to buildings and vehicles.

20. Name the secondary pollutant in the photochemical smog?

a) Ozone

b) Peroxy acetyl nitrate

c) Chlorofluorocarbon

d) Carbon dioxide

Explanation

Peroxy acetyl nitrate (PAN) is a secondary pollutant present in photochemical smog. It is thermally unstable and decomposes into peroxy ethanol radicals and nitrogen dioxide gas causing eye irritation.

21. Which of the following is not classified as a greenhouse gas?

a) Nitrous oxide

b) Helium

c) Methane

d) Carbon dioxide

Explanation

Global warming: Increase in the concentrations of greenhouse gases such as CO2, methane, nitrous oxide, CFCs, and ozone causes greenhouse effect, warming of the earth resulting in sea level rise, submerging of islands and sea shores of various parts of the world.

22. Assertion (A): Ozone depletion is the thinning of the stratospheric ozone layer.

Reasoning(R): Ozone hole results in poor screening of harmful UV rays.

a) Both A and R is True and R is the correct explanation of A.

b) Both A and R is True but R is not the correct explanation of A.

c) A is True but R is False.

d) Both A and R is False.

Explanation

Ozone depletion: Thinning of the stratospheric ozone layer is known as ozone depletion. Such depletion causes the ‘ozone hole’, resulting in poor screening of the harmful UV rays and increase in incidences of skin cancer. Some of the common agents that deplete ozone are CFCs.

23. Choose the Incorrect statements about Acid rain.

i) Acid rain is a form of precipitation with acidic components.

ii) Hydrochloric acid is the main cause of the acid rain.

iii) Trees, crops and marine animals are damaged by the acid rain.

a) i only

b) ii only

c) iii only

d) All the above

Explanation

Acid rain: Acid rain is a form of precipitation that contains acidic components, such as sulphuric acid or nitric acid. It damages trees, crops and harms marine animals (coral reefs) and induces corrosion.

24. Which of the following are used to reduce or remove the pollutants?

a) Forests

b) Catalytic converters

c) Electrostatic precipitators

d) All the above

Explanation

Certain measures help to remove pollutants, reduce their presence or prevent their entry into the atmosphere. Trees are the best remedy for urban particulate and gaseous pollution. Forests act as carbon sinks and lungs of the planet. Catalytic converters in vehicles help to reduce polluting gases drastically. Diesel exhaust filters in automobiles cuts particulates. Electrostatic precipitators reduce release of industrial pollutants. Cost effective air pollution treatment systems like indoor plants and high performance bio filters can improve indoor air quality.

25. Which of the act was enacted in 1981 to prevent and control air pollution in India?

a) The Air (Prevention and Control of Pollution) Act

b) The Nature preservation Act

c) The Ecological Act

d) The Pollution Control Act

Explanation

The Air (Prevention and Control of Pollution) Act was enacted in 1981 and amended in 1987 for the prevention, control and abatement of Air pollution in India.

26. Which of this organization is responsible for the environmental protection in India?

a) The Green Bench and the National Green Tribunal

b) The High Court

c) The Pollution Control Board

d) All the above

Explanation

Traffic Emissions Standards: The Government has decided to enforce Bharat Stage VI norms from 2020. The Green Bench and the National Green Tribunal (NGT) give judicial safeguard to environmental protection.

27. What are the steps taken by the Central and State governments of India for environmental protection?

a) Promote Swachh Bharat Abhiyan

b) Enact and enforce strict environmental laws.

c) Encourage usage of renewable sources.

d) All the above

Explanation

Steps taken by the Central and the State governments in India: Road traffic rationing, encourage public transport, carpooling. Increase green cover alongside roads (planting avenue trees). To Promote Swachh Bharat Abhiyan. To Enact and Enforce of stricter environmental laws. To maintain air standards by proper enforcement and monitoring. To reduce carbon emissions. Encourage use of renewable energy. Limiting the sale of firecrackers and developing eco-friendly crackers. Make Environmental Impact Assessment mandatory

28. Which of these used to measure the polluted air at a given time?

a) Air Quality Index

b) Pollution Level

c) Air Mass

d) Level of Pollution

Explanation

Air Quality Index (AQI) is a number used by government agencies to communicate to the public how polluted the air is at a given time.

29. Assertion (A): Water is essential for life and for the health of the environment.

Reasoning(R): Water is a natural resource comprising of marine, estuarine, fresh water and ground water environments.

a) Both A and R is True and R is the correct explanation of A.

b) Both A and R is True but R is not the correct explanation of A.

c) A is True but R is False.

d) Both A and R is False

Explanation

Quality of Water: Water is essential for life and for the health of the environment. As a valuable natural resource, it comprises marine, estuarine, freshwater (river and lakes) and groundwater environments that stretch across coastal and inland areas.

30. Which of these factors are considered for the quality of the water?

a) Physical

b) Aesthetic

c) Biological

d) All the above

Explanation

Water quality is commonly defined by its physical, chemical, biological and aesthetic (appearance and smell) characteristics. A healthy environment is one in which the water quality supports a rich and varied community of organisms and protects public health.

31. Choose the Incorrect statements.

i) Water pollution occurs when there is a change in the chemical, physical or biological quality of water.

ii) Water pollution does not affect the larger water bodies like Ocean and rivers.

iii) If water becomes unfit for its intended use it is considered as polluted.

a) i only

b) ii only

c) iii only

d) None of the above

Explanation

Water pollution occurs when there is a change in the chemical, physical or biological quality of water that has harmful effect(s) on living organisms that consume it or live in it. Water pollution adversely affects water bodies due to the large amounts of natural or man-made materials let into it. When it becomes unfit for its intended use, water is considered polluted.

32. How many main sources of water pollution are categorized?

a) 7

b) 3

c) 5

d) 4

Explanation

Sources of Water Pollution: Even though water bodies or sources can be polluted by natural causes, water pollution is usually caused by human activities. There are three main types of sources: point sources, non-point sources, leaks and spills.

33. Which of the following is not classified as a point source for water pollution?

a) Underground mines

b) Acid rain

c) Agriculture

d) Factory effluents

Explanation

Point sources: Discharge of pollutants at specific locations through pipelines or sewers into the water body. Factory effluents, sewage, underground mines, oil wells, oil tankers and agriculture are common point sources.

34. Define the non-point sources of water pollution.

a) Site of discharge cannot be traced.

b) Domestic sewages through pipelines.

c) Discharge at specific common locations.

d) Due to natural calamities.

Explanation

Non-point sources: Sources that cannot be traced to a single site of discharge like acid rain, dumping of the plastics in water bodies, agriculture chemical run off are common examples

35. Which of these can cause leaks and spills of polluted water?

a) Oil leakages

b) Ship collision

c) Off shore oil rigs

d) All the above

Explanation

Leaks and Spills: This occurs mostly due to ship collision, off shore oil rigs, oil leakages and discharges into sea. Sources of water pollution can also be classified in three ways. They are municipal wastes, industrial wastes, and agricultural wastes.

36. Assertion (A): The industrial discharges can affect temperatures of the water bodies as well as dissolved oxygen level.

Reasoning(R): Industrial discharge may contain varieties of compounds such as heavy metals, organic / inorganic chemicals and toxic concentrations.

a) Both A and R is True and R is the correct explanation of A.

b) Both A and R is True but R is not the correct explanation of A.

c) A is True but R is False.

d) Both A and R is False.

Explanation

Municipal waste water is from homes and commercial establishments. Industrial discharge (effluents) may contain varieties of compounds such as heavy metals (cadmium, chromium, lead), and organic / inorganic chemicals containing waste water, sometimes in toxic concentrations. These discharges can affect temperatures of the water bodies as well as dissolved oxygen level.

37. Choose the correct statements.

i) Ecosystems is affected or destroyed by water pollution by affecting the soil fertility.

ii) Water pollution disrupts the natural food chains as well as the food webs.

iii) Hot water from industries affects the aquatic density and diversity in the water bodies.

a) i only

b) ii only

c) iii only

d) All the above

Explanation

Destruction of ecosystems: Ecosystems especially aquatic systems can be severely affected or destroyed by water pollution. Water pollutants affect existing niches and habitats and the survival of organisms. Soil fertility is affected and the system becomes uninhabitable.

Disruption of food-chains: Water pollution disrupts the natural food chains as well as food webs. Pollutants such as lead and cadmium are taken up by primary consumers where they can be lethal or get stored. Later, when these animals are consumed by secondary consumers, the food chain can get disrupted at any trophic level or result in enhanced concentration of these pollutants (bio magnification). Hot water from industries when released into the water bodies affects aquatic density and diversity.

38. Assertion (A): Oil spread on the water surface prevents the entry of sunlight and oxygen.

Reasoning(R): Oil leakages and spills decreases the BOD and COD resulting in mass death of organisms and degrade the water quality.

a) Both A and R is True and R is the correct explanation of A.

b) Both A and R is True but R is not the correct explanation of A.

c) A is True but R is False.

d) Both A and R is False.

Explanation

Water pollution can be lethal to aquatic organisms and others that depend on these water bodies. Accidental oil spills from tanker ships can cause substantial environmental damage. Oil spreads on the water surface, prevents the entry of light and oxygen into the water. This increases BOD and COD, resulting in mass death of organisms and degradation of water quality. It also clogs fish gills and the feathers of aquatic birds.

39. Which of the following diseases are caused by the contaminated water and food?

a) Hepatitis

b) Typhoid

c) Fluorosis

d) All the above

Explanation

Humans and other organisms can get affected by diseases such as hepatitis and typhoid by consuming contaminated water and food. Excess of fluoride in drinking water causes fluorosis. In many poor nations, outbreak of water borne diseases and epidemics are a result of contaminated water and poor or absence of water treatment processes.

40. Which of these can cause eutrophication in the water bodies?

a) Red tides

b) Nutrient enrichment

c) Oil spills

d) Increased temperature

Explanation

Water pollution can cause eutrophication due to nutrient enrichment. This causes algal blooms which affect the quality of water bodies. Red tides, if occur, can be lethal to aquatic organisms prohibit the pollution of a stream or well by disposal of polluting matter.

41. What is the role of the Ministry of Environment, Forest and Climate change in India?

a) Planning environmental policies and programmes

b) Promoting forestry policies

c) Overseeing the implementation of India’s environmental policies

d) All the above

Explanation

The Ministry of Environment, Forest and Climate Change (MoEFCC) is the nodal agency of the Central Government for the planning, promotion, co-ordination and for overseeing the implementation of India’s environmental and forestry policies and programmes.

42. In which year the National river conservation plan was enacted in India?

a) 1967

b) 1982

c) 1995

d) 1991

Explanation

National river conservation plan (NRCP) was enacted in 1995 to improve the water quality of the rivers, which are the major fresh water resources in our country. This important assignment taken up under the NRCP includes, To capture the raw sewage flowing into the river through open drains and divert them for treatment.

43. Which of the following are used to prevent the water pollution?

a) Sewage treatment plans and Effluent treatment plans

b) Pretreating waste water by scientific methods

c) Restrict the usage of synthetic fertilizers and pesticides.

d) All the above

Explanation

Prevention

• Regulate or control of pollutant(s) discharge at the point of generation.

• Wastewater can be pretreated by scientific methods before discharge to municipal treatment sources.

• Setting up of Sewage Treatment Plants (STP) and Effluent Treatment Plants (ETP).

• Regulate or restrict the use of synthetic fertilizers and pesticides.

• Public awareness and peoples’ involvement is essential.

44. Assertion (A): Namami Gange is an integrated Conservation Mission approved by the Union Government in the year 2014.

Reasoning(R): National Mission for Clean Ganga was planned for the effective abatement of pollution, conservation and rejuvenation of River Ganga

a) Both A and R is True and R is the correct explanation of A.

b) Both A and R is True but R is not the correct explanation of A.

c) A is True but R is False.

d) Both A and R is False.

Explanation

Namami Gange (National Mission for Clean Ganga) Programme is an Integrated Conservation Mission approved as the ‘Flagship Programme’ of the Union Government in June 2014 with a budget outlay of 20,000 crore to accomplish the twin objectives of effective abatement of pollution, conservation and rejuvenation of River Ganga.

45. Choose the correct statements.

i) Noise pollution is unwanted and undesirable noise which may disrupt one’s quality of life.

ii) The intensity of noise pollution is measured in decibels (dB).

a) i only

b) ii only

c) Both i and ii

d) Neither i nor ii

Explanation

Sound that is unwanted and undesirable or can disrupt one’s quality of life is called as Noise. When there is lot of ‘noise’ in the environment, it is termed as Noise Pollution. The intensity of noise is measured in decibels (dB).

46. What is the threshold pain level due to noise in humans?

a) 20dB

b) 120dB

c) 280dB

d) 500dB

Explanation

The threshold of pain is about 120 db. World Health Organization has proposed that noise must be recognized as a major threat to human well-being. This is applicable for all living organisms.

47. Which of the following is not an effect of Noise pollution?

a) Sleep disruption

b) Lungs infection

c) Hearing loss

d) Productivity loss

Explanation

According to the USEPA (United States Environmental Protection Agency) there are direct links between noise and health. Heart disease, high blood pressure, stress related illness, sleep disruption, hearing loss (deafness), and productivity loss are the problems related to noise pollution. Increased stress and tension, nervousness, irritability, anxiety, depression and panic attacks. Peptic ulcer, severe head ache, memory loss. Marine animals are affected by noise pollution from offshore activities and port activities. Fire crackers frighten animals. Birds are often affected by increased air traffic.

48. What are the control measures for the noise pollution?

a) Servicing and tuning of automobile engines.

b) Machinery lubrication and regular servicing to reduce noise levels.

c) Strict usage of loudspeakers in public places.

d) All the above

Explanation

Control: Planting trees in and around noise sources is an effective solution for noise pollution as plants are known to absorb noise and bring down sound levels. Regular servicing and tuning of automobile engines can effectively reduce noise pollution by vehicles and machinery. Workers should be provided with ear plugs and earmuffs at work sites that generate high noise levels. Lubrication of machinery and regular servicing minimizes noise levels. Regulations should be imposed to restrict the usage of loudspeakers in crowded areas and public places.

49. In which year the Tamil Nadu government environment policy was enacted for noise pollution?

a) 2000

b) 2003

c) 2017

d) 2009

Explanation

Article 48-A and Article 51-A of the Constitution of India, Noise Pollution (Regulation and Control) Rules 2000, and Tamil Nadu State Environment Policy 2017 are some of the legal relief from noise pollution.

50. What is the permissible limit of noise in commercial areas during day time?

a) 65 decibels

b) 40 decibels

c) 55 decibels

d) 35 decibels

Explanation

According to Noise Pollution (Regulation and Control) Rules, 2000, the permissible limit of noise in areas categorized as commercial is 65 decibels (dB) during day and 55 dB during night.

51. Assertion (A): Agrochemicals and agrichemicals are used in agriculture for growth of plants and pest control.

Reasoning(R): Overuse of agrochemicals cause nutrient imbalance and kill beneficial bacteria’s and soil organisms

a) Both A and R is True and R is the correct explanation of A.

b) Both A and R is True but R is not the correct explanation of A.

c) A is True but R is False.

d) Both A and R is False.

Explanation

Chemicals which are used in agriculture for growth of plants and pest control are called agrochemicals or agrichemicals. Overuse of agrochemicals has been observed to generate residues that cause nutrient imbalance, and may kill beneficial bacteria and soil organisms.

52. Which of the following is a mosquito repellent?

a) DEET

b) DDT

c) N2

d) CO

Explanation

Mosquito Repellents DEET (n-n-diethylnetatoluamide) and allethrin used in mosquito coils may cause itching, burning, tingling sensation or numbness.

53. Choose the correct statements.

i) Producers and consumers from trophic levels in a food chain carry the energy flow.

ii) Usage, storage and transformation of food and biomolecules by metabolism are normal process.

iii) Degradation or breakdown is an essential part of any food chain.

a) i only

b) ii only

c) iii only

d) All the above

Explanation

Food chains are components of all ecosystems. Producers and consumers form trophic levels in a chain through which energy flow is carried out by the process of eating and being eaten. Usage, storage and transformation of food and biomolecules by metabolism are a normal process. Degradation or breakdown is an essential part of any food chain and hence all naturally occurring substances are degradable.

54. Assertion (A): Non-degradable substances in the food chain are transferred to the tropic levels without being metabolized and broken down.

Reasoning(R): Bio magnification results in increased toxicity and may even be lethal.

a) Both A and R is True and R is the correct explanation of A.

b) Both A and R is True but R is not the correct explanation of A.

c) A is True but R is False.

d) Both A and R is False.

Explanation

Bio magnification of DDT: When non-degradable substances enter the food chain, they do not get metabolized or broken down or expelled and instead get transferred up the tropic levels of the food chain. During this process, they show an increase in concentration which is referred to as bio magnification. This results in increased toxicity and may even be lethal. This phenomenon is well established for mercury and DDT.

55. Define Eutrophication.

a) Run-off from land containing nutrients reaches water bodies increase the dense growth of plant life.

b) An increase in the toxicity concentration is referred to as bio magnification.

c) Natural aging of lakes also leads to nutrient enrichment of its water.

d) Introduce nutrients such as nitrates and phosphates encourage the growth of aquatic organisms.

Explanation

When run-off from land containing nutrients reaches water bodies like lakes, it results in dense growth of plant life. This phenomenon is called Eutrophication. Natural aging of lakes also leads to nutrient enrichment of its water. In a lake, the water is cold and clear (oligotrophic stage), supporting little life. With time, streams draining into the lake introduce nutrients such as nitrates and phosphates, which encourage the growth of aquatic organisms. Aquatic plants and animal life grow rapidly, and organic remains begin to be deposited on the lake bottom (mesotrophic stage)

56. What is called as Cultural or accelerated eutrophication?

a) Waste water treatment process based on the level of pollution.

b) Pollutants from anthropogenic activities radically accelerate the aging process of water.

c) Stimulating the growth of algae and water hyacinth.

d) Deprival of the dissolved oxygen in the water.

Explanation

Pollutants from anthropogenic activities like effluents from the industries and homes can radically accelerate the aging process. This phenomenon is known as Cultural or Accelerated Eutrophication. Nutrients stimulate the growth of algae, water hyacinth and can cause clogging of canals, rivers and lakes as well as, displacing native plants. It causes unsightly foam and unpleasant odors, and deprives the water of dissolved oxygen.

57. In which year the Indian Government passed the water act?

a) 1974

b) 1982

c) 1963

d) 1988

Explanation

Wastewater or sewage originates from domestic waste waters, industrial wastes and animal wastes. Realizing the importance of clean potable water, the Government passed the Water (Prevention and Control of Pollution) Act in 1974, which made it mandatory to treat wastewater in treatment plants.

58. Assertion (A): Waste water treatment process reduces organic and inorganic components.

Reasoning (R): Microbial growth and potential toxic materials are removed from the waste water treatment process.

a) Both A and R is True and R is the correct explanation of A.

b) Both A and R is True but R is not the correct explanation of A.

c) A is True but R is False.

d) Both A and R is False.

Explanation

The main objective of a wastewater treatment process is to reduce organic and inorganic components in wastewater to a level that it no longer supports microbial growth and to eliminate other potentially toxic materials.

59. Which of this treatment removes the physical removal of solid and particulate materials?

a) Primary Treatment

b) Secondary Treatment

c) Biological Treatment

d) All the above

Explanation

Sewage treatment is usually performed in the following three stages. Primary treatment involves the physical removal of solid and particulate organic and inorganic materials from the sewage through filtration and sedimentation Floating debris is removed by sequential filtration.

60. Which of these contains small pebbles and soil?

a) Dust

b) Particulates

c) Grit

d) Distilled water

Explanation

The grit (soil and small pebbles) are removed by sedimentation. All solids that settle form the primary sludge and the supernatant forms the effluent. The effluent from the primary settling tank is taken for secondary treatment.

61. Choose the incorrect statements regarding the biological treatment of waste water.

i) The Primary effluent of waste water is passed into large aeration tanks and agitated mechanically.

ii) Useful aerobic microbes are allowed to grow as flocs which consists of bacteria masses.

iii) Microbes consume the minor part of inorganic matter in the effluent.

a) i only

b) ii only

c) iii only

d) None of the above

Explanation

Secondary treatment or biological treatment: The primary effluent is passed into large aeration tanks where it is constantly agitated mechanically and air is pumped into it. This allows vigorous growth of useful aerobic microbes into floc (masses of bacteria associated with fungal filaments to form mesh like structures). While growing, these microbes consume the major part of the organic matter in the effluent. This significantly reduces the BOD (Biochemical oxygen demand or Biological oxygen demand).

62. Assertion (A): BOD refers to the amount of oxygen consumed if all the organic matter in water were oxidized by bacteria.

Reasoning(R): Greater the BOD of the waste water the polluting potential is more.

a) Both A and R is True and R is the correct explanation of A.

b) Both A and R is True but R is not the correct explanation of A.

c) A is True but R is False.

d) Both A and R is False.

Explanation

BOD refers to the amount of the oxygen that would be consumed, if all the organic matter in one liter of water were oxidized by bacteria. The sewage water is treated till the BOD is reduced. The greater the BOD of the waste water more is its polluting potential. Once the BOD of sewage water is reduced significantly, the effluent is then passed into a settling tank where the bacterial “flocs” are allowed to sediment. This sediment is called activated sludge.

63. Which of the following is not associated with biogas?

a) Nitrogen

b) Carbon dioxide

c) Methane

d) Hydrogen Sulphide

Explanation

A small part of activated sludge is pumped back into the aeration tank to serve as the inoculum. The remaining major part of the sludge is pumped into large tanks called anaerobic sludge digesters. Here, the bacteria which grow anaerobically digest the bacteria and the fungi in the sludge. During this digestion, bacteria produce a mixture of gases such as methane, hydrogen sulphides and CO2. These gases form biogas and can be used as a source of energy.

64. What is the final treatment to improve the quality of waste water?

a) Distillation

b) Disinfectant usage

c) Tertiary treatment

d) Biological treatment

Explanation

Tertiary treatment is the final process that improves the quality of the waste water before it is reused, recycled or released into natural water bodies. This treatment removes the remaining inorganic compounds and substances, such as nitrogen and phosphorus.

65. Which of the following is an ideal disinfectant for the waste water treatment?

a) UV

b) Chlorine

c) Phosphorous

d) Nitrogen

Explanation

UV is an ideal disinfectant for wastewater since it does not alter the water quality – except for inactivating microorganisms.

66. Which of these statements are not true regarding Solid waste management?

a) Solid wastes come from home, offices, industries and agricultural related activities.

b) Landfill sites produce foul smell if the wastes are not stored and treated properly.

c) Hazardous wastes mixed up with paper and other scraps burnt to produce solid wastes.

d) Dioxins gases are toxic, carcinogenic and pollute the surrounding.

Explanation

Solid Waste Management: Every day tones of solid wastes are disposed of at landfill sites. This waste comes from homes, offices, industries and various other agricultural related activities. These landfill sites produce foul smell if waste is not stored and treated properly. When hazardous wastes like pesticides, batteries containing lead, cadmium, mercury or zinc, cleaning solvents, radioactive materials, e-waste and plastics are mixed up with paper and other scraps and burnt, they produce gases such as dioxins. These gases are toxic and carcinogenic. These pollute the surrounding.

67. Choose the correct statements.

i) G. Nammalvar was an agricultural scientist, environmental activist and an expert of organic farming.

ii) Nammalvar Ecological Foundation for Farm Research and Global Food Security Trust was founded by G. Nammalvar.

iii) Nammalvar wrote Tamil and English books on natural farming, pesticides and fertilizers.

a) i only

b) ii only

c) iii only

d) All the above

Explanation

G. Nammalvar was a supporter and expert of organic farming. He was an agricultural scientist, environmental activist celebrated for his work on spreading Ecological farming & Organic farming. He was against the use of chemical fertilizers and pesticides. He trained hundreds of farmers in natural farming. Nammalvar was the author of several Tamil and English books on natural farming, pesticides & fertilizers and was featured in magazines & television programs. He founded the Nammalvar Ecological Foundation for Farm Research and Global Food Security Trust or simply Vaanagam at Karur, Tamilnadu.

68. What is the main objective of the organization Kudumbam?

a) Spreading the impacts of Chemical fertilizers and pesticides.

b) Preserving and Regenerating native flora and fauna.

c) Encouraging students to become environmental activist.

d) Supporting and developing organic farming.

Explanation

G. Nammalvar developed social forest at Amman kurai and the Kolunji Ecological Farm in Pudukottai. He and his friends made a 10-acre barren land into fertile cultivable land in the dry Pudukottai district. He planted 52 varieties of trees in the same waste land extending in 20 acres. His organization ‘Kudumbam’ preserves and regenerates hundreds of native flora and fauna, in order to ensure a sustainable livelihood.

69. Match

A. Commercial i) Packaging wastes

B. E-waste ii) Drugs and chemicals

C. Industrial iii) Cardboard materials

D. Biomedical iv) Electronics items

a) iv, iii, ii, i

b) ii, i, iv, iii

c) iii, iv, i, ii

d) i, iv, ii, iii

Explanation

Major sources of solid waste

70. Which of these is not a waste management practice?

a) Source segregation

b) Bio fertilizers

c) Vermicomposting

d) Biogas Generation

Explanation

Waste management practices: Source segregation, Composting (Aerobic, Anaerobic), Vermicomposting, Biogas generation and Incineration

71. Assertion (A): Radioactive wastes remain radioactive for few hours or thousands of years.

Reasoning(R): Radioactive wastes can be in gas, liquid or solid and its radioactivity level are varied.

a) Both A and R is True and R is the correct explanation of A.

b) Both A and R is True but R is not the correct explanation of A.

c) A is True but R is False.

d) Both A and R is False.

Explanation

Radioactive wastes are generated during various operations of the nuclear power plant. Radioactive waste can be in gas, liquid or solid form, and its level of radioactivity can vary. The waste can remain radioactive for a few hours or several months or even hundreds of thousands of years.

72. Based on which factor the radioactive wastes are classified?

a) Nature of radioactivity

b) Sources of radioactivity

c) Number of associated sources

d) All the above

Explanation

Depending on the level and nature of radioactivity, radioactive wastes can be classified as exempt waste, Low and Intermediate level waste and High Level Waste.

73. What are the environmental concerns our earth is facing now?

a) Pollution

b) Global warming

c) Deforestation

d) All the above

Explanation

Earth is currently facing a lot of environmental concerns like air pollution, water pollution, and noise pollution,global warming, acid rain, biomagnification, eutrophication, deforestation, waste disposal, ozone layer depletion and climate change.

74. Based on which factor the pollutants are classified into two groups?

a) Eco system

b) Functions

c) Nature

d) Size

Explanation

In terms of eco-system, pollutants can be classified into two basic groups – Nondegradable

and degradable.

75. Match

A. Non-persistent pollutants i) Mercury

B. Persistent pollutants ii) Vegetable waste

C. Non-degradable pollutants iii) DDT

a) i, iii, ii

b) ii, i, iii

c) ii, iii, i

d) iii, i, ii

Explanation

Rapidly degradable or non-persistent pollutants: These can be broken down by natural processes. Domestic sewage and vegetable waste are examples of such pollutants. Slowly degradable or persistent pollutants: These are pollutants that remain in the environment for many years in an unchanged condition and take decades or longer to degrade, as in the case of DDT. Non-degradable pollutants: These cannot be degraded by natural processes. Once they are released into the environment, they are difficult to be eliminated andcontinue to accumulate (biomagnification). Toxic elements like lead, mercury, cadmium, chromium and nickel are such common pollutants.

76. Which of the following is the major air pollutant?

a) Ethanol

b) Nitrogen oxide

c) Cadmium

d) Suplhur

Explanation

The alterations or changes in the composition of the earth’s atmosphere by natural or human

activities (anthropogenic factors) are referred as Air Pollution. Nitrogen oxides are also major air

pollutants. Fossil fuel combustion and automobiles exhausts are the source of nitrogen oxides.

77. Which of the following is hourly updated by the Sameer App?

a) National Air Quality Index

b) Human Development Index

c) Stock Market Index

d) Pollution Index

Explanation

Sameer, an App provides hourly updates on the National Air Quality Index (AQI) published by CPCB.

78. Which of these AQI value indicates the Hazardous air pollution level?

a) 101-150

b) 50+

c) 300+

d) 201-300

Explanation

79. Choose the incorrect statement.

i) Water has three dimensions that are closely linked.

ii) Quantity and quality of water is not connected to each other.

a) i only

b) ii only

c) Both i and ii

d) Neither i nor ii

Explanation

Water has two dimensions that are closely linked: quantity and quality.

80. By which of these process pollutants reach water bodies?

a) Precipitation

b) Run-off

c) Percolation

d) All the above

Explanation

Water pollutants reach water bodies like rivers, streams and the marine system by precipitation, run-off and the groundwater by seepage or percolation.

81. Which of these excess value in drinking water causes fluorosis?

a) Fluroide

b) Chlorine

c) Magnesium

d) Lead

Explanation

Excess of fluoride in drinking water causes fluorosis.

82. In which year the national river conservation plan was enacted?

a) 1991

b) 1983

c) 1995

d) 1970

Explanation

National river conservation plan (NRCP) was enacted in 1995 to improve the water quality of the rivers, which are the major fresh water resources in our country. This important assignment taken up under the NRCP includes, To capture the raw sewage flowing into the river through open drains and divert them for treatment. Setting up sewage treatment plants for treating the diverted sewage. Construction of low cost sanitation toilets to prevent open defecation on river banks.

83. Which of the following is correct regarding Namami Gange,

i) National Mission for Clean Ganga river.

ii) It is a Flagship Programme

iii) In 2014 Union government approved this program.

a) i only

b) ii only

c) iii only

d) All the above

Explanation

Namami Gange (National Mission for Clean Ganga) Programme is an Integrated Conservation Mission approved as the ‘Flagship Programme’ of the Union Government in June 2014.

84. Which year Noise pollution rules declare the permissible limit of noise in India?

a) 2000

b) 1972

c) 1995

d) 1981

Explanation

According to Noise Pollution (Regulation and Control) Rules, 2000, the permissible limit of noise in areas categorized as commercial is 65 decibels (dB) during day and 55 dB during night.

85. What are the effects of over usage of agrochemicals?

a) Kills beneficial bacteria and soil organisms.

b) Eutrophication in water bodies

c) Trigger harmonic disorders and neurotoxicity

d) All the above

Explanation

Overuse of agrochemicals has been observed to generate residues that cause nutrient imbalance, and may kill beneficial bacteria and soil organisms. To can cause eutrophication in water bodies. Affect aquatic animals and their productivity. Pesticide containing water, even in trace quantities is unfit for human consumption. Particles (aerosols) and residues of these chemicals cause air pollution. Inhalation of contaminated air can cause respiratory problems. Consumption can lead to poisoning, side effects and after effects. Chemicals can cause skin rashes and irritation of eyes. Many of these chemicals are reported to be carcinogenic. They can trigger hormonal disorders and neurotoxicity. Beneficial insects and animals can be affected.

86. What is the main reason for the colony collapse syndrome in Honey bees?

a) Global warming

b) Acid rain

c) Pesticides/ Herbicides

d) UV rays

Explanation

Colony collapse syndrome in Honey bees due to pesticides/herbicides can lead to destruction of hives and lower agricultural productivity.

87. Which is not a normal metabolic process?

a) Usage of food

b) Food transformation

c) Waste management

d) Food storage

Explanation

Usage, storage and transformation of food and bio molecules by metabolism are a normal process.

88. Which of this process increase the non-degradable substances in the food chain?

a) Bio magnification

b) Bio ethics

c) Bio molecules

d) Bio reserve

Explanation

When non-degradable substances enter the food chain, they do not get metabolized or broken down or expelled and instead get transferred up the tropic levels of the food chain. During this process, they show an increase in concentration which is referred to as biomagnifications.

89. Which of this natural process increase the nutrient enrichment of water bodies?

a) UV radiation

b) Natural aging

c) Increase in micro organisms

d) Aquatic lifestyle

Explanation

Natural aging of lakes also leads to nutrient enrichment of its water. In a lake, the water is cold and clear (oligotrophic stage), supporting little life. With time, streams draining into the lake introduce nutrients such as nitrates and phosphates, which encourage the growth of aquatic organisms. Aquatic plants and animal life grow rapidly, and organic remains begin to be deposited on the lake bottom (mesotrophic stage)

90. What are the negative effects of excess nutrients in water bodies?

a) Clogging of canals and rivers

b) Displace native plants

c) Unpleasant odors and foam

d) All the above

Explanation

Nutrients stimulate the growth of algae, water hyacinth and can cause clogging of canals, rivers and lakes as well as, displacing native plants. It causes unsightly foam and unpleasant odors, and deprives the water of dissolved oxygen.

91. Which of these micro organisms is essential in the sewage water treatment?

a) Fungus

b) Virus

c) Bacteria

d) Algae

Explanation

Microorganisms mainly bacteria and some protozoa play an essential part in the treatment of sewage to make it harmless. Sewage contains pathogenic bacteria. These bacteria must be destroyed in order to prevent the spread of diseases.

92. What is the effect of primary effluents passed through aeration tanks of waste water?

a) Rapid growth of Inorganic matters.

b) Reduced BOD

c) Increase in Inorganic sedimentation.

d) Increase in BOD

Explanation

The primary effluent is passed into large aeration tanks where it is constantly agitated mechanically and air is pumped into it. This allows vigorous growth of useful aerobic microbes into floc (masses of bacteria associated with fungal filaments to form mesh like structures). While growing, these microbes consume the major part of the organic matter in the effluent. This significantly reduces the BOD (Biochemical oxygen demand or Biological oxygen demand).

93. Which of the following is not related to the BOD of water?

a) Amount of oxygen consumed

b) Organic matter oxidized by bacteria

c) Inorganic matters deoxidized by bacteria

d) Both a and b

Explanation

BOD refers to the amount of the oxygen that would be consumed, if all the organic matter in one litre of water were oxidized by bacteria.

94. What are the effects of tertiary treatment of waste water?

a) Improves the quality of waste water

b) Removes inorganic compounds and substances

c) Final process of waste water management

d) All the above

Explanation

Tertiary treatment is the final process that improves the quality of the waste water before it is reused, recycled or released into natural water bodies. This treatment removes the remaining inorganic compounds and substances, such as nitrogen and phosphorus

95. Which of this process replaces the chlorination system for water treatment?

a) UV method

b) Filtration method

c) Minerals extraction

d) Oxygen diffusion

Explanation

UV is a chemical free process that can completely replace the existing chlorination system and also inactivates chlorine-resistant microorganisms like Cryptosporidium and Giardia.

96. Which of the following statement is correct regarding the organic farming?

i) Farming system aims at cultivating the land and raising crops to keep the soil alive.

ii) Use of organic wastes such as crop, animal and farm wastes, aquatic wastes.

iii) Eco-friendly pollution free environment.

a) i only

b) ii only

c) iii only

d) All the above

Explanation

Organic Farming and Its Implementation : It is a method of farming system which primarily aims at cultivating the land and raising crops in such a way, so as to keep the soil alive and in good health by use of organic wastes (crop, animal and farm wastes, aquatic wastes) and other biological materials along with beneficial microbes (bio-fertilizers) to release nutrients to crops for increased sustainable production in an eco-friendly pollution free environment.

97. Which of the following were focused by Dr.Sultan Ahmed Ismail?

i) Recycling biodegradable waste into fertilizer using earthworms.

ii) Soil bioremediation

a) i only

b) ii only

c) Both i and ii

d) Neither i nor ii

Explanation

Dr. Sultan Ahmed Ismail is an Indian soil biologist and ecologist from Tamil Nadu. His work has centered on techniques for recycling biodegradable waste into fertilizer using varieties of earthworms, and on soil bioremediation.

98. Match

A. Three mile Island i) United States

B. Fukushima Daiichi ii) Ukraine

C. Chernobyl iii) Japan

a) ii, i, iii

b) i, iii, ii

c) iii, i, ii

d) ii, iii, i

Explanation

The Three Mile Island (Pennsylvania, United States), Chernobyl (Pripyat, Ukraine) and Fukushima Daiichi (Ōkuma, Japan) are nuclear disasters the world has seen in the recent period.

99. Which of the disposal method is used for low radioactivity wastes?

a) Dilute and disperse

b) Confine process

c) Delay and decay

d) Limit generation

Explanation

Dilute and disperse – For wastes having low radioactivity, dilution and dispersion are adopted.

100. How many ways are employed to manage nuclear wastes?

a) 5

b) 3

c) 2

d) 4

Explanation

Three ways are employed to manage nuclear wastes Spent Fuel Pools, Vitrification method and geological repositories.

101. Define the vitrification method of nuclear waste management?

a) Submerging nuclear wastes in deep oceans.

b) Encasing nuclear waste in dry cement caskets.

c) Deep geological repository in stable environment.

d) Stored in cooling ponds.

Explanation

Vitrification method – This prevents reaction or degradation of nuclear waste for

extended periods of time and encased in dry cement caskets.

102. Which of the following methodologies provide long term isolation of nuclear wastes without future maintenance?

a) Geological repositories

b) Vitrification method

c) Incineration

d) Landfills

Explanation

Geological Repositories – A deep geological repository is a nuclear waste repository excavated deep within a stable geologic environment. It is suited to provide a high level of long-term isolation and containment without future maintenance.

103. In which of the following places wet storage of spent fuel is maintained in India?

a) Kudankulam

b) Kaiga

c) Kalpakkam

d) Narora

Explanation

In India at Tarapur and Kalpakkam, a wet storage facility of Spent Fuel is the main mode of storage.

104. Which of the following are categorized under medical wastes?

a) Medical research centers

b) Pharmaceutical companies

c) Hospital and laboratories

d) All the above

Explanation

Any kind of waste that contains infectious material generated by hospitals, laboratories, medical research centers, Pharmaceutical companies and Veterinary clinics are called medical wastes.

105. What is the final disposal of wastes as per norms?

a) Incineration

b) Landfills

c) Encapsulation

d) Autoclaving

Explanation

Waste disposal: Involved by incineration, chemical disinfection, autoclaving, encapsulation, microwave irradiation are methods of waste disposals. Final disposal includes landfill and burying as per norms inside premises.

106. Which of these makes the e-wastes as non-degradable?

a) Polychlorinated biphenyl

b) Polycarbonates

c) Polyvinyl saccharin

d) All the above

Explanation

E-wastes are basically PCB (Polychlorinated biphenyl) based, which are Non-degradable

107. Choose the correct statements.

i) Recycling and disposal of e-waste involves significant risk.

ii) E-waste recycling operations are unsafe and leaking of heavy metals from landfills and incinerator ashes.

a) i only

b) ii only

c) Both i and ii

d) Neither i nor ii

Explanation

Recycling and disposal of e-waste may involve significant risk to the health of workers and communities in developed countries and great care must be taken to avoid unsafe exposure in recycling operations and leaking of materials such as heavy metals from landfills and incinerator ashes.

108. Which of this is not a characteristic of the plastics?

a) Low molecular weight

b) Degradable

c) Organic polymers

d) Non-degradable

Explanation

Plastics are low molecular weight organic polymers that are non-degradable in the natural environment.

109. Which of this waste is the major part of municipal solid waste?

a) Plastic waste

b) E-waste

c) Sewage waste

d) Domestic household wastes

Explanation

Packaging materials used in supermarkets, retail outlets, manufacturing industries, households, hotels, hospitals, restaurants and transport companies are major contributors to plastic waste generation. Plastic waste constitutes a major part of municipal solid waste.

110. Which of these is not a remedy for the plastic waste pollution?

a) Regenerate

b) Refuse

c) Reuse

d) Recycle

Explanation

Remedies: ‘4R’- Refuse, Reduce, Reuse and Recycle mantra is the best available remedy for plastic waste pollution.

111. Which of these state implemented the ban on single use plastics from 1st Jan 2019?

a) Telangana

b) Tamil Nadu

c) Kerala

d) Maharashtra

Explanation

Tamil Nadu State government successfully implemented the ban on single use plastics from 1st January 2019.

12th Science Lesson 17 Questions in English

17] Organisms and Populations

1. Which is the origin of the word ecology?

a) Greek

b) Latin

c) Rome

d) Arab

Explanation

The word ‘ecology’ is derived from the Greek term ‘oikos’, meaning ‘house’ and logos, meaning ‘study’. Thus, the study of the environmental ‘house’ includes all the organisms in it and all the functional processes that make the house habitable.

2. Which of this factor is not included in the study of ecology?

a) Population

b) Ecosystem

c) Global warming

d) Community

Explanation

The study of ecology encompasses different levels-organism, population, community, ecosystem, etc., in ecology, the term population, originally coined to denote a group of people is broadened to include groups of individuals of any one kind of organism.

3. Assertion (A): The Ecological system includes all the biotic and abiotic functions.

Reasoning(R): Biotic Community includes all the populations occupying in a given area.

a) Both A and R is True and R is the correct explanation of A.

b) Both A and R is True but R is not the correct explanation of A.

c) A is True but R is False.

d) Both A and R is False.

Explanation

Community in the ecological sense (designated as ‘biotic community’) includes all the populations occupying a given area. The community (Biotic) and the non-living environment (Abiotic) function together as an ecological system (or) ecosystem.

4. Which of these is characterized by the major vegetation type?

a) Biome

b) Ecosphere

c) Abiotic

d) All the above

Explanation

Biome is a term in wide use for a large regional or sub continental system characterized by a major vegetation type.

5. Choose the correct statements.

i) Ecosphere is designated as the largest self-sufficient biological system.

ii) Living environment interacts with the physical environment to regulate their distribution, abundance, production and evolution.

a) i only

b) ii only

c) Both i and ii

d) Neither i nor ii

Explanation

The largest and most nearly self-sufficient biological system is often designated as the Ecosphere, which includes all the living organisms of the Earth, interacting with the physical environment to regulate their distribution, abundance, production and evolution.

6. Which of the following is not an abiotic component?

a) Light

b) UV light

c) Pressure

d) Salinity

Explanation

Every living organism has its own specific surrounding, medium or environment with which it continuously interacts and develops suitable adaptations for survival there. Environment is a collective term which includes the different conditions in which an organism lives or is present. The common and influencing factors in any environment are light, temperature, pressure, water, salinity. These are collectively referred to as Abiotic components.

7. Assertion (A): An Organism’s growth, distribution, number, behavior and reproduction is affected by the environment.

Reasoning(R): The variable and dynamic environment influences the inhabiting organisms.

a) Both A and R is True and R is the correct explanation of A.

b) Both A and R is True but R is not the correct explanation of A.

c) A is True but R is False.

d) Both A and R is False.

Explanation

Environments are variable and dynamic, in which temperature changes and light changes are diurnal and seasonal. These influence the organisms inhabiting them. An organism’s growth, distribution, number, behavior and reproduction is determined by the different factors present in the environment.

8. Choose the correct statements.

i) Habitat includes all biotic and abiotic factors or the environment conditions.

ii) The collection of all habitat areas of a species does not constitute to its geographical range.

iii) Various organisms in a habitat do not need to interact with one other.

a) i only

b) ii only

c) iii only

d) None of the above

Explanation

Habitat refers to the place where an organism or communities of organisms live, including all biotic and abiotic factors or conditions of the surrounding environment. The collection of all the habitat areas of a species constitutes its geographical range. Organisms in a habitat interact with each other and can be part of trophic levels to form food chains and food webs.

9. Assertion (A): Ecological niche includes the physical space occupied by an organism and its functional role in the community.

Reasoning(R): The ecological niche of an organism depends on where it lives and also includes the sum total of its environmental requirements.

a) Both A and R is True and R is the correct explanation of A.

b) Both A and R is True but R is not the correct explanation of A.

c) A is True but R is False.

d) Both A and R is False.

Explanation

Niche (or) Ecological Niche: As every organism has its unique habitat, so also it has an ecological niche which includes the physical space occupied by an organism and its functional role in the community. The ecological niche of an organism not only depends on where it lives but also includes the sum total of its environmental requirements.

10. Which of this term was coined by Charles Elton for the functional status of an organism in a community?

a) Guilds

b) Niche

c) Ecology

d) Nano

Explanation

Charles Elton (1927) was the first to use the term ‘niche’ as the functional status of an organism in its community.

11. Assertion (A): The life style of an individual population in the habitat is known as its niche.

Reasoning(R): Niche of an organism can be defined as the total position and function of an individual in its environment.

a) Both A and R is True and R is the correct explanation of A.

b) Both A and R is True but R is not the correct explanation of A.

c) A is True but R is False.

d) Both A and R is False.

Explanation

Many animals share the same general habitat. But their niches are well defined. The life style of an individual population in the habitat is known as its niche. The cricket and the grasshopper do not interfere with each other’s activities in the same habitat. Thus, niche of an organism can be defined as the total position and function of an individual in its environment.

12. Which of these factors are included in the abiotic factor?

a) Chemical

b) Biological

c) Physiological

d) Atmospheric

Explanation

The abiotic factors include the chemical and physical factors which influence or affect organisms and their functioning in their environment.

13. Which of the factors of organisms are influenced by the temperature?

a) Metabolism

b) Behavior

c) Reproduction

d) All the above

Explanation

Temperature or degree of hotness and coldness is an essential and variable factor in any environment. It influences all forms of life by affecting many vital activities of organisms like metabolism, behavior, reproduction, development and even death in the Biosphere.

14. What is the threshold temperature defined for the Van’t Hoff theory?

a) 10ºC

b) 273K

c) 100ºC

d) 35ºF

Explanation

Van’t Hoff proposed that, with the increase of every 10ºC, the rate of metabolic activity doubles or theReaction rate is halved with the decrease of 10ºC. This rule is referred as the van’t Hoff’s rule.

15. Which of these factors are influenced by the temperature?

a) Sex ratio

b) Reproduction

c) Size and color

d) All the above

Explanation

The metabolism of organisms is regulated by enzymes which are temperature sensitive. In many organisms, determination of sex and sex ratio, maturation of gonads, gametogenesis and reproduction is influenced by temperature. In certain environments, the size and coloration of animals are influenced by temperature.

16. Choose the incorrect statements.

i) According to Bergmann’s rule birds and mammals have greater body size in colder regions than warmer region.

ii) Allen’s rule states that warm blooded animals in colder climates have shorter limbs, ears compared to same species in warmer climates.

iii) Jordon’s rule defines that all the aquatic environments have proportionate relationship between water temperature and fish meristic characters.

a) i only

b) ii only

c) iii only

d) None of the above

Explanation

Birds and mammals attain greater body size in colder regions than warmer regions (Bergmann’s rule). Warm blooded animals, living in colder climates, tend to have shorter limbs, ears and other appendages when compared to the members of the same species in warmer climates (Allen’s rule). In some aquatic environments, an inverse relationship between water temperature and fish meristic characters is observed -lower the temperature, more the vertebrae (Jordon’s rule).

17. Which of these species are larger in size according to Bergman’s rule?

a) Cold regions

b) Temperate region

c) Warm region

d) All the above

Explanation

Bergman’s rule is an eco-geographic principle that states that within broadly distributed taxonomic clade, populations and species of larger size are found in colder and of smaller size are in warmer regions.

18. Assertion (A): Tropic regions have higher diversity and density of populations compared to temperate and Polar Regions.

Reasoning(R): In various regions temperature greatly influences the distribution of organisms.

a) Both A and R is True and R is the correct explanation of A.

b) Both A and R is True but R is not the correct explanation of A.

c) A is True but R is False.

d) Both A and R is False.

Explanation

Temperature influences the distribution of organisms. The tropics have higher diversity and density of populations, when compared to temperate and Polar Regions.

19. Which of these organisms are referred as Eurytherms?

a) Cat

b) Dog

c) Tiger

d) All the above

Explanation

Adaptation to temperature is essential for the survival of the species/organisms. Organisms which can survive a wide range of temperature are referred to as Eurytherms (cat, dog, tiger, human).

20. Which of this aspect is not incorporated by Eurythermy in organisms?

a) Thermoregulation

b) Population diversity

c) Distribution

d) Survival

Explanation

Eurythermy can be an evolutionary advantage: adaptations to cold temperatures (cold-eurythemy) are seen as essential for the survival of species during ice ages. In addition, the ability to survive in a wide range of temperatures increases a species’ ability to inhabit other areas, an advantage for natural selection. Eurythermy is an aspect of thermoregulation in organisms.

21. Which of the following is not a stenotherm organism?

a) Fish

b) Frogs

c) Human

d) Snakes

Explanation

Organisms which can tolerate only a narrow range of temperature are Stenotherm (Fish, Frogs, Lizards and Snakes).

22. Which of these are used to survive in extreme temperatures?

a) Heat resistant spores

b) Cysts

c) Antifreeze proteins

d) All the above

Explanation

Over the course of time, by evolution, animals of different ecological habitats have developed different variations and adaptations to temperature changes. It enabled them to survive in different habitats and develop niches. In case of extreme temperatures, organisms have adapted by forming heat resistant spores, cysts (Entamoeba), antifreeze proteins (Arctic fishes).

23. Assertion (A): Migration is an adaptation to overcome extreme temperatures, water and food scarcity.

Reasoning(R): Hibernation and Aestivation are sleeping adaptations to overcome extreme winters and summer.

a) Both A and R is True and R is the correct explanation of A.

b) Both A and R is True but R is not the correct explanation of A.

c) A is True but R is False.

d) Both A and R is False.

Explanation

Hibernation (winter sleep) and Aestivation (summer sleep) are useful adaptations to overcome extreme winters and summers. In certain conditions, migration is an appropriate adaptation to overcome extreme temperatures and resultant water and food scarcity.

24. Which of the following statements are not true regarding the facts of light?

a) The quality, intensity and duration of light are significant factors for an organism.

b) Light influences growth, pigmentation, migration and reproduction of organisms.

c) Gonads of birds are less active with increasing light in summer.

d) Light influences the locomotion and movement of lower animals.

Explanation

Light: It is an important and essential abiotic factor. Ecologically, the quality (wavelength or color), the intensity (actual energy in gram calories) and duration (length of day) of light are considered significant for organisms. Light is essential for vision. This is proved by the poorly developed or absence of eyes in cave dwelling organisms. Diapause is also influenced by light in animals. Gonads of birds become more active with increasing light in summer. Light influences the locomotion and movement of lower animals.

25. Which of these is not a negative phototaxis organism?

a) Earthworm

b) Moths

c) Volvox

d) Euglena

Explanation

Phototaxis: The movement of organism in response to light, either towards the source of light as in Moths (positive phototaxis) or away from light (Euglena, Volvox, earthworm (negative phototaxis).

26. Which of these is referred as the growth or orientation of organisms in response to light?

a) Phototropism

b) Phototaxis

c) Photo kinesis

d) All the above

Explanation

Phototropism: The growth or orientation of an organism in response to light, either towards the source of light (positive phototropism) as seen in Sunflower, or a way from light (negative phototropism) as in case of the root of plants.

27. Which of this factor is changed in response to light intensity?

a) Growth

b) Speed of locomotion

c) Orientation

d) Movement

Explanation

Photokinesis: A change in the speed of locomotion (or frequency of turning) in a motile organism or cell which is made in response to a change in light intensity is called Photokinesis. It involves undirected random movement in response to light.

28. Choose the correct statements.

i) About three-fourth of the earth’s surface is covered with water.

ii) Water is found in various states and there are many types of water on Earth.

iii) The hardness of the water cannot be removed temporarily by any method.

a) i only

b) ii only

c) iii only

d) All the above

Explanation

Life on earth began in the seas and water is essential for the survival of all forms of life. About three-fourth of the earth’s surface is covered with water (hydrosphere).. If hardness can be removed by boiling, it is temporary hard water and if boiling does not help, it is permanent hard water.

29. Which of the following property is not related to water?

a) Water is present as moisture in the atmosphere.

b) Water is evenly distributed on the earth’s lithosphere.

c) Water is heavier than air.

d) Water is less dense as a solid than as a liquid.

Explanation

Essential properties of water

Water is one of the main agents in Pedogenesis (soil formation). It is the medium for several different ecosystems. It is present as moisture in the atmosphere and the outer layers of the lithosphere and is uneven in distribution on the earth. Water is heavier than air and imparts greater buoyancy to the aquatic medium. This enables organism to float at variable levels. Water is physically unique because it is less dense as a solid (ice) than as a liquid.

30. At which of this value the maximum density of liquid water occurs?

a) 0°C

b) 273K

c) 4°C

d) 100K

Explanation

When water freezes (0°C), it contracts. The maximum density of liquid water occurs at 4ºC. Below that, it expands markedly. This enables ice to float on the top of water bodies. Hence, only the surface of water bodies will freeze, while below the surface, water will be in liquid form, sustaining life.

31. What is the unique property of the water?

a) Available in three states

b) Freezing point

c) Universal solvent

d) Maximum density

Explanation

Water is considered as the Universal solvent. It is the main medium by which chemical constituents are transported from abiotic components to the living components of an ecosystem.

32. Which of this water property allows the water striders to remain at the surface of a water body?

a) High surface tension

b) Maximum density

c) Heavier than air

d) High heat capacity

Explanation

Water has high surface tension. This allows pollen, dust, and even water striders to remain at the surface of a water body even though they are denser than the water.

33. Assertion (A): Soil is a mixture of organic matter, mineral, gases, liquids and organisms.

Reasoning(R): Pedogenesis is formation of rocks from parent materials by weathering.

a) Both A and R is True and R is the correct explanation of A.

b) Both A and R is True but R is not the correct explanation of A.

c) A is True but R is False.

d) Both A and R is False.

Explanation

Soil is a mixture of organic matter, minerals, gases, liquids and organisms that together support life. The soil zone is known as Pedosphere. Soil is formed from rocks which are the parent materials of soil, by weathering and is called embryonic soil (Pedogenesis).

34. What are the major functions of soil?

a) Medium for plant growth

b) Habitat for many organisms

c) Modifying of earth’s atmosphere

d) All the above

Explanation

Soil has four major functions- medium for plant growth, means for water storage and purification, modifier of earth’s atmosphere, habitat for many organisms, which in turn modify the soil. Soil is formed of many horizontal layers called as Soil Profile.

35. What are the types of soil based on their size differences?

a) Sand

b) Silt

c) Clay

d) All the above

Explanation

Texture of soil – The texture of soil is determined by the size of the soil particles. The types of soil include sand, silt and clay on the basis of their size differences.

36. Define porosity of soil.

a) Space between soil particles in a volume

b) Percentage of soil volume occupied by pore space

c) Interstitial spaces occupied by the soil

d) Movement of water through pore spaces

Explanation

Porosity – The space present between soil particles in a given volume of soil are called pore spaces. The percentage of soil volume occupied by pore space or by the interstitial spaces is called porosity of the soil.

37. Choose the Incorrect statements.

i) Permeability of soil determines the movement of water through pore spaces

ii) Soil permeability is indirectly dependent on the pore size.

iii) Water holding capacity is inversely dependent on soil porosity.

a) i only

b) ii only

c) iii only

d) All the above

Explanation

Permeability of soil-The characteristic of soil that determines the movement of water through pore spaces is known as soil permeability. Soil permeability is directly dependent on the pore size. Water holding capacity of the soil is inversely dependent on soil porosity.

38. What are the effects of soil temperature?

a) Germination of seeds

b) Growth of roots

c) Biological activity of organisms

d) All the above

Explanation

Soil Temperature-Soil gets its heat energy from solar radiation, decomposing organic matter, and heat from the interior of earth. Soil temperature effects the germination of seeds, growth of roots and biological activity of soil-inhabiting micro-and macro-organisms.

39. Which of the following is not a function of soil water?

a) Compactness of soil particles

b) Maintains soil texture

c) Regulates the atmospheric pressure

d) Making habitable for plants and animals

Explanation

Soil water- In soil, water is not only important as a solvent and transporting agent, but also maintains soil texture, arrangement and compactness of soil particles making soil habitable for plants and animals.

40. What is the term used to express the invisible water vapor in the atmosphere?

a) Absolute humidity

b) Relative humidity

c) Specific humidity

d) All the above

Explanation

Moisture in the form of invisible vapor in the atmosphere is called humidity which is generally expressed in terms of absolute humidity, relative humidity or specific humidity.

41. Which of this humidity does not consider temperature factor?

a) Relative humidity

b) Absolute humidity

c) Atmospheric humidity

d) Specific humidity

Explanation

Absolute humidity is the total mass of water vapor present in a given volume or mass of air. It does not take temperature into consideration.

42. Assertion (A): Relative humidity is expressed as a percentage of the amount needed for saturation at the same temperature.

Reasoning(R): High percentage of relative humidity means that air-water mixture is more humid at a given temperature.

a) Both A and R is True and R is the correct explanation of A.

b) Both A and R is True but R is not the correct explanation of A.

c) A is True but R is False.

d) Both A and R is False.

Explanation

Relative humidity is the amount of water vapor present in air and is expressed as a percentage of the amount needed for saturation at the same temperature. Relative humidity is expressed as a percentage; a high percentage means that the air-water mixture is more humid at a given temperature.

43. Which of these is used to measure the humidity?

a) Psychrometer

b) Anemometer

c) Hygrometer

d) Barometer

Explanation

Humidity is measured with a Hygrometer.

44. Choose the Incorrect statements.

i) Altitude affects the temperature and precipitation in an ecosystem.

ii) When altitude increases temperature and density of oxygen decreases.

iii) Lower altitudes receive snow because of low temperature.

a) i only

b) ii only

c) iii only

d) None of the above

Explanation

Altitude: This factor is mainly the elevation or gradient and it affects temperature and precipitation in an ecosystem or biome. As altitude increases, temperature and density of oxygen decreases. Higher altitudes usually receive snow instead of rain because of low temperature.

45. Define acclimatization.

a) Amount of air needed for saturation at defined temperature.

b) Species modifying responses to environmental changes in short span.

c) Managing low atmospheric oxygen and low oxygen demand.

d) Effects of temperature and precipitation in an ecosystem.

Explanation

Animals are known to modify their response to environmental changes (stress) inReasonably short time spans. This is known as Acclimatization. This is observed when people who have moved from the plains to higher altitudes show enhanced RBC count within a few days of settling in their new habitat. This helps them cope with lower atmospheric oxygen and higher oxygen demand.

46. Which of the following statements are not correct about Biomes?

a) Larger regions having similar or common vegetation and climatic conditions.

b) Biomes are defined by soil, climate, flora and fauna.

c) Biome is smaller than a habitat space which includes any single variety of habitats.

d) Temperature, light, water availability determine the type of organisms in a biome.

Explanation

Biomes are large regions of earth that have similar or common vegetation and climatic conditions. They play a crucial role in sustaining life on Earth. They are defined by their soil, climate, flora and fauna. Biomes have distinct biological communities that have been formed in response to a shared physio-chemical climate. Biomes are seen to even spread across continents. Thus, it can be observed that a biome is a broader term than habitat. Any biome can comprise a variety of habitats. Factors such as temperature, light, water availability determine what type of organisms and adaptations are observed in a biome.

47. What are the characters of a biome?

a) Geographical position

b) Predominant plant and animal life

c) Climate and physiochemical environment

d) All the above

Explanation

Characters of a biome: Location, Geographical position (Latitude, Longitude), Climate and physiochemical environment Predominant plant and animal life, Boundaries between biomes are not always sharply defined. Transition or transient zones are seen as in case of grassland and forest biomes.

48. Which of these biomes occupy about 71% of the biosphere?

a) Aquatic Biomes

b) Tundra Biome

c) Alpine Biome

d) Taiga Biome

Explanation

Aquatic Biomes: They occupy about 71% of the biosphere. The aquatic biome is home to millions of aquatic organisms like fishes. The climate of coastal zones is influenced by aquatic bodies.

49. Match

A. Marine i) Wetlands

B. Freshwater ii) Pelagic zones

C. Brackish water iii) Lakes

a) i, ii, ii

b) iii, i, ii

c) ii, iii, i

d) iii, ii, i

Explanation

Aquatic biomes of earth: Freshwater (Lakes, ponds, rivers), Brackish water (Estuaries / Wetlands), Marine (Coral reefs, pelagic zones and abyssal zones)

50. What is the unique feature of terrestrial biomes?

a) Deserts

b) Indicator species

c) Forests

d) Climatic conditions

Explanation

Terrestrial biomes: These are large communities of plants and animals that occupy a distinct region. They include grassland, tundra, desert, tropical rainforest and deciduous and coniferous forests. Terrestrial biomes are distinguished primarily by their predominant vegetation and are mainly determined by climate which in turn determines the organisms inhabiting them. These include the keystone species and indicator species which are unique to their respective biomes. The terrestrial biomes are a source of food, O2 and act as CO2 sink apart from the climate regulatory role.

51. Which is not a major biome of the earth?

a) Grassland biome

b) Forest biome

c) Alpine biome

d) Concrete biome

Explanation

Major Biomes of the Earth Tundra biome, Taiga biome, Grassland biome, Alpine biome, Forest biome and Desert biome.

52. Choose the correct statements about Tundra biome.

i) Tundra biome is almost treeless plains.

ii) Long winters with little daylight and short summers with long daylight hours.

iii) Most of the animals are migratory due to severe winter seasons.

a) i only

b) ii only

c) iii only

d) All the above

Explanation

TUNDRA BIOME: This is the almost treeless plain in the northern parts of Asia, Europe and North America. Winters are long with little daylight, summers are short, with long daylight hours. Precipitation is less than 250 mm per year. It is a zone of permafrost. Dwarf willows, birches, mosses, grasses, sedges are the flora here. Reindeer, arctic hare, musk ox, lemmings are important Tundra herbivores. Some important carnivores are the arctic fox, arctic wolf, bobcat and snowy owl. Polar bears live along coastal areas. Because of the severe winters, many of the animals are migratory. For example, the many shore birds and waterfowl such as ducks and geese, nest in the Tundra during the summer and migrate south for the winter.

53. Which of the following statements are not true regarding Taiga Biome?

a) Taiga has long and cold winters.

b) Major source for the logging industry with coniferous trees.

c) Annual precipitation ranges about 380-1000 mm.

d) Migratory herbivores move to Tundra for winters.

Explanation

TAIGA BIOME: The Taiga is 1300-1450 km wide zone south of the Tundra. This area has long and cold winters. Summer temperature ranges from 10⁰ C to 21⁰ C. Precipitation ranges about 380-1000 mm annually. Important migratory herbivores include moose, elk, deer and reindeer. Moose and reindeer migrate to the Taiga for winter and to the Tundra for summers. The common smaller mammals are herbivorous squirrels, snowshoe hare and predatory pine martens. Important predators include the timber wolf, grizzly bear, black bear, bobcat and wolverines.

54. Which of this factor differentiate temperate deciduous forest and grasslands of Taiga biome?

a) Hot summers

b) Low irregular rainfall

c) Long cold winters

d) High winds

Explanation

Taiga Biome has hot summers, cold winters, and irregular rainfall. Often they are characterized by high winds. The low irregular rainfall is the factor which makes the difference between a temperate deciduous forest and temperate grassland. Herbivores like antelope, bison, wild horse, jack rabbit, ground squirrel and prairie dogs are abundant.

55. Which of these zones are included in the alpine biome?

a) Shrub zone

b) Snow zone

c) Meadow zone

d) All the above

Explanation

Alpine biome: The alpine zone (zone between timber line and snow zone) includes in the descending order, a sub-snow zone immediately below the snow zone, a meadow zone in the center and a shrub zone which gradually merges into the timber zone. Flora of alpines includes alpine phacelia, bear grass, bristlecone pine, moss champion, polylepis forest, pygmy bitterroot, and wild potato.

56. Assertion (A): Forest biomes consist of a complex assemblage of different kinds of biotic communities.

Reasoning(R): Tropical forests and the temperate forests are the major forest biomes.

a) Both A and R is True and R is the correct explanation of A.

b) Both A and R is True but R is not the correct explanation of A.

c) A is True but R is False.

d) Both A and R is False.

Explanation

Forest biomes: Forest is a broad term used to describe areas where there are a large number of trees. The forest biomes include a complex assemblage of different kinds of biotic communities. The major forest biomes are the Tropical forests and the temperate forests.

57. Which of the following season is absent in a tropical forest?

a) Rainy

b) Winter

c) Dry

d) Autumn

Explanation

Tropical forest: They occur near the equator (between latitudes 23.5⁰ at north and 23.5⁰ at south). The major characteristic of tropical forests is their distinct seasons. Only two seasons are presented (rainy and dry). Winter is absent. The length of daylight is about 12 hours and varies little.

58. Choose the Incorrect statements regarding the tropical forests characteristics.

i) Annual rainfall exceeds 2000mm and the precipitation is evenly distributed throughout the year.

ii) Average annual temperature ranges between 2°C to 10°C.

iii) Soil is nutrient-poor, acidic and has rapid decomposition.

a) i only

b) ii only

c) iii only

d) All the above

Explanation

Tropical forests average annual temperature ranges between 20° C and 25° C. Precipitation is evenly distributed throughout the year with annual rainfall exceeding 2000 mm. Soil is nutrient-poor and acidic. Decomposition is rapid and soils are subject to heavy leaching.

59. Which of the following places does not have a temperate forest?

a) North eastern Asia

b) North western Africa

c) Central Europe

d) North eastern America

Explanation

Temperate forest occur in eastern North America, northeastern Asia and western and central Europe. Have well-defined seasons with a distinct winter. Moderate climate and a growing season of 140-200 days during 4-6 frost free months distinguish temperate forests. Annual temperature varies from -30° C to 30° C. Precipitation (750-1500 mm) is distributed evenly throughout the year.

60. Which of the following is not a characteristic of a Desert biome?

a) About one third of the earth’s surface is covered by deserts.

b) Evaporation rates exceed the rainfall rates regularly.

c) Sand dunes are common.

d) Average rainfall is below 500mm/year.

Explanation

Desert biomes: Deserts cover about one fifth of the earth’s surface and occur where rainfall is >500 mm/year. Rainfall is usually very low and/or concentrated in short bursts between long rainless periods. Evaporation rates regularly exceed rainfall rates. Soils are course-textured, shallow, rocky or gravely with good drainage and have no subsurface water. The finer dust and sand particles are blown elsewhere, leaving heavier pieces behind. Sand dunes are common.

61. Choose the Incorrect statements regarding the temperature of the deserts.

i) Minimum temperature drops to -18°C sometimes.

ii) Maximum extreme ranges between 43.5°C to 49°C.

iii) Very Hot and Hot deserts are categorized based on the temperature range.

a) i only

b) ii only

c) iii only

d) None of the above

Explanation

Mean annual temperatures range from 20-25° C. The extreme maximum ranges from 43.5 – 49° C. Minimum temperatures sometimes drop to -18° C. Based on the temperature range, deserts can be hot deserts and Cold deserts.

62. Which of the following are the hot deserts of the Earth?

a) Sahara

b) Mexico

c) Thar

d) All the above

Explanation

Hot deserts such as the Sahara of North Africa and the deserts of the southwestern U.S., Mexico, Australia and India (Thar Desert) occur at low latitudes.

63. Which of these common fauna does not belong to the Thar deserts?

a) Blackbuck

b) Bison

c) White foot fox

d) Spiny tailed lizard

Explanation

The dominant animals of warm deserts are reptiles and small mammals. The Indian Spiny-tailed lizard, the blackbuck, the white-footed fox are the common fauna of the Thar deserts. There are also insects, arachnids and birds.

64. Which of the following is not a cold desert?

a) Greenland

b) Antarctic

c) Ladakh

d) Alps

Explanation

Cold deserts occur in Antarctic, Greenland and the NearcticRealm, parts of USA and in parts of western Asia and the Ladakh region in India. Widely distributed animals are jack rabbits, kangaroo rats, kangaroo mice, pocket mice, grasshopper mice, antelope and ground squirrels.

65. Choose the Incorrect statements.

i) Atacama Desert of Chile has the lowest average rainfall of less than 15mm.

ii) Inland Sahara receives the highest rainfall as almost 280 mm a year.

a) i only

b) ii only

c) Both i and ii

d) Neither i nor ii

Explanation

Rainfall is lowest in the Atacama Desert of Chile, where it averages less than 15 mm. Some years are even rainless. Inland Sahara also receives less than 15 mm rainfall a year. Rainfall in American deserts is higher — almost 280 mm a year.

66. Match

A. Migrate i) Constant internal environment

B. Regulate ii) Diapause

C. Conform iii) Osmotic balance

D. Suspend iv) Temporary movement

a) iii, i, ii, iv

b) ii, iv, iii, i

c) iv, iii, i, ii

d) i, iii, iv, ii

Explanation

The types of responses observed are

Regulate: Some organisms are able to maintain homeostasis by physiological means which ensures constant body temperature, ionic / osmotic balance. Birds, mammals and a few lower vertebrate and invertebrate species are capable of such regulation.

Conform: Most animals cannot maintain a constant internal environment. Their body temperature changes with the ambient temperature. In aquatic animals like fishes, the osmotic concentration of the body fluids changes with that of the ambient water osmotic concentration. Such animals are called Conformers. In case of extreme condition, the inhabitants relocate themselves as in migration.

Migrate: Organisms tend to move away temporarily from a stressful habitat to a new, hospitable area and return when the stressful period is over. Birds migrate from Siberia to Vedanthangal in Tamilnadu to escape from the severe winter periods.

Suspend: In certain conditions, if the organisms is unable to migrate, it may avoid the stress by becoming inactive. This is seen commonly in bears going into hibernation during winter. Some snails and fish go into aestivation to avoid summer related problems like heat and desiccation. Some lower animals suspend a certain phase of their life cycle, which is referred to as diapause.

67. Assertion (A): Adaptation is a dynamic process that makes organisms to enhance their evolutionary fitness.

Reasoning(R): Adaptation can be phenotypic or adaptive trait with a functional role in each organism.

a) Both A and R is True and R is the correct explanation of A.

b) Both A and R is True but R is not the correct explanation of A.

c) A is True but R is False.

d) Both A and R is False.

Explanation

In biology, adaptation is a dynamic evolutionary process that fits organisms to their environment and enhancing their evolutionary fitness. Adaptations can be a phenotypic or adaptive trait with a functional role in each individual organism that is maintained and has been evolved by natural selection.

68. Which of the following is not an adaptive trait of an organism?

a) Behavioral adaptation

b) Evolutionary adaptation

c) Structural adaptation

d) Physiological adaptation

Explanation

Adaptations can be a phenotypic or adaptive trait with a functional role in each individual organism that is maintained and has been evolved by natural selection. The adaptive traits may be structural adaptation, behavioral adaptation and physiological adaptation.

69. Choose the correct statements.

i) The External and internal structures of animals helps to adapt to the environment.

ii) Crypsis and mimicry are some of the most attractive adaptations in nature.

a) i only

b) ii only

c) Both i and ii

d) Neither i nor ii

Explanation

Structural adaptations: The external and internal structures of animals can help them to adapt better to their environment. Some of the most common examples are mammals growing thicker fur to survive freezing climates. Some of the most attractive adaptations in nature occur forReasons of crypsis (e.g. camouflage) and mimicry. Cryptic animals are those which camouflage perfectly with their environment and are almost impossible to detect. Certain reptiles and insects such as chameleons and stick insects show this type of adaptation which helps in prey capture or to evade from predators. Likewise, horse legs are suitable for fast running and adapted for grasslands and similar terrestrial environments.

70. List some of the behavioral adaptation actions of organisms.

a) Fleeing from a predator

b) Seeking refuge from climate change

c) Hiding during sleep

d) All the above

Explanation

Behavioral adaptations: Action and behavior of animals are instinctive or learned. Animals develop certain behavioral traits or adaptations for survival. Fleeing from a predator, hiding during sleep, seeking refuge from climate change or moving to find different food sources are all behavioral adaptations.

71. Choose the Incorrect statements about the behavioral adaptations.

i) Migration and Courtship are the most characteristic forms of behavioral adaptations

ii) Migration allows the animals to change their external structures to fit to the environment.

iii) Courtship is a set of behavioral patterns to find mate to reproduce.

a) i only

b) ii only

c) iii only

d) All the above

Explanation

The two most characteristic forms of behavioral adaptations are migration and courtship. Migration allows the animals to find better resources or evade threat. Courtship is a set of behavioral patterns to find a mate to reproduce. Most nocturnal animals remain underground or inactive during daytime. This is a modification of their feeding and activity pattern or habit or behavior.

72. Which of the following describes the physiological adaptation of an organism?

a) Adaptations used to live and survive with unique niches.

b) Behavioral patterns to find a mate.

c) Adaptation used to find better resources.

d) Adaptation techniques used to fit the changing environment.

Explanation

Physiological adaptations: These are adaptations of organisms that help them to live and survive in their environment with unique niches. Example: Lions have sharp canines to hunt and tear meat and a digestive system suitable for digesting raw meat.

73. Choose the correct statements about the Physiological adaptations.

i) Hibernation and Aestivation are the well-known physiological adaptation techniques.

ii) Physiological adaptation methods speed up the metabolic rate of an animal and make them more active.

iii) Organisms are forced externally to adapt the environment conditions.

a) i only

b) ii only

c) iii only

d) All the above

Explanation

The two most well-known physiological adaptations are hibernation and aestivation. These are two different types of inactivity where the metabolic rate slows down so much that the animal can survive without eating or drinking. Aquatic medium and terrestrial habitats have their own respective environmental conditions. Hence organisms have to evolve appropriate adaptations to select suitable habitats and niches.

74. Identify the Incorrect match.

A. Stream line structure i) Swift movement

B. Excretory structures ii) Respiration

C. Pectoral fins iii) Stabilizers

D. Myotomes iv) Locomotion

a) i only

b) ii only

c) iii only

d) iv only

Explanation

Adaptations of aquatic animals

The pectoral fins and dorsal fins act as stabilizers or balancers and the caudal fin helps in changing the direction as a rudder. Arrangement of body muscles in the form of bundles (myotomes) help in locomotion. Stream lined structure helps in the swift movement of the animals in water. Respiration by gills making use of gases dissolved in water. Presence of air-bladders filled with air for buoyancy. Presence of lateral-line system. They function as rheoreceptors which is helpful in echo locating objects in water. Integuments rich in mucous glands are protected by scales. Maintain water and ionic balance in its body with excretory structures.

75. Which of the following is not a property of a population of a species?

a) Population density

b) Nature of origin

c) Age distribution

d) Dispersion

Explanation

Population is defined as any group of organisms of the same species which can interbreed among themselves, and occupy a particular space and function as part of a biotic community. A population has various properties like population density, natality (birth rate), mortality (death rate), age distribution, biotic potential, dispersion and ‘r’, ‘K’ selected growth forms.

76. Which of these factors are directly related to the genetic characteristics of a population?

a) Adaptiveness

b) Reproduction success

c) Persistence

d) All the above

Explanation

A population possesses genetic characteristics that are directly related to their adaptiveness, reproductive success, and persistence in their habitats over time.

77. Which is not a way to calculate the density of population?

a) Abundance

b) Total Numbers

c) Numerical density

d) Biomass density

Explanation

Population density: The density of a population refers to its size in relation to unit of space and time. Population density is the total number of that species within a natural habitat. The size of the population can be measured in several ways, including abundance (absolute number in population), numerical density (number of individuals per unit area (or) volume) and biomass density (biomass per unit area (or) volume).

78. In which of these conditions the density is expressed in terms of individuals?

a) Size of individuals is uniform.

b) Population is proportionate to the number of species.

c) Total number of species in saturated environment.

d) Number of survived species.

Explanation

The population density of a species can also be expressed with reference to the actual area of habitat available to the species when the size of individuals in the population is relatively uniform then density is expressed in terms of number of individuals (numerical density).

79. State the natality rate or birth rate for an organism.

a) Number of survived species / Total number of species

b) Difference between birth and death per unit time

c) Number of birth per unit time / Average population

d) None of the above

Explanation

Populations increase because of natality. Natality is equivalent to birth rate and is an expression of the production of new individuals in the population by birth, hatching, germination (or) fission. The two main aspects of reproduction, namely fertility and fecundity play a significant role in a population. Natality rate may be expressed in crude birth rate number of organisms born per female per unit time.

Birth rate (b) = number of birth per unit time / average population

80. Assertion (A): Specific mortality is the number of members of an original population dying after a given time.

Reasoning(R): Mortality is expressed as a loss of individuals in unit time or death rate.

a) Both A and R is True and R is the correct explanation of A.

b) Both A and R is True but R is not the correct explanation of A.

c) A is True but R is False.

d) Both A and R is False.

Explanation

Mortality is the population decline factor and is opposite to natality. Mortality can be expressed as a loss of individuals in unit time or death rate. Generally, mortality is expressed as specific mortality, that is, the number of members of an original population dying after the lapse of a given time. The crude death rate of a population can be calculated by the equation.

Death rate (d)= number of deaths per unit time average population.

81. Which of these influence the high mortality rate?

a) Overcrowding

b) Increased predation

c) Spread of disease

d) All the above

Explanation

The rate of mortality (death) is determined by density. Mortality is high at high density because of the hazards of overcrowding, increased predation and spread of disease.

82. To escape from which of the seasons the Siberian cranes migrate to Tamil Nadu?

a) Spring

b) Summer

c) Winter

d) Autumn

Explanation

Migration is a peculiar and unique kind of mass population movement from one place to another and back. To avoid the severe winter cold, Siberian cranes migrate from Siberia to Vedanthangal in Tamil Nadu and return back in spring. Some fishes are known to migrate from sea to fresh water (anadromous migration, Salmon) and some from fresh water to sea (catadromous migration, Eel).

83. When will the emigration occur naturally?

a) Overcrowding

b) Under populated

c) Extreme weather condition

d) Mortality increase

Explanation

Emigration: Under natural conditions, emigration usually occurs when there is overcrowding. This is regarded as an adaptive behavior that regulates the population in a particular site and prevents over exploitation of the habitat. Further, it leads to occupation of new areas elsewhere.

84. Which of the following initiates the emigration and immigration?

a) Weather

b) Abiotic factor

c) Biotic factor

d) All the above

Explanation

Immigration: It leads to a rise in population levels. If the population increases beyond the carrying capacity, it can result in increased mortality among the immigrants or decreased reproductive capacity of the individuals. Both emigration and immigration are initiated or triggered by weather and other abiotic and biotic factors.

85. What is used to determine the reproductive status and the future population of the species?

a) Proportion of age groups

b) Distribution of various age classes

c) Population of young individuals

d) Stable populated distribution

Explanation

The proportion of the age groups (pre- reproductive, reproductive and post reproductive) in a population is its age distribution attribute. This determines the reproductive status of the population at the given time and is an indicator of the future population size.

86. Choose the incorrect statement.

i) Stable population has an even distribution of various age classes.

ii) Declining population has smaller proportion of older individuals.

iii) Rapidly growing population will have larger proportion of young individuals.

a) i only

b) ii only

c) iii only

d) None of the above

Explanation

Usually a rapidly growing population will have larger proportion of young individuals. A stable population will have an even distribution of various age classes. A declining population tends to have a larger proportion of older individuals.

87. Which of these species exhibit the J-shaped growth form?

a) Fish

b) Small mammals

c) Insects

d) All the above

Explanation

J shaped growth form: When a population increases rapidly in an exponential fashion and then stops abruptly due to environmental resistance or due to sudden appearance of a limiting factor, they are said to exhibit J-shaped growth form. Many insects show explosive increase in number during the rainy season followed by their disappearance at the end of the season.

88. Which of the following indicates the maximum reproductive capacity of an organism?

a) r

b) m

c) l

d) n

Explanation

Biotic potential or Reproductive capacity (r) : It is the maximum reproductive capacity of an organism under optimum environmental conditions.

89. Define carrying capacity of an organism.

a) Maximum number of organism supported by a region without environmental degradation.

b) Minimum reproductive capacity of an organism under optimum environmental conditions.

c) Sum of total environmental limiting factors to prevent biotic potential of an organism.

d) Increasing the environmental resistance byReaching the equilibrium.

Explanation

Carrying capacity (K): The maximum number of organism that a region can support without environmental degradation is called carrying capacity. Environmental resistance Is the sum total of the environmental limiting factors, both biotic and abiotic which together act to prevent the biotic potential of an organism from beingRealized.

90. Choose the correct statements.

i) Carrying capacity species are larger sized organisms.

ii) Reproductive capacity produces many off-springs.

iii) K – selected species have short life expectancies.

iv) r- selected species can reproduce more than one in life time.

v) r- selected species have unstable environment.

a) i, ii, v only

b) ii, iv, v only

c) i, iii, v only

d) ii, iv, v only

Explanation

Differences between r- selected and K selected species

91. Assertion (A): Population is static or fluctuates depending on environmental conditions

Reasoning(R): Carrying capacity is regulated by the density dependent and independent factors.

a) Both A and R is True and R is the correct explanation of A.

b) Both A and R is True but R is not the correct explanation of A.

c) A is True but R is False.

d) Both A and R is False.

Explanation

The inherent tendency of all animal populations is to increase in number. But it does not increase indefinitely. Once the carrying capacity of the environment isReached, population numbers remain static or fluctuate depending on environmental conditions. This is regulated by many factors which are Density independent – Extrinsic factors, Density dependent – Intrinsic factors

92. Choose the correct statements.

i) Extrinsic factors influencing carrying capacity are space, shelter, weather and food.

ii) Intrinsic factors of carrying capacity involve competition, predation, emigration and diseases.

a) i only

b) ii only

c) Both i and ii

d) Neither i nor ii

Explanation

Carrying capacity is controlled by two factors. Extrinsic factors include availability of space, shelter, weather, food, etc. Intrinsic factors include competition, predation, emigration, immigration and diseases.

93. Assertion (A): Organisms of different populations interact for food, shelter, mating or far necessities.

Reasoning(R): Intra specific association is observed for all livelihood process like feeding, territoriality, breeding and protection

a) Both A and R is True and R is the correct explanation of A.

b) Both A and R is True but R is not the correct explanation of A.

c) A is True but R is False.

d) Both A and R is False.

Explanation

Organisms belonging to different populations interact for food, shelter, mating or for other necessities. Interaction may be intra specific (interaction within the members of same species) or inter specific (among organisms of different species). Intra specific association is observed for all livelihood processes like feeding, territoriality, breeding and protection.

94. Choose the correct statements.

i) The interspecific and intraspecific associations can be categorized as neutral, positive and negative.

ii) Positive interactions are symbiotic relationship where no organism is harmed.

iii) Negative interactions can be classified as mutualism and commensalism.

a) i only

b) ii only

c) iii only

d) All the above

Explanation

Interspecific associations or interactions can be: Neutral: where different species live together but do not affect each other.

Positive: it is a symbiotic relationship in which no organism in association is harmed and either one or both may be benefitted. It is of two types – Mutualism and Commensalism.

Negative: One or both of the interacting organisms will be affected as in case of competition, predation, and parasitism.

95. In which of these interactions an individual species harm another without any benefit?

a) Amensalism

b) Competition

c) Mutualism

d) Commensalism

Explanation

AMENSALISM (–, 0): This is the ecological interaction in which an individual species harm another without obtaining benefit; large powerful animals harm weak animals. e.g., animals destroyed at the feet of elephants.

96. Assertion (A): Commensalism defines the interaction of two or more species mutually associated centering food.

Reasoning(R): One of the species derives benefit from the commensalism and other species is harmed or nor benefited.

a) Both A and R is True and R is the correct explanation of A.

b) Both A and R is True but R is not the correct explanation of A.

c) A is True but R is False.

d) Both A and R is False.

Explanation

COMMENSALISM (+, 0): This defines the interaction in which two or more species are mutually associated in activities centering on food and one species at least, derives benefit from the association while the other associates are neither benefited nor harmed. The concept of commensalism has been broadened in recent years, to apply to coactions other than those centering on food such as cover, support, production, and locomotion.

97. Which of this environmental resource is not limited for the organisms?

a) Food

b) Air

c) Nesting space

d) Water

Explanation

COMPETITION (–, –): It refers to the type of interaction in which individuals of a species or members of different species vie for limited availability of food, water, nesting space, cover, mates or other resources.

98. Which species face challenges according to Hardin competitive exclusion principle?

a) Weakest

b) Least adapted

c) Least aggressive

d) All the above

Explanation

When resources are in more than adequate to meet the demands of the organisms seeking them, competition does not occur, but when inadequate to satisfy the need of the organisms seeking them, the weakest, least adapted, or least aggressive individuals are often forced to face challenges. This phenomenon is known as the competitive exclusion principle of Hardin.

99. Choose the correct statements.

i) Intraspecific competition occurs between individuals of the same species.

ii) If different species require some common materials or resources it is interspecific competition.

a) i only

b) ii only

c) Both i and ii

d) Neither i nor ii

Explanation

DEGREE OF COMPETITION – Competition is usually keen between individuals of the same species (intraspecific competition) because they have identical requirements for food, mates, and so on. Interspecific competition occurs where different species require at least some resource materials or conditions in common.

100. What are theReasons for the severity of competition between the species?

a) Overlap of the requirements

b) Supply shortage

c) Extent of similarity

d) All the above

Explanation

The severity of competition depends on the extent of similarity or overlap in the requirements of different individuals and the shortage of the supply in the habitat as birds compete with squirrels for nuts, and seeds; insects and ungulates compete for food in grasslands.

101. Assertion (A): Parasitism is a non-harmful interaction between two species.

Reasoning(R): Parasites derives shelter, food and protection from the host organism.

a) Both A and R is True and R is the correct explanation of A.

b) Both A and R is True but R is not the correct explanation of A.

c) A is True but R is False.

d) Both A and R is False.

Explanation

PARASITISM (+, –): It is a kind of harmful interaction between two species, wherein one species is the ‘parasite’ and the other its ‘host’. The parasite benefits at the expense of the host. A parasite derives shelter, food and protection from the host. Parasites exhibit adaptations to exploit their hosts.

102. Match

A. Bacteria i) Endoparasites

B. Arthropods ii) Ectoparasites

C. Leech iii) Microbial

D. Ascaris iv) Zooparasites

a) iii, iv, ii, i

b) ii, i, iv, iii

c) i, iv, iii, ii

d) iv, i, ii, iii

Explanation

The parasites may be viral parasites (plant / animal viruses), microbial parasites (e.g., bacteria / protozoa / fungi), phytoparasites (plant parasites) and zooparasites (animal parasites such as Platyhelminthes, nematodes, arthropods). Parasites may inhabit or attach to the surface of the host (Ectoparasites – Head lice, Leech) or live within the body of the host (endoparasites – ascaris, tapeworm). The endoparasites usually live in the alimentary tract, body cavities, various organs or blood or other tissues of the host.

103. Choose the correct statements.

i) After a limited time period temporary parasites does not exists.

ii) Permanent parasites spend their life completely dependent on their host.

a) i only

b) ii only

c) Both i and ii

d) Neither i nor ii

Explanation

Parasites may be permanent or temporary. Temporary parasites spend only a part of their life cycle as parasites. For example, Glochidium larva of Anadonia (fresh water mussel) attaches itself to the body of fish. Permanent parasites spend their life completely dependent on their host organism.

104. Which of the following is not a permanent parasite?

a) Plasmodium

b) Round worms

c) Entamoeba

d) Anadonia

Explanation

The common examples of permanent parasites are Plasmodium, Entamoeba, Round worms, Pin worms, Tape worms, etc.,

105. Assertion (A): In community dynamics a predator tends to be larger than its prey.

Reasoning(R): Predation is a form of interaction where one animal kills another for food.

a) Both A and R is True and R is the correct explanation of A.

b) Both A and R is True but R is not the correct explanation of A.

c) A is True but R is False.

d) Both A and R is False.

Explanation

PREDATION (+, –): It is a form of interaction where one animal kills another animal for food. Like parasitism, predation is important in community dynamics, but both differ in the point that a predator tends to be larger than its prey, and it catches its prey from without, while a parasite is smaller than its host and consumes it from within.

106. Which of these are called as ecological equivalents?

a) Comparable role in niche dimensions within a community

b) Same niche in different geographical regions

c) Different species of different geographical region

d) Group of species or a community of same niche

Explanation

Groups of species with comparable role and niche dimensions within a community are termed ‘guilds’. Species that occupy the same niche in different geographical regions are termed ‘ecological equivalents’.

107. Which of this part of fish physical structure are designed to suit their niche?

a) Mouth

b) Head

c) Gills

d) Fins

Explanation

In a pond ecosystem, where Catla, Rohu and Mrigal are present, the ecological niche of the Catla is a surface feeder, Rohu is a column feeder and Mrigal is a bottom feeder. Their mouths are designed to suit their niche and hence have different positions and functions in their habitat.

108. What is the role of temperature of environment in the physical structure?

a) Gain weight or improvement in the physical growth

b) Improves reproduction methodologies

c) Regulates the survival of cell

d) Increase in population density of species

Explanation

The minimum and maximum temperature of an environment regulates the survival of a cell.

109. Stenotherms are organisms tolerate a ____ range of temperatures.

a) Wide

b) Narrow

c) Equilibrium

d) Zero

Explanation

organisms which can tolerate only a narrow range of temperature are Stenotherms

110. Which of the following situations can be overcome by adaptation?

a) Extreme temperatures

b) Food scarcity

c) Water scarcity

d) All the above

Explanation

In certain conditions, migration is an appropriate adaptation to overcome extreme temperatures and resultant water and food scarcity.

111. Choose the correct statements.

i) Light influences growth, migration and reproduction in living things.

ii) The sources of light influences metabolic activity and induce gene mutations.

a) i only

b) ii only

c) Both i and ii

d) Neither i nor ii

Explanation

Light influences growth, pigmentation, migration and reproduction. The intensity and frequency of light influences metabolic activity, induce gene mutations (UV, X- rays).

112. Which type of movement is involved in photo kinesis?

a) Linear movement

b) Directed movement towards light

c) Undirected random movement

d) Non-linear movement

Explanation

Photokinesis: A change in the speed of locomotion (or frequency of turning) in a motile organism or cell which is made in response to a change in light intensity is called Photokinesis. It involves undirected random movement in response to light.

113. Choose the Incorrect statements.

i) Water is found in three states.

ii) Only two types of water is found in earth.

iii) Based on dissolved salts water is categorized into four types.

a) i only

b) ii only

c) iii only

d) None of the above

Explanation

Water is found in three states: gaseous, liquid, and solid. There are two types of water on Earth. They are the Fresh water (rivers, lakes, ponds) and the Salt water (seas and oceans). Based on the dissolved salts, water can be hard water (sulphates/nitrates of Calcium/Magnesium) or soft water

114. What is the reason for the constant temperature of ocean and lakes?

a) High heat capacity and latent heat of water.

b) Large area for evaporation of water in oceans.

c) Low atmospheric pressure.

d) Boiling temperature of water.

Explanation

Water has high heat capacity and latent heat, due to which it can withhold large amounts of heat. Thus, oceans and lakes tend to maintain a relatively constant temperature, and the biosphere is relatively thermo stable.

115. Which of the following is not an effect of soil temperature?

a) Growth of roots

b) Decreases soil erosion

c) Germination of seeds

d) Biological activity of soil

Explanation

Soil temperature effects the germination of seeds, growth of roots and biological activity of soil-inhabiting micro-and macro organisms.

116. Which of the following statement is not correct regarding wind?

a) Wind is a natural movement of air with high velocity from any direction.

b) Wind helps to transport pollen grains, seeds and bird flights.

c) The speed of wind is measured with an Anemometer.

d) The Coriolis effect and the differential heating between equator and poles cause wind.

Explanation

Wind is the natural movement of air of any velocity from a particular direction. The two main causes are differential heating between the equator and the poles and the rotation of the planet (Coriolis effect). Wind helps to transport pollen grains, seeds, and even flight of birds. While it is the source of wind energy, it also causes erosion. Wind speed is measured with an Anemometer.

117. By which of these characteristics biomes are not defined?

a) Climate

b) Flora and fauna

c) Minerals

d) Soil

Explanation

Biomes are large regions of earth that have similar or common vegetation and climatic conditions. They play a crucial role in sustaining life on Earth. They are defined by their soil, climate, flora and fauna.

118. Which of these are determined by the various factors of biome?

a) Type of organisms

b) Climate conditions

c) Adaptations

d) Both a and c

Explanation

Any biome can comprise a variety of habitats. Factors such as temperature, light, water availability determine what type of organisms and adaptations are observed in a biome.

119. Which of the following is not a property of a terrestrial biome?

a) Climate regulatory

b) Population of organisms

c) Source of food and Oxygen

d) Carbon dioxide sinks

Explanation

Terrestrial biomes are distinguished primarily by their predominant vegetation and are mainly determined by climate, which in turn, determines the organisms inhabiting them. The terrestrial biomes are a source of food, O2 and act as CO2 sink, apart from the climate regulatory role.

120. Which of these coniferous trees are abundant in taiga region?

a) Spruce

b) Fir

c) Pine

d) All the above

Explanation

The Taiga is a forest of coniferous trees such as spruce, fir and pine. This is a major source for the logging industry. Important predators include the timber wolf, grizzly bear, black bear, bobcat and wolverines.

121. Which of the following is not included in the fauna of grasslands?

a) Elephant

b) Crocodiles

c) Antelope

d) Rhinoceros

Explanation

In India, fauna of grasslands includes Elephant, Gaur, Rhino, Antelope.

122. Which of these characterize the Alpine zone of Himalayas?

a) Sparseness of animal groups

b) Grasslands

c) Coniferous trees

d) All the above

Explanation

Alpine zone of Himalayas is characterized by sparseness of animal groups. Many invertebrates of alpine zone are predatory and occur in lakes, streams and ponds. Among fishes, amphibians and vertebrates are totally lacking and reptilian fauna is greatly impoverished.

123. Which of the following statements is not correct regarding the temperate forest?

a) Temperate forest occurs in South Africa.

b) Well defined seasons with distinct winter.

c) Moderate climate

d) 4-6 of frost free months distinguish temperate forests.

Explanation

Temperate forest : These forests occur in eastern North America, northeastern Asia and western and central Europe. Have well-defined seasons with a distinct winter. Moderate climate and a growing season of 140-200 days during 4-6 frost free months distinguish temperate forests.

124. Which of this vegetation are not specialized for hot deserts?

a) Aloe

b) Lemongrass

c) Opuntia

d) Agave

Explanation

Hot deserts have a considerable amount of specialized vegetation (xerophytes), aloe, agave, Opuntia species, and Euphorbia royleana as well as specialized vertebrate and invertebrate animals.

125. Which of the following refers to physiological homeostasis of organisms?

a) Constant body temperature

b) Osmotic balance

c) Ionic balance

d) All the above

Explanation

Regulate: Some organisms are able to maintain homeostasis by physiological means which ensures constant body temperature, ionic / osmotic balance. Birds, mammals and a few lower vertebrate and invertebrate species are capable of such regulation

126. Which of these are the characteristics of the conformers?

a) Maintaining equal temperature of the environment.

b) Osmotic concentration of body fluids changes with the ambient.

c) Adaptable to the changing climatic conditions

d) None of the above

Explanation

In aquatic animals like fishes, the osmotic concentration of the body fluids changes with that of the ambient water osmotic concentration. Such animals are called Conformers.

127. For which of these earthworms secrete mucus coating?

a) For better natality rate.

b) To maintain a moist situation

c) To increase the survival period.

d) For storing food

Explanation

Earthworms, land Planarians secrete a mucus coating to maintain a moist situation for burrowing, coiling, respiration, etc.

128. In which of the following seasons birds rarely reproduce?

a) Drought

b) Rainy

c) Summer

d) Winter

Explanation

Birds make nests and breed before the rainy season as there is availability of abundant food. But during drought birds rarely reproduce.

129. With reference to which attribute the population has a structure and function?

a) Climatic conditions

b) Environment

c) Time

d) Temperature

Explanation

Life history of an organism is an important part of this attribute. The population has a definite structure and function that can be described with reference to time.

130. Which of these is an expression of the production of new individuals?

a) Natality

b) Productivity

c) Nativity

d) Commonality

Explanation

Populations increase because of natality. Natality is equivalent to birth rate and is an expression of the production of new individuals in the population by birth, hatching, germination (or) fission.

131. Which of this factor determines the rate of mortality?

a) Type of species

b) Density

c) Lifestyle

d) All the above

Explanation

The rate of mortality (death) is determined by density.

132. Which of the following influence mortality rates?

a) Storms and floods

b) Predators

c) Desertion by parents

d) All the above

Explanation

Mortality rates vary among species and are correlated and influenced by a number of factors such as destruction of nests, eggs or young by storms, wind, floods, predators, accidents and desertion by parents.

133. Choose the Incorrect statements.

i) Immigration leads to decrease in population levels of the species.

ii) Mortality increases if the population is increased beyond the carrying capacity.

a) i only

b) ii only

c) Both i and ii

d) Neither i nor ii

Explanation

Immigration leads to a rise in population levels. If the population increases beyond the carrying capacity, it can result in increased mortality among the immigrants or decreased reproductive capacity of the individuals.

134. Which of these stops the rapid increase in population in J-shaped growth forms?

a) Environmental resistance

b) Climatic conditions

c) Soil structures

d) Global warming

Explanation

When a population increases rapidly in an exponential fashion and then stops abruptly due to environmental resistance or due to sudden appearance of a limiting factor, they are said to exhibit J-shaped growth form.

135. Which of these species represents S shaped growth curve?

a) Large predators

b) Aquatic species

c) Small mammals

d) Reptiles and birds

Explanation

Some populations, as in a population of small mammals, increase slowly at first then more rapidly and gradually slow down as environmental resistance increases whereby equilibrium is reached and maintained. Their growth is represented by S shaped growth curve.

136. Which of these contribute to the environmental resistance?

a) Environmental limiting factors

b) Atmospheric variations

c) Biotic and abiotic factors

d) Both a and c

Explanation

Environmental resistance is the sum total of the environmental limiting factors, both biotic and abiotic which together act to prevent the biotic potential of an organism from being realized.

137. Which of these factors does not regulate the population of species?

a) Density dependent

b) Intrinsic factors

c) Common factors

d) Density Independent

Explanation

Population numbers remain static or fluctuate depending on environmental conditions. This is regulated by many factors which are

Density independent – Extrinsic factors

Density dependent – Intrinsic factors

138. For which of this livelihood process intra specific association is observed?

a) Territoriality

b) Breeding

c) Protection

d) All the above

Explanation

Intra specific association is observed for all livelihood processes like feeding, territoriality, breeding and protection.

139. What type of interaction occurs between the crocodile and bird?

a) Mutualism

b) Amensalism

c) Commensalism

d) Predation

140. Identify the Incorrect match.

A. Competition i) Direct inhibition of each species by other

B. Parasitism ii) Parasites are smaller than the host

C. Amensalism iii) Two hosts are not affected

D. Mutualism iv) Interaction favourable to both species

a) i only

b) ii only

c) iii only

d) iv only

Explanation

141. Choose the correct statements.

i) Competition refers to type of interaction only between members of different species.

ii) Limited availability of food, water, nesting space and other resources leads to competition.

a) i only

b) ii only

c) Both i and ii

d) Neither i nor ii

Explanation

Competition refers to the type of interaction in which individuals of a species or members of different species vie for limited availability of food, water, nesting space, cover, mates or other resources.

142. Which of the species are forced to challenges as per the competitive exclusion principle of Hardin?

a) Most aggressive

b) Strongest species

c) Least adapted

d) Low density

Explanation

When inadequate to satisfy the need of the organisms seeking them, the weakest, least adapted, or least aggressive individuals are often

forced to face challenges. This phenomenon is known as the competitive exclusion principle of Hardin.

143. What is the reason for the competition between individuals of the same species?

a) Survival of the fittest.

b) Climatic conditions and the soil temperature variation

c) Identical requirements for food, mates and others.

d) To maintain environmental balance.

Explanation

Competition is usually keen between individuals of the same species (intra specific competition) because they have identical requirements for food, mates, and so on.

144. Which of these decides the severity of competition between different species?

a) Extent of similarity

b) Shortage of supply

c) Overlap in requirements

d) All the above

Explanation

Inter specific competition occurs where different species require at least some resource materials or conditions in common. The severity of competition depends on the extent of similarity or overlap in the requirements of different individuals and the shortage of the supply in the habitat as birds compete with squirrels for nuts, and seeds; insects and ungulates compete for food in grasslands.

12th Science Lesson 18 Questions in English

18] Immunology

1. The overall ability of body to fight against the disease-causing pathogen is called______

  1. Immunity
  2. Pathogenicity
  3. Pathology
  4. Immunology

Explanation

The overall ability of body to fight against the disease-causing pathogen is called immunity. It is also called disease resistance and the lack of immunity is known as susceptibility.

2. ______ is the study of immune system.

  1. Pathology
  2. Immunology
  3. Entomology
  4. Virology

Explanation

Immunology is the study of immune system. This system protects an individual from various infective agents. It refers to all the mechanisms used by the body for protection from environmental agents that are foreign to the body.

3. Assertion(A): Immune reactions are destructive in nature

Reason(R): Normally many of the responses of the immune system initiate the destruction and

elimination of invading organisms

  1. Both (A) and (R) are correct, but (R) does not explain (A)
  2. Both (A) and (R) are wrong
  3. Both (A) and (R) are correct and (R) explains (A)
  4. (A) is Correct and (R) is wrong

Explanation

Normally many of the responses of the immune system initiate the destruction and elimination of invading organisms and any toxic molecules produced by them. These immune reactions are destructive in nature and are made in response only to molecules that are foreign to the host and not to those of host itself.

4. Which of the following statement is correct?

  1. The ability to distinguish foreign molecules from self is another fundamental feature of the immune system
  2. Occasionally, it fails to make its distinction and reacts destructively against the host’s own molecules
  3. Auto-immune diseases can be fatal to the organism
  4. 1, 2
  5. 1, 3
  6. 2, 3
  7. All the above

Explanation

The ability to distinguish foreign molecules from self is another fundamental feature of the immune system. However, occasionally, it fails to make its distinction and reacts destructively against the host’s own molecules; such autoimmune diseases can be fatal to the organism.

5. Which of the following substances are foreign to recipient organism can induce immune response?

  1. Protein
  2. Nucleic acids
  3. Polysaccharides
  4. 1, 2
  5. 1, 3
  6. 2, 3
  7. All the above

Explanation

Almost all the macromolecules e.g. proteins, polysaccharides, nucleic acids, etc., as long as they are foreign to recipient organism can induce immune response.

6. Any substance capable of eliciting immune response is called_______

  1. Antibody
  2. Antigen
  3. Antisen
  4. Immunity

Explanation

Any substance capable of eliciting immune response is called an ANTIGEN (ANTIbody GENerator). There are two broad classes of immunity responses namely, innate immunity and acquired immunity.

7. Match the following

  1. Active Adaptive Natural Immunity 1. antibody transfer
  2. Passive Adaptive Natural Immunity 2. immunization
  3. Active Adaptive Artificial Immunity 3. infection
  4. Passive Adaptive Artificial Immunity 4. Maternal
  5. 2, 1, 3, 4
  6. 4, 1, 2, 3
  7. 3, 4, 2, 1
  8. 3, 2, 1, 4

Explanation

8. Which of the following statement is correct?

  1. Innate immunity is the natural phenomenon of resistance
  2. An individual possesses right from the birth
  3. 1 alone
  4. 2 alone
  5. 1, 2
  6. None

Explanation

Innate immunity is the natural phenomenon of resistance to infection which an individual possesses right from the birth.

9. What is the pH range of skin?

  1. 3 – 5
  2. 2 – 4
  3. 1 – 3
  4. 8 – 9

Explanation

Skin prevents the entry of microbes. Its acidic environment (pH 3-5) retards the growth of microbes. Mucus entraps foreign microorganisms and competes with microbes for attachment.

10. Which of the following statement is correct?

  1. Acidity of gastric secretions (HCl) kills most ingested microbes.
  2. Normal body temperature allows the growth of pathogens
  3. 1 alone
  4. 2 alone
  5. 1, 2
  6. None

Explanation

Normal body temperature inhibits the growth of pathogens. Fever also inhibits the growth of pathogens. Acidity of gastric secretions (HCl) kills most ingested microbes.

11. _____ acts as antibacterial agent which cleaves the bacterial cell wall

  1. Leucocytes
  2. Lysozyme
  3. Phagocytes
  4. Monocyte

Explanation

Lysozyme acts as antibacterial agent and cleaves the bacterial cell wall. Interferons induce antiviral state in the uninfected cells. Complementary substances produced from leucocytes lyse the pathogenic microbes or facilitate phagocytosis.

12. Which of the following can digest whole micro-organisms?

  1. Monocytes
  2. Neutrophils
  3. RBC
  4. Phagocytose
  5. 1, 2
  6. 1, 3, 4
  7. 2, 3, 4
  8. 1, 2, 4

Explanation

Specialized cells (Monocytes, neutrophils, tissue macrophages) phagocytose, and digest whole microorganisms.

13. Which of the following are chemotactic signals?

  1. Cerebrospinal fluid
  2. Serotonin
  3. Histamine
  4. Prostaglandins
  5. 1, 2, 3
  6. 1, 3, 4
  7. 2, 3, 4
  8. All the above

Explanation

Tissue damage and infection induce leakage of vascular fluid, containing chemotactic signals like serotonin, histamine and prostaglandins. They influx the phagocytic cells into the affected area. This phenomenon is called diapedesis.

14. Which of the following statement is correct?

  1. Innate immunity is specific in their action
  2. They are effective against a wide range of potentially infectious agents
  3. It is also known as natural immunity
  4. 1, 2
  5. 1, 3
  6. 2, 3
  7. All the above

Explanation

The innate defence mechanisms are non-specific in the sense that they are effective against a wide range of potentially infectious agents. It is otherwise known as non-specific immunity or natural immunity.

15. Which of the following statement is correct?

  1. The immunity that an individual acquires after birth is known as acquired immunity
  2. It is the body’s resistance to wide range of pathogens
  3. The unique features of acquired immunity are antigenic specificity, diversity, recognition of self and non-self and immunological memory
  4. 1, 2
  5. 1, 3
  6. 2, 3
  7. All the above

Explanation

The immunity that an individual acquires after birth is known as acquired immunity. It is the body’s resistance to a specific pathogen. The unique features of acquired immunity are antigenic specificity, diversity, recognition of self and non-self and immunological memory.

16. How many components does acquired immunity have?

  1. 3
  2. 4
  3. 1
  4. 2

Explanation

Acquired immunity has two components – cell mediated immunity (CMI) and antibody mediated immunity or humoral immunity.

17. Which of the following cells bring the cell mediated immunity?

  1. T cells
  2. Macrophages
  3. Natural killer cells
  4. 1, 2
  5. 1, 3
  6. 2, 3
  7. All the above

Explanation

When pathogens are destroyed by cells without producing antibodies, then it is known as cell mediated immune response or cell mediated immunity. This is brought about by T cells, macrophages and natural killer cells.

18. Which of the following statement is correct?

  1. When pathogens are destroyed by the production of antibodies, then it is known as antibody mediated immunity
  2. This is brought about by B cells with the help of antigen presenting cells and T helper cells.
  3. Antibody production is the characteristic feature of vertebrates only
  4. 1, 2
  5. 1, 3
  6. 2, 3
  7. All the above

Explanation

When pathogens are destroyed by the production of antibodies, then it is known as antibody mediated or humoral immunity. This is brought about by B cells with the help of antigen presenting cells and T helper cells. Antibody production is the characteristic feature of vertebrates only.

19. Which of the following statement about Acquired immunity is correct?

  1. Acquired immunity may be active immunity or passive immunity
  2. The immunological resistance developed by the organisms through the production of antibodies in their body is called active immunity
  3. Active acquired immunity is innate immunity
  4. 1, 2
  5. 1, 3
  6. 2, 3
  7. All the above

Explanation

Acquired immunity may be active immunity or passive immunity. The immunological resistance developed by the organisms through the production of antibodies in their body is called active immunity. Active immunity is acquired through the use of a person’s immune responses, which lead to the development of memory cells. Active immunity results from an infection or an immunization

20. Which of the following statement about Passive Acquired immunity is incorrect?

  1. Person’s immune response has the memory of passive immunity
  2. Passive immunity does not require the body to produce antibodies to antigens
  3. 1 alone
  4. 2 alone
  5. 1, 2
  6. None

Explanation

Passive immunity does not require the body to produce antibodies to antigens. The antibodies are introduced from outside into the organism. Thus, passive immunity is acquired without the activation of a person’s immune response, and therefore there is no memory

21. The process of production of blood cells in the bone marrow is called_____

  1. Haematopoiesis
  2. Haematosis
  3. Haematoporiesis
  4. Haemorrage

Explanation

The process of production of blood cells in the bone marrow which is present inside the bone is called as haematopoiesis.

22. Which of the following statement is correct about Active immunity?

  1. Active immunity is produced actively by host’s immune system.
  2. It is durable and effective in protection.
  3. Immunological memory is present.
  4. 1, 2
  5. 1, 3
  6. 2, 3
  7. All the above

Explanation

23. Which of the following statement is correct about passive immunity?

  1. Passive immunity is received passively and there is active host participation
  2. It is transient and less effective
  3. Immunity develops immediately
  4. 1, 2
  5. 1, 3
  6. 2, 3
  7. All the above

Explanation

24. The primary immune response is_______

  1. Slow and long lived
  2. Slow and short lived
  3. Fast and short lived
  4. Fast and long lived

Explanation

The primary immune response occurs when a pathogen comes in contact with the immune system for the first time. The primary immune response is slow and short-lived.

25. Which of the following take place during Primary immune response?

  1. Immune system learns to recognize the Antigen
  2. Immune system produces Antibody against Antibody
  3. Produces memory Lymphocytes
  4. 1, 2
  5. 1, 3
  6. 2, 3
  7. All the above

Explanation

During primary immune response, the immune system has to learn to recognize the antigen, produce antibody against it and eventually produce memory lymphocytes.

26. The secondary immune response occurs when a person is exposed to______

  1. Same antigen
  2. Different antigen
  3. Same antibody
  4. Different antibody

Explanation

The secondary immune response occurs when a person is exposed to the same antigen again. During this time, immunological memory has been established and the immune system can start producing antibodies immediately.

27. Which of the following is called booster immune response?

  1. Primary immune response
  2. Secondary immune response
  3. Both a and b
  4. None

Explanation

During Secondary immune response, within 2 to 3 days, the antibody concentration in the blood rises steeply to reach much higher level than primary response. This is also called as “booster response”.

28. Which of the following statement is/are correct about Lymphoid organs?

  1. These organs are involved in origin, maturation and proliferation of lymphocytes
  2. Immune system of an organism consists of several structurally and functionally different organs and tissues that are widely dispersed in the body, Lymphoid organs are one of these.
  3. 1 alone
  4. 2 alone
  5. 1, 2
  6. None

Explanation

Immune system of an organism consists of several structurally and functionally different organs and tissues that are widely dispersed in the body. The organs involved in the origin, maturation and proliferation of lymphocytes are called lymphoid organs.

29. In how many days Antibody level reaches peak level during primary immune response?

  1. 5 to 6 days
  2. 3 to 4 days
  3. 7 to 10 days
  4. 15 to 17 days

Explanation

In Primary Immune Response, Antibody level reaches peak in 7 to 10 days. It occurs as a result of primary contact with an antigen.

30. Which of the following take place during primary immune response?

  1. Prolonged period is required to establish
  2. There is rapid increase in Antibody level
  3. Primary immune response appears mainly in the lymph nodes and spleen
  4. 1, 2
  5. 1, 3
  6. 2, 3
  7. All the above

Explanation

Primary Immune Response appears mainly in the lymph nodes and spleen. There is rapid decline in antibody level. Prolonged period is required to establish immunity.

31. During Secondary Immune Response, Antibody level reaches peak in_______

  1. 5 to 6 days
  2. 3 to 5 days
  3. 7 to 10 days
  4. 15 to 17 days

Explanation

Secondary Immune Response occurs as a result of second and subsequent contacts with the same antigen. Antibody level reaches peak in 3 to 5 days.

32. Which of the following take place during secondary immune response?

  1. It establishes immunity in a short time.
  2. Antibody level remains high for longer period.
  3. It appears mainly in the bone marrow, followed by the spleen and lymph nodes
  4. 1, 2
  5. 1, 3
  6. 2, 3
  7. All the above

Explanation

Secondary Immune Response establishes immunity in a short time. Antibody level remains high for longer period. It appears mainly in the bone marrow, followed by the spleen and lymph nodes.

33. Which of the following are primary Lymphoid organs?

  1. Spleen
  2. Thymus
  3. Bone marrow
  4. 1, 2
  5. 1, 3
  6. 2, 3
  7. All the above

Explanation

34. _______ organs provide appropriate environment for lymphocytic maturation

  1. Primary lymphoid
  2. Secondary lymphoid
  3. Peripheral lymphoid
  4. None

Explanation

Based on their functions, they are classified into primary or central lymphoid organs and secondary or peripheral lymphoid organs. The primary lymphoid organs provide appropriate environment for lymphocytic maturation.

35. _____ is a primary lymphoid organ of birds

  1. Spleen
  2. Liver
  3. Thymus
  4. Bursa of Fabricius

Explanation

Immune response of birds and animals are different from each other. Bursa of Fabricius is a primary lymphoid organ of birds.

36. Which of the following mature in Bursa of birds?

  1. T – Lymphocytes
  2. B – Lymphocytes
  3. RBC
  4. WBC

Explanation

Bursa of Fabricius is attached to the dorsal side of the cloaca. B lymphocytes mature in the bursa and bring about humoral immunity.

37. Which of the following are matched correctly?

  1. Bursa of Fabricius – birds
  2. thymus gland – mammals
  3. 1 alone
  4. 2 alone
  5. 1, 2
  6. None

Explanation

Bursa of Fabricius of birds, bone marrow and thymus gland of mammals constitute the primary lymphoid organs involved in the production and early selection of lymphocytes. These lymphocytes become dedicated to a particular antigenic specificity.

38. Where does the mammal T cell maturation take place?

  1. Thymus
  2. Bone marrow
  3. Spleen
  4. Lymph node

Explanation

Only when the lymphocytes mature in the primary Lymphoidal organs, they become immunocompetent cells. In mammals, B cell maturation occurs in the bone marrow and T cells maturation occurs in the thymus.

39. How many lobes does Thymus have?

  1. 3
  2. 2
  3. 5
  4. 6

Explanation

The thymus is a flat and bilobed organ located behind the Sternum, above the heart. Each lobe of the thymus contains numerous lobules, separated from each other by connective tissue called septa.

40. Inner compartment of thymus lobule is filled with____ and outer cortex is filled with_____

  1. Mature thymocytes, Immature thymocytes
  2. Immature thymocytes, Mature thymocytes
  3. Mature thymocytes, Mature thymocytes
  4. Immature thymocytes, Immature thymocytes

Explanation

Each lobule is differentiated into two compartments, the outer compartment or outer cortex, is densely packed with immature T cells called thymocytes, whereas the inner compartment or medulla is sparsely populated with mature thymocytes.

41. What is the main secretion of thymus?

  1. Trypsin
  2. Thymosin
  3. Theonine
  4. Phagotysin

Explanation

One of thymus main secretions is the hormone thymosin. It stimulates the T cell to become mature and immunocompetent.

42. During which stage thymus undergoes Atrophy?

  1. Adult
  2. Childhood
  3. Early teens
  4. Elderly

Explanation

By the early teens, the thymus gland begins to atrophy and is replaced by adipose tissue. (Atrophy – reduction in size of cell, organ or tissue after attaining its normal mature growth)

43. Thymus is most active during______

  1. Neonatal
  2. Pre- Adolescent
  3. Adolescent
  4. 1, 2
  5. 1, 3
  6. 2 alone
  7. 1 alone

Explanation

Thymus gland is most active during the neonatal and pre-adolescent periods. Thymus produce Thymosin hormone.

44. Bone marrow contains stem cells known as_____

  1. Haematopoietic cells
  2. Haematopoiesios cells
  3. Lymphoidal cells
  4. Leucocyte

Explanation

Bone marrow is a lymphoid tissue found within the spongy portion of the bone. Bone marrow contains stem cells known as haematopoietic cells. These cells have the potential to multiply through cell division and either remain as stem cells or differentiate and mature into different kinds of blood cells.

45. In secondary or peripheral lymphoid organs, antigen is______

  1. Generalized
  2. Localized
  3. Localized to specific area
  4. None

Explanation

In secondary or peripheral lymphoid organs, antigen is localized so that it can be effectively exposed to mature lymphocytes.

46. Which of the following are examples of lymph nodes?

  1. Spleen
  2. Tonsils
  3. GALT
  4. BALT
  5. 1, 2, 4
  6. 2, 3, 4
  7. 1, 3, 4
  8. All the above

Explanation

The best examples are lymph nodes, appendix, Peyer’s patches of gastrointestinal tract, tonsils, adenoids, spleen, MALT (Mucosal-Associated Lymphoid Tissue), GALT (Gut-Associated Lymphoid Tissue), BALT (Bronchial/Tracheal-Associated Lymphoid Tissue).

47. Peyer’s patches are found in________

  1. Stomach
  2. Small intestine
  3. Large intestine
  4. Pancreas

Explanation

Peyer’s patches are oval-shaped areas of thickened tissue that are embedded in the mucus-secreting lining of the small intestine of humans and other vertebrate animals.

48. Which of the following immune cells are found in Peyer’s patches?

  1. Dendritic cells
  2. Macrophages
  3. T cells
  4. B cells
  5. 1, 2, 3
  6. 2, 3, 4
  7. 1, 3, 4
  8. All the above

Explanation

Peyer’s patches contain a variety of immune cells, including macrophages, dendritic cells, T cells, and B cells.

49. Tonsils are found at the back of_____

  1. Throat
  2. Testes or Ovaries
  3. Spleen
  4. Pancreas

Explanation

The tonsils (palatine tonsils) are a pair of soft tissue masses located at the back of the throat (pharynx).

50. Which of the following statement is correct?

  1. The tonsils are part of the lymphatic system, which help to fight infections
  2. They stop invading germs including bacteria and viruses
  3. 1 alone
  4. 2 alone
  5. 1, 2
  6. None

Explanation

The tonsils are part of the lymphatic system, which help to fight infections. They stop invading germs including bacteria and viruses.

51. Which of the following statement is correct?

  1. Spleen is a secondary lymphoid organ located in the upper part of the abdominal cavity close to the diaphragm
  2. Spleen contains T cells alone
  3. It brings humoral and cell mediated immunity
  4. 1, 2
  5. 1, 3
  6. 2, 3
  7. All the above

Explanation

Spleen is a secondary lymphoid organ located in the upper part of the abdominal cavity close to the diaphragm. Spleen contains B and T cells. It brings humoral and cell mediated immunity.

52. The adenoids are glands located in________

  1. Abdomen
  2. Mouth
  3. Neck
  4. Chest

Explanation

The adenoids are glands located in the roof of the mouth, behind the soft palate where the nose connects to the throat.

53. Which of the following statement is correct?

  1. The adenoids produce antibodies that help to fight infections.
  2. Adenoids shrink during adolescence and may disappear by adulthood
  3. 1 alone
  4. 2 alone
  5. 1, 2
  6. None

Explanation

The adenoids produce antibodies that help to fight infections. Typically, the adenoids shrink during adolescence and may disappear by adulthood.

54. What is the shape of Lymph node?

  1. Pea shaped
  2. Bean shaped
  3. Oval shaped
  4. Spherical shaped

Explanation

Lymph node is a small bean-shaped structure and is part of the body’s immune system. Lymph nodes are present all over the body.

55. ______ is the first one to encounter the antigen that enters the tissue spaces

  1. Spleen
  2. Lymph nodes
  3. Pancreas
  4. Kidney

Explanation

Lymph node is the first one to encounter the antigen that enters the tissue spaces. Lymph nodes filter and trap substances that travel through the lymphatic fluid.

56. Lymph nodes are packed with____

  1. Lymphocytes
  2. Macrophages
  3. Monocytes
  4. 1, 2
  5. 1, 3
  6. 2, 3
  7. All the above

Explanation

Lymph node are packed tightly with white blood cells, namely lymphocytes and macrophages. There are hundreds of lymph nodes found throughout the body. They are connected to one another by lymph vessels.

57. Which of the following are features of Lymph?

  1. Clear
  2. Transparent
  3. Mobile
  4. Colourless
  5. 1, 2, 4
  6. 2, 3, 4
  7. 1, 3, 4
  8. All the above

Explanation

Lymph is a clear, transparent, colourless, mobile and extracellular fluid connective tissue. As the lymph percolates through the lymph node, the particulate antigen brought in by the lymph will be trapped by the phagocytic cells, follicular and interdigitating dendritic cells.

58. How many zones does Lymph node has?

  1. 2
  2. 3
  3. 4
  4. 5

Explanation

Lymph node has three zones. They are divided into cortex, paracortex and medulla.

59. Which of the following statement is correct?

  1. The outer most layer of the lymph node is called cortex, which consists of B-lymphocytes, macrophages, and follicular dendritic cells.
  2. The paracortex zone is beneath the cortex, which is richly populated by T lymphocytes and interdigitating dendritic cells
  3. The inner most zone is called the medulla which is sparsely populated by lymphocytes, but many of them are plasma cells, which actively secrete antibody molecules
  4. 1, 2
  5. 1, 3
  6. 2, 3
  7. All the above

Explanation

The outer most layer of the lymph node is called cortex, which consists of B-lymphocytes, macrophages, and follicular dendritic cells. The paracortex zone is beneath the cortex, which is richly populated by T lymphocytes and interdigitating dendritic cells. The inner most zone is called the medulla which is sparsely populated by lymphocytes, but many of them are plasma cells, which actively secrete antibody molecules.

60. MALT is a diffuse system of small concentrations of lymphoid tissue in_____ tract

  1. Respiratory
  2. Alimentary
  3. Urino – genital
  4. 1, 2
  5. 1, 3
  6. 2, 3
  7. All the above

Explanation

The mucosa-associated lymphoid tissue (MALT) is a diffuse system of small concentrations of lymphoid tissue in the alimentary, respiratory and urino-genital tracts.

61. Which of the following possesses IgA antibody?

  1. MALT
  2. GALT
  3. BALT
  4. None

Explanation

MALT is populated by lymphocytes such as T and B cells, as well as plasma cells and macrophages, each of which is well situated to encounter antigens passing through the mucosal epithelium. It also possesses IgA antibodies.

62. Which of the following statement is correct?

  1. GALT is a component of the mucosa associated lymphoid tissue (MALT)
  2. It works in the immune system to protect the body from invasion in the gut
  3. 1 alone
  4. 2 alone
  5. 1, 2
  6. None

Explanation

Gut-associated lymphoid tissue (GALT) is a component of the mucosa associated lymphoid tissue (MALT) which works in the immune system to protect the body from invasion in the gut.

63. BALT is found in_____

  1. Respiratory mucosae
  2. Alimentary mucosae
  3. Gastric mucosae
  4. None

Explanation

Bronchus Associated Lymphoid Tissues (BALT) also a component of MALT is made of lymphoid tissue (tonsils, lymph nodes, lymph follicles) is found in the respiratory mucosae from the nasal cavities to the lungs.

64. The most frequently enlarged lymph nodes are found in______

  1. Chin
  2. Neck
  3. Armpits
  4. Groin
  5. 1, 2
  6. 1, 3, 4
  7. 2, 3, 4
  8. All the above

Explanation

Sometimes visible swelling of lymph nodes occurs due to active immune response and increased concentration of lymphocytes. Thus, swollen lymph nodes may signal an infection. There are several groups of lymph nodes. The most frequently enlarged lymph nodes are found in the neck, under the chin, in the armpits and in the groin.

65. Which of the following statement is correct?

  1. The immune system is composed of many interdependent cells
  2. They protect the body from microbial infections and the growth of tumour cells.
  3. 1 alone
  4. 2 alone
  5. 1, 2
  6. None

Explanation

The immune system is composed of many interdependent cells that protect the body from microbial infections and the growth of tumour cells.

66. Match the following

  1. Lymphocytes 1. 40-500
  2. Monocytes 2. 1500 – 4000
  3. Neutrophils 3. 200 – 950
  4. Eosinophils 4. 2000-7000
  5. 2, 1, 4, 3
  6. 2, 3, 4, 1
  7. 1, 3, 2, 4
  8. 3, 1, 2, 4

Explanation

67. Which of the following cells can be produced from stem cells?

  1. RBC
  2. WBC
  3. Platelets
  4. 1, 2
  5. 1, 3
  6. 2, 3
  7. All the above

Explanation

All Blood cells are derived from pluripotent haematopoetic stem cells. Each stem cell has the capacity to produce RBC, WBC and platelets.

68. Which of the following statemen is correct?

  1. The only cells capable of specifically recognising and producing an immune response are the lymphocytes
  2. The other types of white blood cells play an important role in non specific immune response, antigen presentation and cytokine production
  3. 1 alone
  4. 2 alone
  5. 1, 2
  6. None

Explanation

The only cells capable of specifically recognising and producing an immune response are the lymphocytes. The other types of white blood cells play an important role in non specific immune response, antigen presentation and cytokine production.

69. _____ % of the white blood cells are lymphocytes

  1. 40 – 50
  2. 30 – 40
  3. 20 – 30
  4. 10 – 20

Explanation

About 20-30% of the white blood cells are lymphocytes. They have a large nucleus filling most of the cell, surrounded by a little cytoplasm.

70. Which of the following statement is correct?

  1. The two main types of lymphocytes are B and T lymphocytes
  2. B lymphocytes are produced in the bone marrow.
  3. B lymphocytes (B cells) stay in the bone marrow until they are mature
  4. 1, 2
  5. 1, 3
  6. 2, 3
  7. All the above

Explanation

The two main types of lymphocytes are B and T lymphocytes. Both these are produced in the bone marrow. B lymphocytes (B cells) stay in the bone marrow until they are mature. Then they circulate around the body.

71. Which of the following statement is correct?

  1. Lymphocytes have receptor proteins on their surface
  2. When receptors on a B cell bind with an antigen, the B cell becomes activated and divides rapidly to produce plasma cells.
  3. The Plasma cell produces Antibodies
  4. 1, 2
  5. 1, 3
  6. 2, 3
  7. All the above

Explanation

Lymphocytes have receptor proteins on their surface. When receptors on a B cell bind with an antigen, the B cell becomes activated and divides rapidly to produce plasma cells. The plasma cells produce antibodies.

72. Which cell become memory cells?

  1. B – cells
  2. T – cells
  3. Monocytes
  4. Leucocytes

Explanation

B cells do not produce antibodies but become memory cells. These cells are responsible for secondary immune response.

73. Which of the following statement is correct?

  1. T lymphocytes produces antibodies
  2. The two important types of T cells are Helper T cells and Killer T cells
  3. Helper T cells release a chemical called cytokine which activates B cells
  4. 1, 2
  5. 1, 3
  6. 2, 3
  7. All the above

Explanation

T lymphocytes do not produce antibodies. They recognize antigen presenting cells and destroy them. The two important types of T cells are Helper T cells and Killer T cells. Helper T cells release a chemical called cytokine which activates B cells. Killer cells move around the body and destroy cells which are damaged or infected.

74. ________ destroy foreign cells by phagocytosis

  1. B cells
  2. T cells
  3. Monocytes and neutrophils
  4. All the above

Explanation

Apart from these cells neutrophils and monocytes destroy foreign cells by phagocytosis. Monocytes when they mature into large cells, they are called macrophages which perform phagocytosis on any foreign organism.

75. Which of the following statement about Dendritic cell is correct?

  1. Dendritic cells are called so because its covered with long, thin membrane extensions that resemble dendrites of nerve cells
  2. These cells present the antigen to T-helper cells
  3. 1 alone
  4. 2 alone
  5. 1, 2
  6. None

Explanation

Dendritic cells are called so because its covered with long, thin membrane extensions that resemble dendrites of nerve cells. These cells present the antigen to T-helper cells.

76. How many types of Dendritic cells are known?

  1. 2
  2. 3
  3. 4
  4. 5

Explanation

Four types of dendritic cells are known. They are Langerhans, interstitial cells, myeloid and lymphoid cells.

77. Which of the following statement is correct?

  1. The term antigen (Ag) is used in two senses
  2. The first to describe a molecule which generates an immune response
  3. The second, a molecule which reacts with antibodies.
  4. 1, 2
  5. 1, 3
  6. 2, 3
  7. All the above

Explanation

The term antigen (Ag) is used in two senses, the first to describe a molecule which generates an immune response and the second, a molecule which reacts with antibodies. In general antigens are large, complex molecular substances that can induce a detectable immune response.

78. Which of the following statement is incorrect?

  1. An antigen is a substance that is specific to an antibody or a T-cell receptor and is often used as a synonym for immunogen
  2. The histocompatibility antigens are cell surface antigens that induce an immune response leading to rejection of allografts
  3. 1 alone
  4. 2 alone
  5. 1, 2
  6. None

Explanation

An antigen is a substance that is specific to an antibody or a T-cell receptor and is often used as a synonym for immunogen. The histocompatibility antigens are cell surface antigens that induce an immune response leading to rejection of allografts.

79. ______ are substance that are non-immunogenic but can react with the products of a specific

immune response

  1. Immunogen
  2. Haptens
  3. Epitope
  4. Adjuvants

Explanation

An immunogen is a substance capable of initiating an immune response. Haptens are substance that are non-immunogenic but can react with the products of a specific immune response.

80. Substances that can enhance the immune response to an antigen are called_____

  1. Immunogen
  2. Haptens
  3. Epitope
  4. Adjuvants

Explanation

Substances that can enhance the immune response to an antigen are called adjuvants. Epitope is an antigenic determinant and is the active part of an antigen.

81. Which of the following statement is correct?

  1. A paratope is the antigen – binding site and is a part of an antibody which recognizes and binds to an antigen
  2. Antigenicity is the property of a substance (antigen) that allows it to react with the products of the specific immune response
  3. 1 alone
  4. 2 alone
  5. 1, 2
  6. None

Explanation

A paratope is the antigen – binding site and is a part of an antibody which recognizes and binds to an antigen. Antigenicity is the property of a substance (antigen) that allows it to react with the products of the specific immune response.

82. On which basis antigens are classified into exogenous and endogenous type?

  1. Origin
  2. Cell wall
  3. Mode of Nutrition
  4. All the above

Explanation

On the basis of origin, antigens are classified into two types: exogenous antigens and endogenous antigens.

83. Which of the following are exogenous antigens?

  1. Micro-organisms
  2. Pollens
  3. Drugs
  4. 1, 2
  5. 1, 3
  6. 2, 3
  7. All the above

Explanation

The antigens which enter the host from the outside in the form of microorganisms, pollens, drugs, or pollutants are called exogenous antigens. The antigens which are formed within the individual are endogenous antigens. The best examples are blood group antigens.

84. Which of the following are non- covalent in nature?

  1. Hydrogen bond
  2. Electrostatic bond
  3. Hydrophobic bond
  4. 1, 2
  5. 1, 3
  6. 2, 3
  7. All the above

Explanation

The bonds that hold the antigen to the antibody combining site are all non-covalent in nature. These include hydrogen bonds, electrostatic bonds, Van der Waals forces and hydrophobic bonds. Antibody affinity is the strength of the reaction between a single antigenic determinant and a single combining site on the antibody.

85. What is the chief application of antigen – antibody reactions?

  1. Determine blood group
  2. Study serological ascertainment of exposure to infectious agents
  3. To develop immunoassays for the quantification of various substances
  4. All the above

Explanation

The chief application of antigen – antibody reactions are to determine blood groups for transfusion, to study serological ascertainment of exposure to infectious agents, to develop immunoassays for the quantification of various substances, to detect the presence or absence of protein in serum and to determine the characteristics of certain immunodeficiency diseases.

86. Assertion(A): Antibodies that bring about precipitate formation on reacting with antigens are

called as precipitins

Reason(R): The reaction between soluble antigen and antibody leads to visible precipitate

formation, which is called precipitin reaction

  1. Both (A) and (R) are correct, but (R) does not explain (A)
  2. Both (A) and (R) are wrong
  3. Both (A) and (R) are correct and (R) explains (A)
  4. (A) is Correct and (R) is wrong

Explanation

The reaction between soluble antigen and antibody leads to visible precipitate formation, which is called precipitin reaction. Antibodies that bring about precipitate formation on reacting with antigens are called as precipitins. Whenever a particulate antigen interacts with its antibody, it would result in clumping or agglutination of the particulate antigen, which is called agglutination reaction. The antibody involved in bringing about agglutination reaction is called agglutinin.

87. Match the following

  1. Neutralization 1. Soluble antigen
  2. Agglutination 2. Exotoxin
  3. Precipitation 3. Particulate antigen
  4. 3, 1, 2
  5. 1, 3, 2
  6. 2, 3, 1
  7. 2, 1, 3

Explanation

88. Which of the following statement is correct?

  1. A vaccine is a biological preparation that provides active acquired immunity to a particular disease
  2. Vaccines “teach” our body how to defend itself when viruses or bacteria, invade it
  3. Vaccines deliver only very little amounts of inactivated or weakened viruses or bacteria, or parts of them
  4. 1, 2
  5. 1, 3
  6. 2, 3
  7. All the above

Explanation

A vaccine is a biological preparation that provides active acquired immunity to a particular disease and resembles a disease-causing microorganism and is often made from weakened or attenuated or killed forms of the microbes, their toxins, or one of its surface proteins. Vaccines “teach” our body how to defend itself when viruses or bacteria, invade it. Vaccines deliver only very little amounts of inactivated or weakened viruses or bacteria, or parts of them. This allows the immune system to recognize the organism without actually experiencing the disease. Some vaccines need to be given more than once (i.e., a ‘booster’ vaccination) to make sure the immune system can overcome a real infection in the future.

89. Which of the following statement is correct?

  1. First generation vaccine is further subdivided into live attenuated vaccine, killed vaccine and toxoids
  2. Measles, mumps and rubella (MMR) vaccine and the Varicella (chickenpox) vaccine, Killed (inactivated) vaccine are 1st generation vaccines
  3. They make us immune to the harmful effects of the infection, instead of to the infection itself
  4. 1, 2
  5. 1, 3
  6. 2, 3
  7. All the above

Explanation

First generation vaccine is further subdivided into live attenuated vaccine, killed vaccine and toxoids (Fig. 8.9). Live attenuated vaccines use the weakened (attenuated), aged, less virulent form of the virus. E.g. Measles, mumps and rubella (MMR) vaccine and the Varicella (chickenpox) vaccine, Killed (inactivated) vaccines are killed or inactivated by heat and other methods. E.g. Salk’s polio vaccine. Toxoid vaccines contain a toxin or chemical secreted by the bacteria or virus. They make us immune to the harmful effects of the infection, instead of to the infection itself. E.g. DPT vaccine (Diphtheria, Pertussis and Tetanus).

90. The latest revolution in vaccine is______

  1. RNA vaccine
  2. Mitochondrial vaccine
  3. DNA vaccine
  4. All the above

Explanation

Second generation vaccine contains the pure surface antigen of the pathogen. E.g Hepatitis-B vaccine. Third generation vaccine contains the purest and the highest potency vaccines which are synthetic in generation. The latest revolution in vaccine is DNA vaccine or recombinant vaccine.

91. Who prepared the 1st vaccine?

  1. Edward Lewis
  2. Edward Jenner
  3. Dr. Jonas Salk
  4. Lewis Pasteur

Explanation

Vaccino therapy is the method of use of vaccine for treatment of disease. Dr. Edward Jenner prepared first vaccine for small pox in 1796.

92. Polio vaccine (vaccine consists of inactivated microorganism) was developed by_______

  1. Edward Lewis
  2. Edward Jenner
  3. Dr. Jonas Salk
  4. Lewis Pasteur

Explanation

Polio vaccine was developed by Dr. Jonas Salk (vaccine consists of inactivated microorganism) and Dr. Albert Sabin (live attenuated oral polio vaccine).

93. BCG vaccine was developed by______

  1. Calmette and Guerin
  2. Edward Jenner
  3. Louis Pasteur
  4. Albert Sabin

Explanation

Louis Pasteur (1885) discovered vaccine against rabies, anthrax and cholera. BCG vaccine was developed by Calmette and Guerin against tuberculosis in France in the year 1908.

94. Which of the following statement is correct?

  1. Vaccination is the process of administrating a vaccine into the body or the act of introducing a vaccine into the body to produce immunity to a specific disease
  2. Immunization is the process of the body building up immunity to a particular disease.
  3. Vaccines work by fighting the pathogen and then recording it in their memory system to ensure that the next time this pathogen enters the body, it is eliminated far quickly
  4. 1, 2
  5. 1, 3
  6. 2, 3
  7. All the above

Explanation

“Vaccination is the process of administrating a vaccine into the body or the act of introducing a vaccine into the body to produce immunity to a specific disease.” Immunization is the process of the body building up immunity to a particular disease. Immunization describes the actual changes in the body after receiving a vaccine. Vaccines work by fighting the pathogen and then recording it in their memory system to ensure that the next time this pathogen enters the body, it is eliminated far quickly. Once, the body is able to fight against the disease, it is believed to have built the immunity for it, also known as the body being immunized against the disease.

95. Which of the following statement is correct?

  1. Some of the individuals are very sensitive to some particles present in the environment.
  2. The exaggerated response of the immune system to certain antigens present in the environment is called allergy
  3. The substances to which such an immune response is produced are called allergens.
  4. 1, 2
  5. 1, 3
  6. 2, 3
  7. All the above

Explanation

Some of the individuals are very sensitive to some particles present in the environment. The exaggerated response of the immune system to certain antigens present in the environment is called allergy (allo-altered, erg-reaction). The substances to which such an immune response is produced are called allergens.

96. Allergy is a form of over active immune response mediated by___

  1. IgM
  2. IgE
  3. IgA
  4. IgG

Explanation

Allergy is a form of over active immune response mediated by IgE and mast cells. It can also be due to the release of chemicals like histamine and serotonin from the mast cells.

97. Which of the following statement is correct?

  1. Immunodeficiency results from the failure of one or more components of the immune system.
  2. Primary immune deficiencies are caused by genetic developmental defects. Secondary immune deficiencies arise due to various reasons like radiation, use of cytolytic and immunosuppressive drugs and infections.
  3. AIDS is an acronym for Acquired Immuno Deficiency Syndrome.
  4. 1, 2
  5. 1, 3
  6. 2, 3
  7. All the above

Explanation

Immunodeficiency results from the failure of one or more components of the immune system. Primary immune deficiencies are caused by genetic developmental defects. Secondary immune deficiencies arise due to various reasons like radiation, use of cytolytic and immunosuppressive drugs and infections. AIDS is an acronym for Acquired Immuno Deficiency Syndrome. It is the deficiency of immune system, acquired during the life time of an individual indicating that it is not a congenital disease.

98. Which cell is selectively infected by HIV?

  1. T cells
  2. B cells
  3. Neutrophils
  4. Monocytes

Explanation

AIDS is caused by Human Immuno- Deficiency Virus (HIV). It selectively infects helper T cells. The infected helper T cells will not stimulate antibody production by B-cells resulting in loss of natural defence against viral infection. On the basis of genetic characteristics and differences in the viral antigens, HIV is classified into the types 1 and 2 (HIV-1, HIV-2).

99. Which of the following statement is correct?

  1. The human immunodeficiency virus belongs to the genus Lentivirus.
  2. At the core, there are two large double stranded RNA.
  3. When observed under the electron microscope, HIV is seen as a spherical virus, 100-120 nm in diameter, containing a dense core surrounded by a lipoprotein envelope.
  4. 1, 2
  5. 1, 3
  6. 2, 3
  7. All the above

Explanation

The human immunodeficiency virus belongs to the genus Lentivirus. When observed under the electron microscope, HIV is seen as a spherical virus, 100-120 nm in diameter, containing a dense core surrounded by a lipoprotein envelope. The envelope has glycoprotein (gp) spikes termed gp 41 and gp 120. At the core, there are two large single stranded RNA.

100. HIV can survive for___ days inside a cell

  1. 2
  2. 3
  3. 1.5
  4. 2.5

Explanation

The HIV is often located within the cells especially in macrophages. HIV can survive for 1.5 days inside a cell but only about 6 hours outside a cell.

101. Which of the following are the routes of HIV transmission?

  1. Unsafe sexual contact
  2. Blood – contaminated needles
  3. Vertical transmission from HIV infected mother to child
  4. 1, 2
  5. 1, 3
  6. 2, 3
  7. All the above

Explanation

Routes of HIV transmission include unsafe sexual contact, blood-contaminated needles, organ transplants, blood transfusion and vertical transmission from HIV infected mother to child. HIV is not transmitted by insects or by casual contact.

102. Which of the following statement is correct?

  1. A simple blood test is available that can determine whether the person has been infected with HIV
  2. The ELISA test is confirmatory for HIV
  3. 1 alone
  4. 2 alone
  5. 1, 2
  6. None

Explanation

A simple blood test is available that can determine whether the person has been infected with HIV. The ELISA test (Enzyme Linked Immuno Sorbent Assay) detects the presence of HIV antibodies. It is a preliminary test. Western blot test is more reliable and a confirmatory test. It detects the viral core proteins. If both tests detect the presence of the antibodies, the person is considered to be HIV positive.

103. Which of the following statement is correct?

  1. Autoimmunity is due to an abnormal immune response in which the immune system fails to properly distinguish between self and non-self and attacks its own body.
  2. Our body produces antibodies (auto antibodies) and cytotoxic T cells that destroy our own tissues. If a disease-state results, it is referred to as auto-immune disease.
  3. 1 alone
  4. 2 alone
  5. 1, 2
  6. None

Explanation

Autoimmunity is due to an abnormal immune response in which the immune system fails to properly distinguish between self and non-self and attacks its own body. Our body produces antibodies (auto antibodies) and cytotoxic T cells that destroy our own tissues.

104. Which of the following statement is correct?

  1. A tumour or neoplasm is a group of cells whose growth has gone unchecked
  2. They spread to other parts of the body from the tumour and give rise to secondary tumour.
  3. Benign or non-cancerous tissues are capable of indefinite growth and do not invade other body parts.
  4. 1, 2
  5. 1, 3
  6. 2, 3
  7. All the above

Explanation

A tumour or neoplasm is a group of cells whose growth has gone unchecked. When a tumour continues to grow and invades healthy tissue, it is called cancer. They spread to other parts of the body from the tumour and give rise to secondary tumour. This is known as metastasis. Tumour may be benign or malignant depending on its characteristics. Benign or non-cancerous tissues are capable of indefinite growth and do not invade other body parts. In the malignant tumour, the cells grow indefinitely, detach and migrate into healthy surrounding tissues.

105. Which of the following are treatment for cancer?

  1. Radiotherapy
  2. Chemotherapy
  3. Immunotherapy
  4. 1, 2
  5. 1, 3
  6. 2, 3
  7. All the above

Explanation

Different approaches have been attempted in the immunotherapy of cancer. Immunotherapy appears to be important in getting rid of the residual malignant cells after the gross tumour has been removed. The best results in the treatment of cancer is to follow an integrated approach to therapy, combining surgery, radiotherapy, chemotherapy and immunotherapy.

106. Which of the following are the features of Cancer cells?

  1. Large, Variable shaped nuclei relatively small cytoplasmic volume
  2. Variation in cell size and shape
  3. Loss of normal specialized features
  4. 1, 2
  5. 1, 3
  6. 2, 3
  7. All the above

Explanation

12th Science Lesson 19 Questions in English

19] Microbes in Human Welfare

1. Which of the following are major components of biological system on earth?

  1. Viroids
  2. Algae
  3. Prions
  4. Fungi
  5. 1, 2, 4
  6. 1, 3, 4
  7. 2, 3, 4
  8. All the above

Explanation

Microbes such as bacteria, fungi, protozoa, certain algae, viruses, viroids and prions are some of the major components of the biological system on Earth.

2. Which of the following statement is incorrect?

  1. Several microorganisms are beneficial and contribute to human welfare.
  2. Microbes like bacteria and fungi can be grown on nutritive media to form colonies which can be visibly seen
  3. 1 alone
  4. 2 alone
  5. 1, 2
  6. None

Explanation

Microbes are present everywhere – in soil, water, air and within bodies of animals and plants. Microbes like bacteria and fungi can be grown on nutritive media to form colonies which can be visibly seen.

3. Which of the following are Lactic acid bacteria?

  1. Streptococcus lactis
  2. Lactobacillus acidophilus
  3. Lactobacillus lactis
  4. 1, 3
  5. 2, 3
  6. 1, 2
  7. All the above

Explanation

Bacteria like Lactobacillus acidophilus, L. lactis and Streptococcus lactis commonly called lactic acid bacteria (LAB) are probiotics which check the growth of pathogenic microbes in the stomach and other parts of the digestive tract.

4. Which of the following preparation microbes are necessary?

  1. Yogurt
  2. Cheese
  3. Vinegar
  4. Milk
  5. 1, 2, 4
  6. 2, 3, 4
  7. 1, 2, 3
  8. All the above

Explanation

In everyday life, microbes and their products are used in the preparation of idli, dosa, cheese, curd, yogurt, dough, bread, vinegar, etc.,

5. LAB bacteria consumes which of the following protein?

  1. Casein
  2. Pepsin
  3. Renin
  4. Actin

Explanation

The LAB (lactic acid bacteria) grows in milk and convert it into curd, thereby digesting the milk protein casein.

6. What is the suitable temperature for conversion of Milk into curd?

  1. ≤ 40oC
  2. <40oC
  3. >40oC
  4. 40oC

Explanation

A small amount of curd added to fresh milk as a starter or inoculum contains millions of Lactobacilli, which under suitable temperature (≤40oC) multiply and convert milk into curd.

7. Which of the following statement is correct?

  1. Curd is more nutritious than milk
  2. Milk is more nutritious than curd
  3. Both Curd and Milk are equally nutritious
  4. None

Explanation

Curd is more nutritious than milk, because curd contains a large number of organic acids and vitamins.

8. ____ induce the growth or activity of beneficial microorganisms

  1. Probiotics
  2. Prebiotics
  3. Epitopes
  4. None

Explanation

Prebiotics are compounds in food (fibres) that induce the growth or activity of beneficial microorganisms.

9. Which of the following helps in restoring the gut flora?

  1. Probiotics
  2. Prebiotics
  3. Epitopes
  4. None

Explanation

Probiotics are live microorganisms intended to provide health benefits when consumed, generally by improving or restoring the gut flora.

10. _____ is produced as by-product in Yogurt preparation

  1. Acetic acid
  2. Lactic acid
  3. Acetaldehyde
  4. Acetylsalicylic acid

Explanation

Yogurt is produced by the bacterial fermentation of milk, and lactic acid is produced as a by-product.

11. The flavour in yogurt is due to______

  1. Acetaldehyde
  2. Acetic acid
  3. Lactic acid
  4. Formic acid

Explanation

Microorganisms such as Streptococcus thermophilus and Lactobacillus bulgaricus coagulate the milk protein and convert the lactose in the milk to lactic acid. The flavour in yogurt is due to acetaldehyde.

12. Which of the following statement is correct?

  1. Cheese is a dairy product produced in a wide range of flavours, textures and is formed by coagulation of the milk protein, casein
  2. During cheese production, milk is usually acidified and the enzyme rennet is added to cause coagulation
  3. Most cheese are made with a starter-bacteria, Lactococcus, Lactobacillus or Streptococcus.
  4. 1, 2
  5. 1, 3
  6. 2, 3
  7. All the above

Explanation

Cheese is a dairy product produced in a wide range of flavours, textures and is formed by coagulation of the milk protein, casein. During cheese production, milk is usually acidified and the enzyme rennet is added to cause coagulation. The solids are separated and pressed to form cheese. Most cheese are made with a starter – bacteria, Lactococcus, Lactobacillus or Streptococcus.

13. Cottage cheese is common in_____

  1. Asia
  2. South Asia
  3. South East Asia
  4. Nepal

Explanation

Paneer (cottage cheese) is fresh cheese common in South Asia, especially in India. It is made by curdling milk with lemon juice, vinegar and other edible acids.

14. Which bacteria is used in the production of Swiss cheese?

  1. Propionibacterium shermanii
  2. Streptococcus Lacti
  3. Lactobacillus
  4. All the above

Explanation

Large holes in Swiss cheese is due to the production of large amount of carbon-di-oxide by the bacterium Propionibacterium shermanii.

15. Which Bacteria can be used to ferment Idli Dough?

  1. Propionibacterium shermanii
  2. Streptococcus Lacti
  3. Leuconostoc mesenteroides
  4. Saccharomyces cerevisiae

Explanation

The dough used in the preparation of idlis and dosas are fermented by the bacteria Leuconostoc mesenteroides.

16. Which of the following is known as Baker’s yeast?

  1. Propionibacterium shermanii
  2. Streptococcus Lacti
  3. Leuconostoc mesenteroides
  4. Saccharomyces cerevisiae

Explanation

The dough which is used in bread making is fermented by Saccharomyces cerevisiae (Baker’s Yeast).

17. Glucose fermentation produces________

  1. Ethanol and CO2
  2. Ethyl Alcohol and O2
  3. Methyl Alcohol and CO2
  4. Methyl Alcohol and O2

Explanation

Fermentation of glucose mainly forms ethyl alcohol and carbon di-oxide, which is responsible for leavening of dough. When leavened dough is baked, both carbon-di-oxide and ethyl alcohol evaporate making the bread porous and soft.

18. Which of the following statement is correct?

  1. Single cell protein refers to edible unicellular microorganisms like Spirulina
  2. Protein extracts from pure or mixed cultures of algae, yeasts, fungi or bacteria may be used as ingredient or as a substitute for protein rich foods
  3. 1 alone
  4. 2 alone
  5. 1, 2
  6. None

Explanation

Single cell protein refers to edible unicellular microorganisms like Spirulina. Protein extracts from pure or mixed cultures of algae, yeasts, fungi or bacteria may be used as ingredient or as a substitute for protein rich foods and is suitable for human consumption or as animal feed.

19. Which of the following can be synthesized using microbes?

  1. Steroids
  2. Enzymes
  3. Biofuels
  4. Amino acids
  5. 1, 3, 4
  6. 1, 2, 4
  7. 2, 3, 4
  8. All the above

Explanation

Microbes are used to synthesize a number of products valuable to human beings. Products like beverages, antibiotics, organic acids, amino acids, vitamins, biofuels, single cell protein, enzymes, steroids, vaccines, pharmaceutical drugs, etc., are produced in industries.

20. Which of the following are necessary for Fermentor?

  1. Aeration
  2. Agitation
  3. pH
  4. Temperature
  5. 1, 2, 3
  6. 1, 3, 4
  7. 2, 3, 4
  8. All the above

Explanation

Production on a large scale requires growing microbes in very large vessels called fermentors. A fermentor (bioreactor) is a closed vessel with adequate arrangement for aeration, agitation, temperature, pH control and drain or overflow vent to remove the waste biomass of cultured microorganisms along-with their products.

21. What does the term anti – biotics mean?

  1. Against life
  2. Along life
  3. Supporting life
  4. Approving life

Explanation

Antibiotics are chemical substances produced by microorganisms which can kill or retard the growth of other disease-causing microbes even in low concentration. Antibiotic means “against life”.

22. Who was the 1st to use the term Antibiotic?

  1. Selman Waksman
  2. Alexander Cunningham
  3. Alexander Fleming
  4. Selman Fleming

Explanation

Antibiotics are used to treat diseases such as plague, meningitis, diphtheria, syphilis, leprosy, tuberculosis etc., Selman Waksman discovered Streptomycin and was the first to use the term “antibiotic” in 1943.

23. What coloured mould was discovered by Fleming while working on Staphylococci bacteria?

  1. Black
  2. Blue
  3. Green
  4. Red

Explanation

While working on Staphylococci bacteria, Alexander Fleming observed a green mould growing in one of his unwashed culture plates around which Staphylococci could not grow.

24. Who discovered the 1st antibiotic?

  1. Selman Waksman
  2. Alexander Cunningham
  3. Alexander Fleming
  4. Selman Fleming

Explanation

Alexander Fleming found that it was due to a chemical produced by the mould and he named it as penicillin, which was the first antibiotic discovered by Alexander Fleming in 1926.

25. Penicillin is produced by_______

  1. Bacteria
  2. Fungi
  3. Virus
  4. Algae

Explanation

Penicillin is produced by the fungi Penicillium notatum and Penicillium chrysogenum. It is bactericidal (antibiotics that kill bacteria) in action and inhibits the synthesis of the bacterial cell wall.

26. Which drug is known as Queen of Drugs?

  1. Amoxicillin
  2. Penicillin
  3. Azithromycin
  4. Cephalexin

Explanation

Penicillin is also referred as the “queen of drugs” and its full potential as an effective antibiotic was established much later by Earnest Chain and Howard Florey when they treated the wounded soldiers in World War II with penicillin.

27. Who among the following were awarded Nobel prize for discovery of Penicillin?

  1. Fleming
  2. Chain
  3. Florey
  4. 1 alone
  5. 1, 3
  6. 2, 3
  7. All the above

Explanation

Alexander Fleming, Chain and Florey were awarded the Nobel prize in 1945 for the discovery of penicillin.

28. Which of the following statement is correct?

  1. Antibiosis is the property of antibiotics to kill microorganisms
  2. Broad-spectrum antibiotics act against a wide range of disease-causing bacteria
  3. Narrow-spectrum antibiotics are active against a selected group of bacterial types.
  4. 1, 2
  5. 1, 3
  6. 2, 3
  7. All the above

Explanation

Antibiosis is the property of antibiotics to kill microorganisms. Broad-spectrum antibiotics act against a wide range of disease-causing bacteria. Narrow-spectrum antibiotics are active against a selected group of bacterial types.

29. Which of the following are the hypersensitivity reaction of Penicillin?

  1. Epilepsy
  2. Vomiting
  3. Cardiovascular collapse
  4. Wheezing
  5. 1, 2, 4
  6. 2, 3, 4
  7. 1, 3, 4
  8. 1, 2, 3

Explanation

Hypersensitivity reaction of Penicillin is a major problem with the use of penicillin, resulting in nausea, vomiting, wheezing and ultimately cardiovascular collapse.

30. Which of the following statement is correct?

  1. To check the sensitivity reaction, doctors use a needle to prick the forearm of the patients to give a weak dose of penicillin
  2. An itchy red region in the forearm is an indication that the patient is allergic to penicillin
  3. 1 alone
  4. 2 alone
  5. 1, 2
  6. None

Explanation

To check the sensitivity reaction, doctors use a needle to prick the forearm of the patients to give a weak dose of penicillin. An itchy red region in the forearm is an indication that the patient is allergic to penicillin. This test is important before administration of penicillin to a patient.

31. _____ is a broad-spectrum bacteriostatic antibiotic

  1. Tetracycline
  2. Fidaxomicin
  3. Sarecycline
  4. All the above

Explanation

Tetracycline is a broad- spectrum bacteriostatic antibiotic (antibiotics that limit the growth of bacteria) that inhibits microbial protein synthesis.

32. _______ is the first antibiotic of Tetracycline group

  1. Chlortetracycline
  2. Streptomycin
  3. Penicillin
  4. None

Explanation

Chlortetracycline is the first antibiotic of this group, isolated from the cultures of Streptomyces aureofaciens.

33. Which of the following statement is correct?

  1. Streptomycin is a broad-spectrum antibiotic isolated from the actinomycetes, Streptomyces griseus
  2. It is bactericidal against both gram positive and gram positive bacteria, especially against Mycobacterium tuberculosis.
  3. 1 alone
  4. 2 alone
  5. 1, 2
  6. None

Explanation

Streptomycin is a broad- spectrum antibiotic isolated from the actinomycetes, Streptomyces griseus. It is bactericidal against both gram positive and gram- negative bacteria, especially against Mycobacterium tuberculosis.

34. Which of the following statement is microbial products?

  1. Erythromycin
  2. Chloromycetin
  3. Griseofulvin
  4. Neomycin
  5. 1, 2, 4
  6. 1, 3, 4
  7. 2, 3, 4
  8. All the above

Explanation

Antibiotics, such as erythromycin, chloromycetin, griseofulvin, neomycin, kenamycin, bacitracin, etc., are also isolated as microbial products.

35. Which of the following statement is correct?

  1. Antibiotic resistance occurs when bacteria develop the ability to defeat the drug designed to kill or inhibit their growth
  2. Antibiotic resistance is accelerated by the misuse and over use of antibiotics, as well as poor infection prevention control
  3. 1 alone
  4. 2 alone
  5. 1, 2
  6. None

Explanation

Antibiotic resistance occurs when bacteria develop the ability to defeat the drug designed to kill or inhibit their growth. It is one of the most acute threat to public health. Antibiotic resistance is accelerated by the misuse and over use of antibiotics, as well as poor infection prevention control. Antibiotics should be used only when prescribed by a certified health professional.

36. ____ is a term used to describe strains of bacteria that are resistant to the majority of antibiotics

  1. Superbugs
  2. Bio- bug
  3. Neo- bug
  4. None

Explanation

“Superbug” is a term used to describe strains of bacteria that are resistant to the majority of antibiotics commonly used today.

37. Which of the following beverages can be produced using yeast?

  1. Wine
  2. Beer
  3. Brandy
  4. 1, 2
  5. 1, 3
  6. 2, 3
  7. All the above

Explanation

Microbes especially yeast is being used from time immemorial for the production of beverages like wine, beer, whisky, brandy and rum.

38. _______ is the oldest alcoholic beverage known.

  1. Rum
  2. Wine
  3. Beer
  4. Brandy

Explanation

Wine is among the oldest alcoholic beverages known and is produced by fermentation of fruit juice by yeast.

39. _____ is the applied science that deals with process of Bio-chemical process

  1. Oncology
  2. Oenology
  3. Pathology
  4. Zymology

Explanation

Zymology is an applied science which deals with the biochemical process of fermentation and its practical uses.

40. _______ is commonly called brewer’s yeast

  1. Zymomonas mobilis
  2. Sarcina ventriculi
  3. Saccharomyces cerevisiae
  4. Lactobacillus lacti

Explanation

Saccharomyces cerevisiae commonly called brewer’s yeast is used for fermenting malted cereals and fruit juices to produce various alcoholic beverages.

41. Which of the following beverage are obtained without distillation?

  1. Rum
  2. Wine
  3. Beer
  4. Whisky
  5. 1, 2
  6. 1, 3, 4
  7. 2, 3
  8. 1, 2, 4

Explanation

Wine and beer are produced without distillation, whereas whisky, brandy and rum are obtained by fermentation and distillation.

42. _______ effect is the inhibiting effect of oxygen on the fermentation process

  1. Bayour
  2. Pasteur
  3. Boyle
  4. Gay Luccas

Explanation

The Pasteur effect is the inhibiting effect of oxygen on the fermentation process.

43. _____ is the science and study of wine and wine making.

  1. Oncology
  2. Oenology
  3. Pathology
  4. Zymology

Explanation

Oenology is the science and study of wine and wine making. Wine is made from the fermentation of grape juice.

44. Grape juice is fermented by various strains of________ into alcohol

  1. Zymomonas mobilis
  2. Sarcina ventriculi
  3. Saccharomyces cerevisiae
  4. Lactobacillus lacti

Explanation

Grape juice is fermented by various strains of Saccharomyces cerevisiae into alcohol. Grape wine is of two types, red wine and white wine.

45. Which of the following are used in red wine making?

  1. Black grapes including skins and sometimes stems are used
  2. Green grapes including skins and sometimes stems are used
  3. Black grapes without skins and sometimes stems are used
  4. Green grapes without skins and sometimes stems are used

Explanation

For red wine, black grapes are used including skins and sometimes the stems also are used. In contrast white wine is produced only from the juice of either white or red grapes without their skin and stems.

46. Which of the following statement is correct?

  1. Beer is made from fermented sugarcane or molasses or directly from sugarcane juice by Saccharomyces cerevisiae
  2. Whisky is a type of distilled alcoholic beverage made from fermented grain mash by Saccharomyces cerevisiae
  3. 1 alone
  4. 2 alone
  5. 1, 2
  6. None

Explanation

Beer is produced from germinated barley malt grain by Saccharomyces carlsbergensis or Saccharomyces cerevisiae. Rum is made from fermented sugarcane or molasses or directly from sugarcane juice by Saccharomyces cerevisiae. Whisky is a type of distilled alcoholic beverage made from fermented grain mash by Saccharomyces cerevisiae.

47. Wine contains_____ percent alcohol

  1. 2 to 5
  2. 3 to 5
  3. 9 to 14
  4. 35 to 50

Explanation

Wine contains 9 to 14 percent alcohol. Wine coolers are made of wine mixed with carbonated water and flavourings. Wine coolers have about 4 to 6 percent alcohol.

48. Beer contains______ percent of alcohol.

  1. 2 to 5
  2. 3 to 5
  3. 9 to 14
  4. 35 to 50

Explanation

Beer contains 3 to 5 percent of alcohol. Distilled spirits such as whiskey, gin, scotch and vodka usually contain 35 to 50 percent alcohol.

49. From which of the following plants pathaneer is obtained?

  1. Coconut
  2. Mango
  3. Palm
  4. 1, 2
  5. 3 alone
  6. 1, 3
  7. All the above

Explanation

In some parts of South India, a traditional drink called pathaneer is obtained from fermenting sap of palms and coconut trees.

50. Which of the following statement is correct?

  1. A common source is tapping of unopened spadices of coconut
  2. It is a refreshing drink, which on boiling produces jaggery or palm sugar
  3. 1 alone
  4. 2 alone
  5. 1, 2
  6. None

Explanation

A common source is tapping of unopened spadices of coconut. It is a refreshing drink, which on boiling produces jaggery or palm sugar.

51. After____ hours toddy becomes unpalatable and is used for the production of vinegar

  1. 12
  2. 6
  3. 24
  4. 48

Explanation

When pathaneer is left undisturbed for few hours it gets fermented to form toddy with the help of naturally occurring yeast, to form a beverage that contains 4 percent alcohol. After 24 hours toddy becomes unpalatable and is used for the production of vinegar

52. ______ is the major producer of ethanol

  1. Zymomonas mobilis
  2. Sarcina ventriculi
  3. Saccharomyces cerevisiae
  4. Lactobacillus lacti

Explanation

Saccharomyces cerevisiae is the major producer of ethanol (C2H5OH). It is used for industrial, laboratory and fuel purposes. So, ethanol is referred to as industrial alcohol. Bacteria such as Zymomonas mobilis and Sarcina ventriculi are also involved in ethanol production.

53. The two most common type of biofuels in use today are_____

  1. Methanol
  2. Ethanol
  3. Biodiesel
  4. 1, 2
  5. 1, 3
  6. 2, 3
  7. All the above

Explanation

The two most common type of biofuels in use today are ethanol and biodiesel, both of them represent the first generation of biofuel technology.

54. _____ is often used as a fuel, mainly as a biofuel additive for gasoline

  1. Methanol
  2. Ethanol
  3. Methane
  4. All the above

Explanation

The process of ethanol production starts by milling a feed stock followed by the addition of dilute or fungal amylase (enzyme) from Aspergillus to break down the starch into fermentable sugars. Yeast is then added to convert the sugars to ethanol which is then distilled off to obtain ethanol which is up to 96 percent in concentration. Ethanol is often used as a fuel, mainly as a biofuel additive for gasoline.

55. From which of the following Biodiesel can be produced?

  1. Fats
  2. Vegetable oils
  3. Greases
  4. 1, 2
  5. 1, 3
  6. 2, 3
  7. All the above

Explanation

Biodiesel is a fuel made from vegetable oils, fats or greases. Biodiesel fuel can be used in diesel engines without altering the engine

56. When did Government of India approved the National Policy on Biofuels?

  1. December 2009
  2. December 2000
  3. September 2001
  4. September 2010

Explanation

The Government of India approved the National Policy on Biofuels in December 2009. Pure biodiesel is non-toxic, biodegradable and produces lower level of air pollutants than petroleum-based diesel fuel.

57. ____ is identified as most suitable oilseed for biodiesel production by Government of india

  1.  Jatropha curcas
  2. Sesamum indicum
  3. Solanum tuberosum
  4. Titika vitika

Explanation

The Government of India identified Jatropha curcas as the most suitable oilseed for biodiesel production. Pongamia species is also a suitable choice for production of biodiesel.

58. C6H12O6 🡪 2C2H5OH +_____ ↑

  1. CO2
  2. 2 O2
  3. 2 CO2
  4. O2

Explanation

59. World biofuel day is observed every year on____

  1. 1st October
  2. 10th August
  3. 10th September
  4. 1st December

Explanation

World biofuel day is observed every year on 10th August to create awareness about the importance of renewable bio-fuels as an alternative to conventional non-renewable fossil fuels. This day also highlights the various efforts taken by the Government in the biofuel sector.

60. Match the following

  1. Aspergillus niger 1. lactic acid
  2. Acetobacter aceti 2. citric acid
  3. Rhizopus oryzae 3. acetic acid
  4. Lactobacillus 4. fumaric acid
  5. 1, 3, 4, 2
  6. 2, 3, 4, 1
  7. 2, 4, 1, 3
  8. 3, 2, 1, 4

Explanation

Examples of organic acid producers are Aspergillus niger for citric acid, Acetobacter aceti for acetic acid, Rhizopus oryzae for fumaric acid, Clostridium butyricum for butyric acid and Lactobacillus for lactic acid.

61. Which of the following statement is correct?

  1. Yeast (Saccharomyces cerevisiae) and bacteria are used for commercial production of enzymes
  2. Lipases are used in detergent formulations and are used for removing oily stains from the laundry
  3. 1 alone
  4. 2 alone
  5. 1, 2
  6. None

Explanation

Yeast (Saccharomyces cerevisiae) and bacteria are used for commercial production of enzymes. Lipases are used in detergent formulations and are used for removing oily stains from the laundry.

62. Which of the following are clarified under Bottled juice?

  1. Pectinase
  2. Protease
  3. Cellulase
  4. 1, 2
  5. 1, 3
  6. 2, 3
  7. All the above

Explanation

Bottled juices are clarified by the use of pectinase, protease and cellulase. Lipases are used in detergent formulations and are used for removing oily stains from the laundry.

63. ____ can also be used to separate milk into solid curds for cheese making

  1. Pectinase
  2. Cellulase
  3. Protease
  4. Rennet

Explanation

Rennet can also be used to separate milk into solid curds for cheese making. Streptokinase produced by the bacterium Streptococcus and genetically engineered Streptococci are used as “clot buster” for removing clots from the blood vessels of patients who have undergone myocardial infarction.

64. Which of the following can be used as immunosuppressant used in organ transplantation?

  1. Cyclosporin A
  2. Cyclosporin B
  3. Penicillin
  4. Aspirin

Explanation

Cyclosporin A, an immunosuppressant used in organ transplantation is produced from the fungus Trichoderma polysporum.

65. Which of the following are the properties of Cyclosporin A?

  1. anti-inflammatory
  2. anti-fungal
  3. anti-parasitic
  4. 1, 2
  5. 1, 3
  6. 2, 3
  7. All the above

Explanation

Cyclosporin A is also used for important properties such as anti-inflammatory, antifungal and anti-parasitic.

66. Which of the following statement is correct?

  1. Statins produced by the yeast Monascus purpureus have been used to lower blood cholesterol levels
  2. Recombinant human insulin been produced predominantly using E.coli and Saccharomyces cerevisiae for therapeutic use in human
  3. 1 alone
  4. 2 alone
  5. 1, 2
  6. None

Explanation

Statins produced by the yeast Monascus purpureus have been used to lower blood cholesterol levels. Recombinant human insulin has been produced predominantly using E. coli and Saccharomyces cerevisiae for therapeutic use in human.

67. Which of the following statement is correct?

  1. Sewage is the waste generated every day in cities and towns containing human excreta.
  2. It contains large amounts of organic matter and microbes, which are pathogenic to humans and are bio-degradable pollutants
  3. Domestic waste consists of approximately 99 percent water, suspended solids and other soluble organic and inorganic substances
  4. 1, 2
  5. 1, 3
  6. 2, 3
  7. All the above

Explanation

Sewage is the waste generated every day in cities and towns containing human excreta. It contains large amounts of organic matter and microbes, which are pathogenic to humans and are bio-degradable pollutants. Domestic waste consists of approximately 99 percent water, suspended solids and other soluble organic and inorganic substances. Sewage should not be discharged directly into natural water bodies like rivers and streams. Before disposal, sewage should be treated in sewage treatment plants to make it less polluting.

68. Which of the following statement is incorrect?

  1. Microbes (mass of bacteria floc) are allowed to grow in aerated water (secondary treatment).
  2. They consume major part of organic matter in the effluent and reduce the BOD in the waste water
  3. 1 alone
  4. 2 alone
  5. 1, 2
  6. None

Explanation

Microbes (mass of bacteria floc) are allowed to grow in aerated water (secondary treatment). They consume major part of organic matter in the effluent and reduce the BOD in the waste water

69. Which of the following statement is correct?

  1. A microbial fuel cell is a bio-electrochemical system that drives an electric current by using bacteria and mimicking bacterial interaction found in nature
  2. Bacterial respiration is basically one big redox reaction in which electrons are being moved around.
  3. Microbes at the anode reduce the organic fuel generating protons
  4. 1, 2
  5. 1, 3
  6. 2, 3
  7. All the above

Explanation

A microbial fuel cell is a bio-electrochemical system that drives an electric current by using bacteria and mimicking bacterial interaction found in nature. Microbial fuel cells work by allowing bacteria to oxidize and reduce organic molecules. Bacterial respiration is basically one big redox reaction in which electrons are being moved around. A MFC consists of an anode and a cathode separated by a proton exchange membrane. Microbes at the anode oxidize the organic fuel generating protons which pass through the membrane to the cathode and the electrons pass through the anode to the external circuit to generate current.

70. Which of the following statement is correct?

  1. Biogas is a mixture of different gases produced by the breakdown of organic matter in the absence of oxygen
  2. Biogas is produced under aerobic condition
  3. Biogas can be produced from raw materials such as agricultural wastes, manure, municipal wastes, plant material, sewage, food waste, etc.
  4. 1, 2
  5. 1, 3
  6. 2, 3
  7. All the above

Explanation

Biogas is a mixture of different gases produced by the breakdown of organic matter in the absence of oxygen. Biogas can be produced from raw materials such as agricultural wastes, manure, municipal wastes, plant material, sewage, food waste, etc., Biogas is produced under anaerobic condition, when the organic materials are converted through microbiological reactions into gas and organic fertilizer.

71. Biogas primarily consists of____ % methane

  1. 50
  2. 63
  3. 70
  4. 34

Explanation

Biogas primarily consists of methane (63 percent), along with CO2 and hydrogen. Methane producing bacteria are called methanogens and one such common bacterium is Methano-bacterium.

72. Which of the following statement is correct?

  1. The Methanogens are also present in anaerobic sludge and rumen of cattle.
  2. In rumen, these bacteria help in the breakdown of cellulose.
  3. The excreta of cattle called dung is commonly called “Gobar”
  4. 1, 2
  5. 2, 3
  6. 1, 3
  7. All the above

Explanation

The Methanogens are also present in anaerobic sludge and rumen of cattle. In rumen, these bacteria help in the breakdown of cellulose. The excreta of cattle called dung is commonly called “Gobar”. Gobar gas is generated by the anaerobic decomposition of cattle dung. It consists of methane, CO2 with some hydrogen, nitrogen and other gases in trace amounts.

73. Which of the following developed technology of Bio-gas?

  1. IARI
  2. IGCAR
  3. KVIC
  4. 1, 2
  5. 1, 3
  6. 2, 3
  7. 3 alone

Explanation

Biogas is used for cooking and lighting. The technology of biogas production was developed in India mainly due to the efforts of Indian Agricultural Research Institute (IARI) and Khadi and Village Industries Commission (KVIC).

74. Which of the following statement is correct?

  1. Aerobic microbes degrade the pollutants in the presence of oxygen
  2. They mainly degrade pesticides and hydrocarbons
  3. Anurag Chetty obtained patent for this recombinant bacterial strain
  4. 1, 2
  5. 1, 3
  6. 2, 3
  7. All the above

Explanation

Aerobic microbes degrade the pollutants in the presence of oxygen. They mainly degrade pesticides and hydrocarbons. Pseudomonas putida is a genetically engineered microorganism (GEM). Ananda Mohan Chakrabarty obtained patent for this recombinant bacterial strain. It is multi- plasmid hydrocarbon-degrading bacterium which can digest the hydrocarbons in the oil spills.

75. ______ is currently tried for recycling of PET plastics

  1. Nitrosomonas europaea
  2. Ideonella sakaiensis
  3. Dechloromonas aromatica
  4. Phanerochaete chrysosporium

Explanation

Ideonella sakaiensis is currently tried for recycling of PET plastics. These bacteria use PETase and MHETase enzymes to breakdown PET plastic into terephthalic acid and ethylene glycol

76. _________ is also capable of degrading benzene

  1. Nitrosomonas europaea
  2. Ideonella sakaiensis
  3. Dechloromonas aromatica
  4. Phanerochaete chrysosporium

Explanation

Nitrosomonas europaea is also capable of degrading benzene and a variety of halogenated organic compounds including trichloroethylene and vinyl chloride.

77. _______ has the ability to degrade benzene anaerobically and to oxidize toluene and xylene

  1. Nitrosomonas europaea
  2. Ideonella sakaiensis
  3. Dechloromonas aromatica
  4. Phanerochaete chrysosporium

Explanation

Anaerobic microbes degrade the pollutants in the absence of oxygen. Dechloromonas aromatica has the ability to degrade benzene anaerobically and to oxidize toluene and xylene.

78. ______an anaerobic fungus exhibits strong potential for bioremediation of pesticides

  1. Nitrosomonas europaea
  2. Ideonella sakaiensis
  3. Dechloromonas aromatica
  4. Phanerochaete chrysosporium

Explanation

Phanerochaete chrysosporium an anaerobic fungus exhibits strong potential for bioremediation of pesticides, polyaromatic hydrocarbons dyes, trinitrotoluene, cyanides, carbon tetrachloride, etc., Dehalococcoides species are responsible for anaerobic bioremediation of toxic trichloroethene to non-toxic ethane.

Leave a Reply

Your email address will not be published. Required fields are marked *

Back to top button
error: Content is protected !!